Está en la página 1de 144

| Labor Relations-2A

ACOSTA members of their immediate


family shall be employed by
SAN JOSE CITY ELECTRIC SERVICE the cooperative
COOPERATIVE, INC. (SAJELCO) V.
The above-cited provision, however,
MINISTRY OF LABOR AND mentions two types of employees, namely:
EMPLOYMENT and MAGKAISA-ADLO the members-consumers and the members
May 31, 1969 of their immediate families. As regards
employees of SAJELCO who are members-
FACTS: consumers, the rule is settled that they are
Private respondent MAGKAISA-ADLO filed a not qualified to form, join or assist labor
petition for direct certification election with organizations for purposes of collective
the Regional Office No. 111 of the Department bargaining. The reason for withholding
of Labor and Employment in San Fernando,
from employees of a cooperative who are
members-co-owners the right to collective
Pampanga which was opposed by SALJECO on
bargaining is clear: an owner cannot
the ground that the employees who sought to
bargain with himself. However, employees
be represented by private respondent
who are not members-consumers may
(MAGKAISA-ADLO) are members-consumers
form, join or assist labor organizations
of the Cooperative itself and at the same time for purposes of collective bargaining
members of the General Assembly which, notwithstanding the fact that employees
pursuant to the By-laws is also the final arbiter of of SAJELCO who are not members-
any dispute arising in the Cooperative and consumers were employed ONLY
therefore cannot fairly and prudently represent because they are members of the
such opposing personalities that merge into one immediate family of members-
juridical or natural person. consumers. The fact remains that they are
not themselves members-consumers, and
The Med-Arbiter who was assigned to the case as such, they are entitled to exercise the
issued an Order granting the petition for direct rights of all workers to organization,
certification election on the basis of the collective bargaining, negotiations and
pleadings filed. On appeal, Respondent Director
others as are enshrined in Section 8, Article
of the Bureau of Labor Relations sustained the
ruling of the Med-Arbiter. Accordingly, a direct
III and Section 3, Article XIII of the 1987
certification election was conducted in Constitution, Labor Code of the Philippines
SALJECO, there being no restraining order from and other related laws (Cooperative Rural
thie Court enjoining the holding thereof. Hence, Bank of Davao City, Inc., supra, p. 10).
the instant petition.
The assailed Order of respondent Pura
ISSUE: Whether or not the employees- Ferrer-Calleja, Director of the Bureau of
members of an electric cooperative can Labor Relations was MODIFIED to the
organize themselves for purposes of effect that only the rank-and-file
collective bargaining. employees of petitioner who are not its
members-consumers are entitled to self-
HELD: NO. SAJELCO claims that its organization, collective bargaining, and
employees are also members of the negotiations, while other employees who
cooperative. It cited Section 17(18) of its By- are members-consumers thereof cannot
laws which declares that: enjoy such right. The direct certification
The Board shall also create election conducted on April 13, 1987 is
positions for subordinate hereby set aside.
employees and fix their
duties and remunerations.
Only member-consumers or

1
| Labor Relations-2A

BARRIETA unless the Appellee could achieve a


satisfactory arrangement with the Union, the
VICTORIANO V. ELIZALDE ROPE Company would be constrained to dismiss
WORKERS UNION him from the service. This prompted the
September 12, 1974 appellee to file a motion for injunction to
PONENTE: Zaldivar, J.: enjoin the Company and the Union from
dismissing him.
FACTS:
Benjamin Victoriano (hereinafter referred to In its answer, the Union contended, among
as Appellee), a member of the religious sect others, that Republic Act No. 3350 infringes
known as the "Iglesia ni Cristo", had been in on the fundamental right to form lawful
the employ of the Elizalde Rope Factory Inc. associations; that "the very phraseology of
(hereinafter referred to as Company) since said Republic Act 3350, that 'membership in
1958. As such employee, he was a a labor organization is banned to all those
member of the Elizalde Rope Workers' belonging to such religious sect prohibiting
Union (hereinafter referred to as Union) affiliation with any labor organization’”,
which had with the Company a collective "prohibits all the members of a given
bargaining agreement containing a closed religious sect from joining any labor union if
shop provision which reads as follows: such sect prohibits affiliations of their
members thereto”; and, consequently,
"Membership in the Union shall be required deprives said members of their
as a condition of employment for all constitutional right to form or join lawful
permanent employees workers covered by associations or organizations guaranteed by
this Agreement.” The collective bargaining the Bill of Rights, and thus becomes
agreement expired on March 3, 1964 but obnoxious to Article III, Section 1 (6) of the
was renewed the following day, March 4, 1935 Constitution.
1964.
ISSUE:
Under Section 4 (a), paragraph 4, of Whether or not R.A. No. 3350 violates the
Republic Act No. 875, prior to its constitutional provision on freedom of
amendment by Republic Act No. 3350, the association as it prohibits and bans the
employer was not precluded "from making members of religious sects that forbid
an agreement with a labor organization to affiliation of their members with labor unions
require as a condition of employment from joining labor unions.
membership therein, if such labor
organization is the representative of the HELD:
employees." On June 18, 1961, however, No. Appellant Union's contention that
Republic Act No. 3350 was enacted, Republic Act No. 3350 prohibits and bans
introducing an amendment to paragraph (4) the members of such religious sects that
subsection (a) of section 4 of Republic Act forbid affiliation of their members with labor
No. 875, as follows: . . "but such agreement unions from joining labor unions appears
shall not cover members of any religious nowhere in the wording of Republic Act No.
sects which prohibit affiliation of their 3350; neither can the same be deduced by
members in any such labor organization”. necessary implication therefrom. It is not
surprising, therefore, that appellant, having
Being a member of a religious sect that thus misread the Act, committed the error of
prohibits the affiliation of its members with contending that said Act is obnoxious to the
any labor organization, Appellee presented constitutional provision on freedom of
his resignation to appellant Union. The association. Both the Constitution and
management of the Company in turn Republic Act No. 875 recognize freedom of
notified Appellee and his counsel that association. Section 1 (6) of Article III of the

2
| Labor Relations-2A

Constitution of 1935, as well as Section 7 of limited. The legal protection granted to


Article IV of the Constitution of 1973, such right to refrain from joining is
provide that the right to form associations or withdrawn by operation of law, where a
societies for purposes not contrary to law labor union and an employer have agreed
shall not be abridged. Section 3 of Republic on a closed shop, by virtue of which the
Act No. 875 provides that employees shall employer may employ only members of the
have the right to self-organization and to collective bargaining union, and the
form, join or assist labor organizations of employees must continue to be members of
their own choosing for the purpose of the union for the duration of the contract in
collective bargaining and to engage in order to keep their jobs. Thus Section 4 (a)
concerted activities for the purpose of (4) of the Industrial Peace Act, before its
collective bargaining and other mutual aid or amendment by Republic Act No. 3350,
protection. What the Constitution and the provides that although it would be an unfair
Industrial Peace Act recognize and labor practice for an employer "to
guarantee is the "right" to form or join discriminate in regard to hire or tenure of
associations. Notwithstanding the different employment or any term or condition of
theories propounded by the different employment to encourage or discourage
schools of jurisprudence regarding the membership in any labor organization" the
nature and contents of a "right", it can be employer is, however, not precluded "from
safely said that whatever theory one making an agreement with a labor
subscribes to, a right comprehends at least organization to require as a condition of
two broad notions, namely: first, liberty or employment membership therein, if such
freedom, i.e., the absence of legal restraint, labor organization is the representative of
whereby an employee may act for himself the employees". By virtue, therefore, of a
without being prevented by law; and closed shop agreement, before the
second, power, whereby an employee may, enactment of Republic Act No. 3350, if any
as he pleases, join or refrain from joining an person, regardless of his religious beliefs,
association. It is, therefore, the employee wishes to be employed or to keep his
who should decide for himself whether he employment, he must become a member of
should join or not an association; and the collective bargaining union. Hence, the
should he choose to join, he himself makes right of said employee not to join the labor
up his mind as to which association he union is curtailed and withdrawn.
would join; and every after he has joined, he
still retains the liberty and the power to To that all embracing coverage of the
leave and cancel his membership with said closed shop arrangement, Republic Act No.
organization at any time. It is clear, 3350 introduced an exception, when it
therefore, that the right to join a union added to Section 4 (a) (4) of the Industrial
includes the right to abstain from joining any Peace Act the following proviso: "but such
union. Inasmuch as what both the agreement shall not cover members of any
Constitution and the Industrial Peace Act religious sects which prohibit affiliation of
have recognized, and guaranteed to the their members in any such labor
employee, is the "right" to join associations organization". Republic Act No. 3350
of his choice, it would be absurd to say that merely excludes ipso jure from the
the law also imposes, in the same breath, application and coverage of the closed shop
upon the employee the duty to join agreement the employees belonging to any
associations. The law does not enjoin an religious sects which prohibit affiliation of
employee to sign up with any association. their members with any labor organization.
What the exception provides, therefore, is
The right to refrain from joining labor that members of said religious sects cannot
organizations recognized by Section 3 of be compelled or coerced to join labor unions
the Industrial Peace Act is, however, even when said unions have closed shop

3
| Labor Relations-2A

agreements with the employers; that in spite BATALLER


of any closed shop agreement, members of
said religious sects cannot be refused KAPATIRAN SA MEAT AND CANNING
employment or dismissed from their jobs on DIVISION vs. CALLEJA
the sole ground that they are not members
G.R. No. 82914, June 20 1988
of the collective bargaining union. It is
clear, therefore, that the assailed Act, far
from infringing the constitutional provision FACTS:
on freedom of association, upholds and From 1984 to 1987 Petitioner Kapatiran sa
reinforces it. It does not prohibit the Meat and Canning Division (TUPAS) was
members of said religious sects from the sole and exclusive collective bargaining
affiliating with labor unions. It still leaves to representative of the workers in the Meat
said members the liberty and the power to
and Canning Division of the Universal
affiliate, or not to affiliate, with labor unions.
If, notwithstanding their religious beliefs, the Robina Corporation, with a 3-year collective
members of said religious sects prefer to bargaining agreement (CBA) which was to
sign up with the labor union, they can do expire on November 15, 1987. Within the
so. If in deference and fealty to their freedom period of 60 days prior to the
religious faith, they refuse to sign up, they expiration of its CBA, TUPAS filed an
can do so; the law does not coerce them to amended notice of strike as a means of
join; neither does the law prohibit them from
pressuring the company to extend, renew,
joining; and neither may the employer or
labor union compel them to join. Republic or negotiate a new CBA with it. On October
Act No. 3350, therefore, does not violate the 8, 1987, the NEW ULO, composed mostly
constitutional provision on freedom of of workers belonging to the IGLESIA NI
association. KRISTO sect, registered as a labor union.
On October 12, 1987, the TUPAS staged a
strike. ROBINA obtained an injunction
against the strike, resulting in an agreement
to return to work and for the parties to
negotiate a new CBA. The next day NEW
ULO filed a petition for a certification
election at the Bureau of Labor Relations.
TUPAS moved to dismiss the petition
claiming among others that the members of
the NEW ULO were mostly members of the
Iglesia ni Kristo sect which three years
previous refused to affiliate with any labor
union. It also accused the company of using
the NEW ULO to defeat TUPAS’ bargaining
rights. The Med-Arbiter ordered the holding
of a certification election. TUPAS appealed
to the Bureau of Labor Relations. In the
meantime, it was able to negotiate a new 3-
year CBA with ROBINA, which was signed
on December 3, 1987 and to expire on
November 15, 1990. On January 27, 1988,

4
| Labor Relations-2A

respondent BLR Director Calleja dismissed


the appeal. CANAPE

ISSUE: UP vs. FERRER-CALLEJA


W/N members of a sect who are not allowed G.R. No. 96189, July 14, 1992
by their religion to join a labor union may
form their own union? FACTS:
The case was initiated in the Bureau of
HELD: Labor Relations by a petition filed on March
2, 1990 by a registered labor union, the
YES. This Court’s decision in Victoriano vs.
"Organization of Non-Academic Personnel
Elizalde Rope Workers’ Union, 59 SCRA of UP" (ONAPUP). Claiming to have a
54, upholding the right of members of the membership of 3,236 members —
IGLESIA NI KRISTO sect not to join a labor comprising more than 33% of the 9,617
union for being contrary to their religious persons constituting the non-academic
beliefs, does not bar the members of that personnel of UP-Diliman, Los Baños,
sect from forming their own union. The Manila, and Visayas, it sought the holding of
a certification election among all said non-
public respondent correctly observed that
academic employees of the University of the
the “recognition of the tenets of the sect Philippines. At a conference thereafter held
… should not infringe on the basic right on March 22, 1990 in the Bureau, the
of self-organization granted by the University stated that it had no objection to
constitution to workers, regardless of the election.
religious affiliation.”
On April 18, 1990, another registered labor
union, the "All UP Workers' Union," filed a
The fact that TUPAS was able to negotiate
comment, as intervenor in the certification
a new CBA with ROBINA within the 60-day election proceeding. Alleging that its
freedom period of the existing CBA, does membership covers both academic and
not foreclose the right of the rival union, non-academic personnel, and that it aims to
NEW ULO, to challenge TUPAS’ claim to unite all UP rank-and-file employees in one
majority status, by filing a timely petition for union, it declared its assent to the holding of
the election provided the appropriate
certification election before TUPAS’ old
organizational unit was first clearly defined.
CBA expired and before it signed a new It observed in this connection that the
CBA with the company. As pointed out by Research, Extension and Professional Staff
Med-Arbiter Abdullah, a “certification (REPS), who are academic non-teaching
election is the best forum in ascertaining personnel, should not be deemed part of the
the majority status of the contending organizational unit.
unions wherein the workers themselves For its part, the University, through its
General Counsel, made of record its view
can freely choose their bargaining
that there should be two (2) unions: one for
representative thru secret ballot.” Since it academic, the other for non-academic or
has not been shown that this order is tainted administrative, personnel considering the
with unfairness, this Court will not thwart the dichotomy of interests, conditions and rules
holding of a certification election governing these employee groups.
(Associated Trade Unions [ATU] vs. Noriel,
88 SCRA 96). Director Calleja ruled on the matter on
August 7, 1990. She declared that "the
appropriate organizational unit should

5
| Labor Relations-2A

embrace all the regular rank-and-file or managerial or whose


employees, teaching and non-teaching, of duties are of a highly
the University of the Philippines, including confidential nature shall not
all its branches" and that there was no be eligible to join the
sufficient evidence "to justify the grouping of organization of rank-and file
the non-academic or administrative government employees;
personnel into an organization unit apart
and distinct from that of the academic or 2) In the University hierarchy, not all
teaching personnel." Director Calleja teaching and non-teaching personnel
adverted to Section 9 of Executive Order belong the rank-and file: just as there are
No. 180 those occupying managerial positions within
the non-teaching roster, there is also a
The Director thus commanded that a dichotomy between various levels of the
certification election be "conducted among teaching or academic staff;
rank-and-file employees, teaching and non-
teaching" in all four autonomous campuses 3) Among the non-teaching employees
of the UP, and that management appear composed of Administrative Staff and
and bring copies of the corresponding Research personnel, only those holding
payrolls for January, June, and July, 1990 at positions below Grade 18 should be
the "usual pre-election conference . . ." regarded as rank-and-file, considering that
those holding higher grade positions, like
At the pre-election conference held on Chiefs of Sections, perform supervisory
March 22, 1990 at the Labor Organizational functions including that of effectively
Division of the DOLE, 8 the University recommending termination of appointments
sought further clarification of the coverage or initiating appointments and promotions;
of the term, "rank-and-file" personnel, and
asserting that not every employee could
properly be embraced within both teaching 4) Not all teaching personnel may be
and non-teaching categories since there are deemed included in the term, "rank-and-
those whose positions are in truth file;" only those holding appointments at the
managerial and policy-determining, and instructor level may be so considered,
hence, excluded by law. because those holding appointments from
Assistant Professor to Associate Professor
At a subsequent hearing (on October 4, to full Professor take part, as members of
1990), the University filed a Manifestation the University Council, a policy-making
seeking the exclusion from the body, in the initiation of policies and rules
organizational unit of those employees with respect to faculty tenure and
holding supervisory positions among non- promotion.
academic personnel, and those in teaching
staff with the rank of Assistant Professor or The ONAPUP quite categorically made of
higher, submitting the following as grounds record its position; that it was not opposing
therefor: the University's proferred classification of
rank-and file employees. On the other hand,
1) Certain "high-level employees" with the "All UP Workers' Union" opposed the
policy-making, managerial, or confidential University's view, in a Position Paper
functions, are ineligible to join rank-and-file presented by it under date of October 18,
employee organizations under Section 3, 1990.
EO 180:
Sec. 3. High-level employees Director Calleja subsequently promulgated
whose functions are normally an Order dated October 30, 1990, resolving
considered as policy-making the "sole issue" of "whether or not

6
| Labor Relations-2A

professors, associate professors and to the Board of Regents the


assistant professors are included in the conferment of degrees, and
definition of high-level employee(s)" in light disciplinary power over
of Rule I, Section (1) of the Implementing students. The policy-
Guidelines of Executive Order No. 180, determining functions
defining "high level employee" as follows: contemplated in the definition
of a high-level employee
1. High Level Employee — is pertain to managerial,
one whose functions are executive, or organization
normally considered policy policies, such as hiring, firing,
determining, managerial or and disciplining of
one whose duties are highly employees, salaries,
confidential in nature. A teaching/working hours,
managerial function refers to other monetary and non-
the exercise of powers such monetary benefits, and other
as: terms and conditions of
1. To employment. They are the
effectively usual issues in collective
recommend bargaining negotiations so
such that whoever wields these
managerial powers would be placed in a
actions; situation of conflicting
2. To interests if he were allowed
formulate or to join the union of rank-and-
execute file employees.
management
policies and The University seasonably moved for
decisions; or reconsideration, seeking to make the
3. To hire, following points, to wit:
transfer,
suspend, lay- 1) UP professors do "wield the most potent
off, recall, managerial powers: the power to rule on
dismiss, tenure, on the creation of new programs
assign or and new jobs, and conversely, the abolition
discipline of old programs and the attendant re-
employees. assignment of employees.

The Director adjudged that said teachers 2) To say that the Council is "limited to
are rank-and-file employees "qualified to (acting on) academic matters" is error, since
join unions and vote in certification academic decisions "are the most important
elections." According to her — decisions made in a University . . (being, as
A careful perusal of the it were) the heart, the core of the University
University Code . . shows as a workplace.
that the policy-making
powers of the Council are 3) Considering that the law regards as a
limited to academic matters, "high level" employee, one who performs
namely, prescribing courses either policy-determining, managerial, or
of study and rules of confidential functions, the Director erred in
discipline, fixing student applying only the "managerial functions"
admission and graduation test, ignoring the "policy-determining
requirements, recommending functions" test.

7
| Labor Relations-2A

because 'the basic test of an asserted


4) The Director's interpretation of the law bargaining unit's acceptability is whether or
would lead to absurd results, e.g.: "an not it is fundamentally the combination
administrative officer of the College of Law which will best assure to all employees the
is a high level employee, while a full exercise of their collective bargaining rights'
Professor who has published several (Rothenberg on Labor Relations, 490)."
treatises and who has distinguished himself Hence, in that case, the Court upheld the
in argument before the Supreme Court is a trial court's conclusion that two separate
mere rank-and-file employee. A dormitory bargaining units should be formed, one
manager is classified as a high level consisting of regular and permanent
employee, while a full Professor or Political employees and another consisting of casual
Science with a Ph. D. and several Honorary laborers or stevedores.
doctorates is classified as rank-and-file."
In the case at bar, the University
ISSUE: Whether or not they, and other employees may, as already suggested,
employees performing academic quite easily be categorized into two general
12
functions, should comprise a collective classes: one, the group composed of
bargaining unit distinct and different from employees whose functions are non-
that consisting of the non-academic academic, i.e., janitors, messengers, typists,
employees of the University, 13 considering clerks, receptionists, carpenters,
the dichotomy of interests, conditions and electricians, grounds-keepers, chauffeurs,
rules existing between them. mechanics, plumbers; and two, the group
made up of those performing academic
HELD: functions, i.e., full professors, associate
The Supreme Court applied the professors, assistant professors, instructors
principles laid down in Democratic Labor — who may be judges or government
Association vs. Cebu Stevedoring executives — and research, extension and
Company, Inc ( 103 Phil. 1103). In the said professorial staff. Not much reflection is
case, the Supreme Court said that there are needed to perceive that the community or
various factors which must be satisfied and mutuality of interests which justifies the
considered in determining the proper formation of a single collective bargaining
constituency of a bargaining unit. No one unit is wanting between the academic and
particular factor is itself decisive of the non-academic personnel of the university. It
determination. The weight accorded to any would seem obvious that teachers would
particular factor varies in accordance with find very little in common with the University
the particular question or questions that clerks and other non-academic employees
may arise in a given case. What are these as regards responsibilities and functions,
factors? Rothenberg mentions a good working conditions, compensation rates,
number, but the most pertinent to our case social life and interests, skills and
are: (1) will of the employees (Globe intellectual pursuits, cultural activities, etc.
Doctrine); (2) affinity and unit of employees' On the contrary, the dichotomy of interests,
interest, such as substantial similarity of the dissimilarity in the nature of the work
work and duties, or similarity of and duties as well as in the compensation
compensation and working conditions; (3) and working conditions of the academic and
prior collective bargaining history; and (4) non-academic personnel dictate the
employment status, such as temporary, separation of these two categories of
seasonal probationary employees. employees for purposes of collective
bargaining. The formation of two separate
The Court further explained that "the bargaining units, the first consisting of the
test of the grouping is community or rank-and-file non-academic personnel, and
mutuality of interests. And this is so the second, of the rank-and-file academic

8
| Labor Relations-2A

employees, is the set-up that will best


assure to all the employees the exercise of
their collective bargaining rights. These
special circumstances, i.e., the dichotomy of
interests and concerns as well as the
dissimilarity in the nature and conditions of
work, wages and compensation between
the academic and non-academic personnel,
bring the case at bar within the exception
contemplated in Section 9 of Executive
Order No. 180. It was grave abuse of
discretion on the part of the Labor Relations
Director to have ruled otherwise, ignoring
plain and patent realities.

The Order of August 7, 1990 is


MODIFIED in the sense that the non-
academic rank-and-file employees of the
University of the Philippines shall constitute
a bargaining unit to the exclusion of the
academic employees of the institution —
i.e., full professors, associate professors,
assistant professors, instructors, and the
research, extension and professorial staff,
who may, if so minded, organize
themselves into a separate collective
bargaining unit; and that, therefore, only
said non-academic rank-and-file personnel
of the University of the Philippines in
Diliman, Manila, Los Baños and the Visayas
are to participate in the certification election.

9
| Labor Relations-2A

CARILLO to the elections above adverted to by


ratifying and registering the same, and
GENARO BAUTISTA, petitioner, vs. HON. recognizing as officers the persons
COURT OF APPEALS and THE supposedly elected; and for the latter to
OFFICIALS AND BOARD OF DIRECTORS refrain from assuming office and acting as
OF KAISAHAN AT KAPATIRAN NG MGA officers of the KKMK-MWSS. However on
MANGGAGAWA AT KAWANI SA 09 October 1995, a Decision was rendered
METROPOLITAN WATERWORKS AND by the Court of Appeals finding for the
SEWERAGE SYSTEM UNION(KKMK- private respondents, upholding that the BLR
MWSS), REPRESENTED BY ITS had jurisdiction over an intra-union dispute.
PRESIDENT, PRUDENCIO CRUZ,
respondents. Issue: WON the BLR has jurisdiction to call
for and conduct the election of officers of an
Facts: employee’s association in the public sector?
A Petition for Prohibition with Prayer for a
Temporary Restraining Order/Injunction was Ruling: YES.
filed by Genaro Bautista, et al., against Insofar as power to call for and supervise
Perlita Bathan-Velasco, Director, Eugenia the conduct of certification elections is
Fernandez, Med-Arbiter, and Johnny P. concerned, we rule against the petitioner.
Garcia, Chief, Labor Organizations Division, The authority of the BLR in assuming
all of the BLR, before the Regional Trial jurisdiction over a certification election, or
Court (RTC), Quezon City, Branch 87. The any inter-union or intra-union conflicts, is
petition sought to enjoin the herein found in Article 226 of the Labor Code of the
respondents from proceeding with the Philippines, which reads: ART. 226.
election of officers of KKMK-MWSS BUREAU OF LABOR RELATIONS. – The
scheduled on 02 December 1993, and to Bureau of Labor Relations and the Labor
permanently prohibit them from exercising Relations Division in the regional offices of
jurisdiction over the conduct of election of the Department of Labor shall have original
the officers of the KKMK-MWSS. On 26 and exclusive authority to act, at their own
November 1993, the RTC, Quezon City, initiative or upon request of either or both
Branch 87, through Judge Elsie Ligot Telan, parties, on all inter-union and intra-union
issued a temporary restraining order On 02 conflicts, and all disputes, grievances or
December 1993, the election of the officers problems arising from or affecting labor-
of KKMK-MWSS pushed through despite management relations in all workplaces
the issuance of the temporary restraining whether agricultural or nonagricultural,
order. Another Order was issued by Branch except those arising from the
87 on the same date. On 29 December implementation or interpretation of collective
1993, a Writ of Preliminary Injunction was bargaining agreements which shall be the
issued by the RTC, the pertinent portion of subject of grievance procedure and/or
which reads: NOW THEREFORE, you the voluntary arbitration. The Bureau shall have
respondents, your agents and fifteen (15) working days to act on labor
representatives, particularly the officers cases before it, subject to extension by
concerned ordering them until further orders agreement of the parties. It is quite clear
of this Court to refrain from giving any effect from this provision that BLR has the original

10
| Labor Relations-2A

and exclusive jurisdiction on all inter-union CASTRO


and intra-union conflicts. An intra-union
conflict would refer to a conflict within or PAMANTASAN NG LUNGSOD NG
MAYNILA V. CIVIL SERVICE
inside a labor union, and an inter-union
COMMISSION
controversy or dispute, one occurring or G.R. No. 107590, February 21, 1995
carried on between or among unions. The
subject of the case at bar, which is the FACTS:
election of the officers and members of the The 16 individual private
board of KMKK-MWSS, is, clearly, an intra- respondents were full-time instructors of
union conflict, being within or inside a labor PLM under "temporary contracts" of
employment renewable on a yearly basis.
union. It is well within the powers of the BLR
They joined the Pamantasan ng Lungsod ng
to act upon. The petitioner is asking us to Maynila Faculty Organization. Uniform
make an illogical edict by declaring that our notices of termination were individually sent
ruling in the ACAE case, considering that it to private respondents informing them of the
involved an inter-union conflict, should not expiration of their temporary appointments
apply to the instant case for the reason that and the non-renewal of their appointments
the latter involves an intra-union conflict. for the school year (SY) 1990-1991. A
series of letter-complaints addressed to the
This, we cannot do because the law is very
CSC by private respondents evoked a
clear on this matter. letter-response from PLM, traversing the
complainants' right to compel a renewal of
the appointments. They were advised that
their retention was not recommended by
their respective Deans. Private
respondents, through PLMFO, filed with the
CSC a verified complaint for illegal
dismissal and unfair labor practice against
petitioner and its officers. Public respondent
CSC referred the case to the Public Sector
Labor-Management Council. The latter
found petitioner guilty of unfair labor
practice and held that private respondents
should be reinstated. CSC sustained the
findings of PSLMC and directed
reinstatement with back salaries of private
respondents.

ISSUE:
Whether or not petitioner is guilty of
unfair labor practice.

RULING:
Yes. Employees who hold temporary
contracts of employment may not expect
renewal of appointment as a matter of right
since the decision is a management
prerogative. However, when the exercise of
this privilege is alleged to be the means by
which management hinders unionism or

11
| Labor Relations-2A

outrightly bust unions and such allegation is DELAROSA-REID


supported by evidence, the act needs to be
examined and studied. Had complainants UNITED PEPSI-COLA SUPERVISORY
not been among those active officers and/or UNION (UPSU), petitioner, vs. HON.
members of the PLMFO, and had their BIENVENIDO E. LAGUESMA and PEPSI-
qualifications, training, experience and COLA PRODUCTS, PHILIPPINES,
performance rating not been impressive, the INC. respondents.
Commission would have agreed that the [G.R. No. 122226. March 25, 1998]
termination or non-renewal of the contracts
of complainants does not constitute unfair FACTS:
labor practice. Hence, there is no reason for Petitioner is a union of supervisory
PLM Management to terminate the services employees. They filed a petition for
of these employees except to bust their certification election on behalf of the route
organization. managers at Pepsi-Cola Products
Philippines, Inc. However, its petition was
Even temporary employees enjoy denied by the med-arbiter and, on appeal,
that basic right to form organization or by the Secretary of Labor and Employment,
association for purposes not contrary to law. on the ground that the route managers are
PLMFO is that organization. Thus, its managerial employees and, therefore,
members cannot be separated from the ineligible for union membership pursuant to
service for the simple reason of Art. 245 of the Labor Code, which provides:
membership in the said organization. And
when the appointment status of these “Ineligibility of managerial employees to join
members happens to be temporary in any labor organization; right of supervisory
nature, such becomes merely incidental and employees. — Managerial employees are
the doctrine that temporary employees have not eligible to join, assist or form any labor
no security of tenure must yield or is not organization. Supervisory employees shall
applicable. When the clear intent therefore not be eligible for membership in a labor
of PLM Management in terminating the organization of the rank-and-file employees
services of these employees is to abridge but may join, assist or form separate labor
their constitutional right to self-organization, organizations of their own.”
the Commission has the duty to give them
protection and uphold their basic right. This Petitioner brought this suit challenging the
constitutional right of employees is superior validity of the order of the Secretary of
to the right of management not to renew the Labor and Employment. Its petition was
temporary appointment of its employees. dismissed by the Third Division for lack of
When the exercise of discretion by the showing that respondent committed grave
management is calculated to bust the union abuse of discretion. But petitioner filed a
as what PLM Management had done, the motion for reconsideration, pressing for
Commission has no choice but to declare it resolution its contention that the first
as a grave abuse of discretion. sentence of Art. 245 of the Labor Code, so
far as it declares managerial employees to
be ineligible to form, assist or join unions,
contravenes the constitution which provides
in Art. III, Section 8 that:

The right of the people, including those


employed in the public and private sectors,
to form unions, associations, or societies for
purposes not contrary to law shall not be
abridged.

12
| Labor Relations-2A

the organization’s interactions with its


Citing the Court’s ruling in Nasipit Lumber environment. Typical titles of top managers
Co. v. National Labor Relations are “chief executive officer,” “president,” and
Commission, petitioner argues that previous “senior vice-president.” Actual titles vary
administrative determinations of the NLRC from one organization to another and are
do not have the effect of res judicata in this not always a reliable guide to membership
case, because “labor relations proceedings” in the highest management classification.
are “non-litigious and summary in nature
without regard to legal technicalities.” A distinction exists between those who have
the authority to devise, implement and
ISSUE: Whether or not the route managers control strategic and operational policies
at Pepsi-Cola Products Philippines, Inc. are (top and middle managers) and those
managerial employees whose task is simply to ensure that such
policies are carried out by the rank-and-file
HELD: Managers constitute three levels of employees of an organization (first-level
a pyramid: (1) Top management; (2) Middle managers/supervisors). What distinguishes
Management; and (3) First-line them from the rank-and-file employees is
Management also called supervisors. that they act in the interest of the employer
in supervising such rank-and-file
FIRST-LINE MANAGERS — The lowest employees.
level in an organization at which individuals
are responsible for the work of others is “Managerial employees” may therefore be
called first-line or first-level management. said to fall into two distinct categories: the
First-line managers direct operating “managers” per se, who compose the
employees only; they do not supervise other former group described above, and the
managers. Examples of first-line managers “supervisors” who form the latter group.
are the “foreman” or production supervisor
in a manufacturing plant, the technical It appears that this question was the subject
supervisor in a research department, and of two previous determinations by the
the clerical supervisor in a large office. First- Secretary of Labor and Employment, in
level managers are often called supervisors. accordance with which this case was
decided by the med-arbiter. To qualify as
MIDDLE MANAGERS — The term middle managerial employee, there must be a clear
management can refer to more than one showing of the exercise of managerial
level in an organization. Middle managers attributes under paragraph (m), Article 212
direct the activities of other managers and of the Labor Code as amended.
sometimes also those of operating Designations or titles of positions are not
employees. Middle managers’ principal controlling. As to the route managers and
responsibilities are to direct the activities accounting manager, they are managerial
that implement their organizations’ policies employees as their job descriptions clearly
and to balance the demands of their reveal so in the case of Worker’s Alliance
superiors with the capacities of their Trade Union (WATU) v. Pepsi-Cola
subordinates. A plant manager in an Products Philippines, Inc.,
electronics firm is an example of a middle
manager.

TOP MANAGERS — Composed of a


comparatively small group of executives,
top management is responsible for the
overall management of the organization. It
establishes operating policies and guides

13
| Labor Relations-2A

DELEON Bureau of Labor Relations affirmed the


decision of the Labor Arbiter.
FRANKLIN BAKER COMPANY V.
TRAJANO
Petitioner’s Motion for Reconsideration was
G.R. No. 75039 January 28, 1988
denied by MOLE Director Cresencio
Trajano for lack of merit.
Doctrine: Employees who do not
participate in policy-making but are given
ready policies to execute and standard ISSUE:
practices to observe are not managerial Whether or not subject employees are
employees. managerial employees and thus not entitled
to join the union of rank and file employees.
FACTS:
Private respondent Franklin Baker HELD:
Brotherhood Association Board filed for NO. Art. 219 [212] (m) of the Labor Code
certification election among the office and defines managerial employees as “one who
technical employees of petitioner company is vested with the powers or prerogatives to
before the Ministry of Labor and lay down and execute management policies
Employment (MOLE). It alleges that and/or to hire, transfer, suspend, lay-off,
petitioner has 90 regular technical and office recall, discharge, assign or discipline
employees, which group is separate and employees. Supervisory employees are
distinct from the regular rank and file those who, in the interest of the employer,
employees and is excluded from the effectively recommend such managerial
coverage of the existing Collective actions xxx”
Bargaining Agreement.
It will be noted, however, that in the
Petitioner did not object to the holding of performance of their duties and functions
such an election, but manifested that out of and in the exercise of their recommendatory
the 90 employees sought to be represented powers, subject employees may only
by respondent union, 74 are managerial recommend, as the ultimate power to hire,
employees and 2 are confidential fire or suspend as the case may be, rests
employees, hence must be excluded from upon the plant personnel manager.
the certification election and from the
bargaining unit that may result from such The test of "supervisory" or "managerial
election. status" depends on whether a person
possesses authority to act in the interest of
The Labor Arbiter granted the petition and his employer in the matter specified in
ordered for the certification election. Article 212 (k) of the Labor Code and
Section 1 (m) of its Implementing Rules and
Petitioner appealed to the Bureau of Labor whether such authority is not merely
Relations, praying that the appealed order routinary or clerical in nature, but requires
may be set aside and another to be issued the use of independent judgment. Thus,
declaring the 74 inspectors, foremen and where such recommendatory powers as in
supervisors as managerial employees. The the case at bar, are subject to evaluation,
review and final action by the department

14
| Labor Relations-2A

heads and other higher executives of the DEL ROSARIO


company, the same, although present, are
not effective and not an exercise of PAPER INDUSTRIES CORP. VS HON.
independent judgment as required by law. LAGUESMA ET AL
GR No. 101738 April 12, 2000
Furthermore, in line with the ruling of the
Court, subject employees are not FACTS:
managerial employees because as borne by Petitioner PICOP is engaged in the
the records, they do not participate in policy manufacture of paper and timber products.
making but are given ready policies to More or less 487 of its supervisory and
execute and standard practices to observe, technical staff employees are signatory
thus having little freedom of action. members of the private respondent PICOP-
Bislig Supervisory and Technical Staff
Lastly, even if the Court regard the Employees Union (PBSTSEU) who had
employees concerned as "managerial instituted a Petition for Certification Election
employees," they can still join the union of to determine the sole and exclusive
the rank and file employees. They cannot bargaining agent of the supervisory and
however form their own exclusive union as technical staff employees of PICOP for CBA
"managerial employees" purposes. Meanwhile, other private
respondents Federation of Free Workers
(FFW) and Associated Labor Union (ALU)
filed their respective petitions for
intervention.

On September 14, 1989, Med-


Arbiter Arturo L. Gamolo issued an
Order holding of a certification election
among PICOP's supervisory and technical
staff employees in Tabon, Bislig, Surigao
del Sur, with four (4) choices, namely: (1)
PBSTSEU; (2) FFW; (3) ALU; and (4) no
union. However, during the pre-election
conference on January 18, 1990, PICOP
questioned and objected to the inclusion of
some section heads and supervisors in the
list of voters whose positions it averred were
reclassified as managerial employees in the
light of the reorganization effected by it
under its Revised Organizational Structure.
PICOP advanced the view that considering
the alleged present authority of these
section managers and unit managers to hire
and fire, they are classified as managerial

15
| Labor Relations-2A

employees, and hence, ineligible to form or


join any labor organization. In the petition before us, a thorough
dissection of the job description of the
Med-Arbiter Pura holds that the concerned supervisory employees and
supervisors and section heads of the section heads indisputably show that they
petitioner are managerial employees and are not actually managerial but only
therefore excluded from the list of voters for supervisory employees since they do not lay
purposes of certification election. Upon down company policies. PICOP's contention
appeal of the unions Bienvenido E. that the subject section heads and unit
Laguesma, acting as the then managers exercise the authority to hire and
Undersecretary of Labor, issued the fire is ambiguous and quite misleading for
assailed Order dated April 17, 1991 setting the reason that any authority they exercise
aside the Order of the Med-Arbiter and is not supreme but merely advisory in
declaring that the subject supervisors and character. Theirs is not a final determination
section heads are supervisory employees of the company policies inasmuch as any
eligible to vote in the certification election. action taken by them on matters relative to
Petitioner then sought before SC the hiring, promotion, transfer, suspension and
annulment of Hon. Laguesma’s Order. termination of employees is still subject to
confirmation and approval by their
ISSUE: Whether or not PICOP’s managerial respective superior.
employees may join unions and took part on
the certification election Finally, considering all the foregoing, the
fact that PICOP voiced out its objection to
HELD: the holding of certification election, despite
Managerial employees are ranked as Top numerous opportunities to ventilate the
Managers, Middle Managers and First Line same, only after respondent Undersecretary
Managers. Top and Middle Managers of Labor affirmed the holding thereof, simply
have the authority to devise, implement and bolstered the public respondents'
control strategic and operational policies conclusion that PICOP raised the issue
while the task of First-Line Managers is merely to prevent and thwart the concerned
simply to ensure that such policies are section heads and supervisory employees
carried out by the rank-and- file employees from exercising a right granted them by law.
of an organization. Under this distinction, Needless to stress, no obstacle must be
"managerial employees" therefore fall in two placed to the holding of certification
(2) categories, namely, the elections, for it is a statutory policy that
"managers" per se composed of Top and should not be circumvented.
Middle Managers, and the "supervisors"
composed of First-Line Managers. Thus, the
mere fact that an employee is designated
manager" does not ipso facto make him
one. Designation should be reconciled with
the actual job description of the employee,
for it is the job description that determines
the nature of employment.

16
| Labor Relations-2A

DIOLA duly registered with the DOLE, including


any branch or local thereof.
SAN MIGUEL CORPORATION vs.
MANDAUE PACKING PLANTS-SAN Only legitimate labor organizations may file
PACKAGING PRODUCTS SAN MIGUEL a petition for certification election. Article
CORPORATION MONTHLIES RANK- 234 of the Labor Code enumerates the
AND-FILE UNION FFW requirements for registration of an applicant
G.R. No. 152356, August 16, 2005 labor organization, association in order that
such entity could acquire legal personality
FACTS: and entitlement to the rights and privileges
Respondent union filed a petition for granted by law to legitimate labor
certification election with the DOLE organizations.
Regional Office seeking to be certified and
to represent the permanent rank-and-file Although the present law in effect regarding
monthly paid employees of Petitioner. The the procedure of applications for a
required documents for the certification local/chapter is Department Order No. 40, it
were attached to the petition of Respondent. is still Department Order No. 9 which
Respondent submitted the same documents governs the case at bar because it was the
to the Bureau of Labor Relations hoping that law in effect during the time of the filing of
the submission would facilitate the listing of this petition. Under Sec. 1, Rule VI of the
the respondent under the roster of D.O. No. 9, a duly registered federation or
legitimate labor organizations. national union may directly create a
local/chapter by submitting to the Regional
Petitioner San Miguel filed a motion to Office or to the Bureau certain specific
dismiss the petition for two grounds: (1) that documents. Sec. 3 of the same rule
respondent is not included in the roster of provides that the local/chapter acquires
legitimate labor organizations based on the legal personality from the date of the filing of
certification of one of the Regional Directors the complete documentary requirements,
of DOLE and (2) that the Respondent has and not from the issuance of a certification
no legal personality to file the petition for to such effect by the Regional Office or
certification election. Bureau.

ISSUE: WON the Respondent has legal In regular order, it is the federation or
personality? national union, already in possession of
legal personality, which initiates the creation
RULING: YES. of the local/chapter. It issues a charter
The Labor Code defines a “labor certificate indicating the creation or
organization” as – as any union or establishment of the local/chapter. It then
association of employees which exists in submits this charter certificate, along with
whole or in part for the purpose of collective the names of the local/chapters officers,
bargaining or of dealing with employers constitution and by-laws to the Regional
concerning terms and conditions of Office or Bureau. It is the submission of
employment; and a “legitimate labor these documents which vests legal
organization” as – any labor organization personality in the local/chapter, which is

17
| Labor Relations-2A

then free to file on its own a petition for DIZON


certification election.
METROLAB INDUSTRIES, INC. VS. HON.
In this case, the federation in question, the ROLDAN-CONFESOR
FFW, did not submit any of these G.R. No. 108855 February 28, 1996
documentary requirements to the Regional
FACTS:
Office or Bureau. It did however issue a Metro Drug Corporation Employees
charter certificate to the putative Association-Federation of Free Workers
local/chapter along with the other (hereinafter referred to as the Union) is a
documentary requirements to the Regional labor organization representing the rank and
Office, but not for the specific purpose of file employees of petitioner Metrolab
creating the local/chapter, but for filing the Industries, Inc. (hereinafter referred to as
Metrolab/MII) and also of Metro Drug, Inc.
petition for certification election.
After the CBA between the parties expired,
The peculiar circumstances in this case negotiations for new CBA ended into
allow the Court to rule that respondent deadlock. Both parties failed to settle their
acquired the requisite legal personality dispute hence the order issued by the
at the same time it filed the petition for Secretary of Labor and Employment, Ruben
certification election. In doing so, the D. Torres, that any strike or acts that might
exacerbate the situation is ceased and
Court acknowledges that the strict letter of
ordered the parties to execute a new CBA.
the procedural rule was not complied with.
However, labor laws are generally Later, the petitioner moved two lay-off acts
construed liberally in favor of labor, to its rank and file employees and was
especially if doing so affirms the opposed by the union. Petitioner assailed
constitutionally guaranteed right to self- that the move was temporary and exercise
of its management prerogative.
organization. True enough, there was no
On various dates, Metrolab recalled some of
attempt made by the national federation, or the laid off workers on a temporary basis
the local/chapter for that matter, to submit due to availability of work in the production
the enumerated documentary requirements lines.
to the Regional Office or Bureau for the
specific purpose of creating the Acting Labor Secretary Nieves Confesor,
local/chapter. However, these same herein public respondent, declared that the
petitioner’s act illegal and ordered
documents were submitted by the
reinstatement with full backwages. He also
local/chapter to the Regional Office as issued two resolution of cease and desist
attachments to its petition for certification stating that the move exacerbate and
election. Under Section 3, Rule VI of caused conflict to the case at bar. Included
Department Order No. 9, it is the on the last resolution issued by the public
submission of these same documents to the respondent which states that executive
Regional Office or Bureau that operates to secretaries are excluded from the closed-
shop provision of the CBA, not from the
vest legal personality on the local/chapter.
bargaining unit.

A petition for certiorari seeking the


annulment of the Resolution and Omnibus
Resolution of Roldan-Confessor on grounds

18
| Labor Relations-2A

that they were issued with grave abuse of shall immediately return to work and
discretion and excess of jurisdiction. the employer shall immediately
resume operations and readmit all
ISSUES: workers under the same terms and
1. W/N public respondent Labor Secretary conditions prevailing before the
committed grave abuse of discretion strike or lockout. The Secretary of
and exceeded her jurisdiction in Labor and Employment or the
declaring the subject layoffs instituted by Commission may seek the
Metrolab illegal. assistance of law enforcement
agencies to ensure compliance with
2. W/N Executive Secretaries must be this provisionas well as with such
included as part of the bargaining unit of orders as he may issue to enforce
rank and file employees? the same. . . . (Emphasis ours.)
That Metrolab's business is of national
RULING: interest is not disputed. Metrolab is one of
the leading manufacturers and suppliers of
1. No. This Court recognizes the exercise of medical and pharmaceutical products to the
management prerogatives and often country.
declines to interfere with the legitimate
business decisions of the employer. Metrolab's management prerogatives,
However, this privilege is not absolute but therefore, are not being unjustly curtailed
subject to limitations imposed by law. but duly balanced with and tempered by the
limitations set by law, taking into account its
The case at bench constitutes one of the special character and the particular
exceptions. The Secretary of Labor is circumstances in the case at bench.
expressly given the power under the Labor
Code to assume jurisdiction and resolve Metrolab and the Union were still in the
labor disputes involving industries process of resolving their CBA deadlock
indispensable to national interest. The when petitioner implemented the subject
disputed injunction is subsumed under this layoffs. As a result, motions and oppositions
special grant of authority. Art. 263 (g) of the were filed diverting the parties', attention,
Labor Code specifically provides that: delaying resolution of the bargaining
xxx xxx xxx deadlock and postponing the signing of their
(g) When, in his opinion, there exists new CBA, thereby aggravating the whole
a labor dispute causing or likely to conflict.
cause a strike or lockout in an
industry indispensable to the We, likewise, find untenable Metrolab's
national interest, the Secretary of contention that the layoff of the 94 rank-and-
Labor and Employment may assume file employees was temporary, despite the
jurisdiction over the dispute and recall of some of the laid off workers.
decide it or certify the same to the
Commission for compulsory If Metrolab intended the layoff of the 94
arbitration. Such assumption or workers to be temporary, it should have
certification shall have the effect of plainly stated so in the notices it sent to the
automatically enjoining the intended affected employees and the Department of
or impending strike or lockout as Labor and Employment.
specified in the assumption or
certification order. If one has already 2. No. Although Article 245 of the Labor
taken place at the time of Code limits the ineligibility to join, form and
assumption or certification, all assist any labor organization to managerial
striking or locked out employees employees, jurisprudence has extended this

19
| Labor Relations-2A

prohibition to confidential employees or DOÑA


those who by reason of their positions or
nature of work are required to assist or act MANILA ELECTRIC COMPANY v.
in a fiduciary manner to managerial
SECRETARY OF LABOR LEONARDO
employees and hence, are likewise privy to
sensitive and highly confidential records. QUISUMBING and MERALCO
EMPLOYEES AND WORKERS
By recognizing the expanded scope of the ASSOCIATION (MEWA)
right to self-organization, our intent was to G.R. No. 127598 January 27, 1999
delimit the types of employees excluded
from the close shop provision, not from the FACTS:
bargaining unit, to executive secretaries
MERALCO seeks to annul the
only. Otherwise, the conversion of the
exclusionary provision to one that refers to orders of the Secretary of labor wherein the
the bargaining unit from one that merely Secretary required MERALCO and MEWA,
refers to the close shop provision would the duly recognized labor organization of
effectively curtail all the organizational rights the rank-and-file employees of MERALCO,
of executive secretaries. to execute a CBA as t0 re-negotiate the
terms and conditions of their existing 1992-
The executive secretaries of General
1997 Collective Bargaining Agreement
Manager and Management Committees
should not only be exempted from the (CBA) covering the remaining period of two
closed-shop provision but should not be years. That when the negotiating panels of
permitted to join in the bargaining unit of the MERALCO and MEWA failed to arrive at
rank and file employees as well as on the terms and conditions acceptable to both of
grounds that the executive secretaries are them, MEWA filed a Notice of Strike with the
confidential employees, having access to
NCMB-NCR on the grounds of bargaining
“vital labor information”
deadlock and unfair labor practices. The
As stated in several cases, confidential NCMB then conducted a series of
employees are prohibited and disqualified to conciliation meetings but the parties failed
join any bargaining unit since the very to reach an amicable settlement.
nature of the fuction are to assist and act in
a confidential capacity, or to have access to The Labor Secretary granted the
confidential matters of, person who exercise
petition of MERALCO to assume jurisdiction
managerial fuctions in the field of labor
relations. over the labor dispute and to enjoin the
striking employees to go back to work.
Finally, confidential employees cannot be Consequently, the Secretary resolved the
classified as rank and file from the very labor dispute and awarded various
nature of their work. Excluding confidential economic demands, wages and benefits in
employees from the rank and file of favor of MEWA. MERALCO filed a motion
bargaining unit, therefore is not tantamount
for reconsideration alleging that the
to discrimination.
Secretary of Labor did not properly
Therefore, executive secretaries of appreciate the effect of the awarded wages
petitioners’ General Manager and its and benefits on MERALCOs financial
Management Committee are permanently viability. The union disputes the allegation of
excluded from the bargaining unit of MERALCO that the Secretary abused his
petitioner’s rank and file employees.
discretion in issuing the assailed orders.

20
| Labor Relations-2A

interests of the parties to the dispute and


RULING: of those who might be affected by the
The Secretary of Labor’s statutory dispute. The best way in approaching this
power to assume jurisdiction over a labor task holistically is to consider the available
dispute in an industry indispensable to the objective facts, including, where applicable,
national interest, and, to render an award on factors such as the bargaining history of the
compulsory arbitration, does not exempt the company, the trends and amounts of
exercise of this power from the judicial arbitrated and agreed wage awards and the
review. It is the duty of the courts to company’s previous CBAs, and industry
determine whether or not there has been a trends in general. As a rule, affordability or
grave abuse of discretion amounting to lack capacity to pay should be taken into
or excess of jurisdiction on the part of any account but cannot be the sole yardstick in
branch or instrumentality of the government. determining the wage award, especially in a
This Court may review the substance of the public utility like MERALCO. In considering
Secretary’s award when grave abuse of a public utility, the decision maker must
discretion is alleged to exist in the award, always take into account the public interest
i.e., in the appreciation of and the aspects of the case; MERALCOs income
conclusions the Secretary drew from the and the amount of money available for
evidence presented. operating expenses - including labor costs -
are subject to State regulation.
The union posits that no reviewable
abuse of discretion could have attended the SCOPE OF THE BARGAINING UNIT
Secretary’s arbitral award because the The union is demanding that the
Secretary complied with constitutional collective bargaining unit shall be composed
norms in rendering the dispute award of all regular rank and file employees hired
including the obligation of the State to by the company in all its offices and
protect workers, both organized and operating centers through its franchise and
unorganized. Every legal power of the those it may employ by reason of
Secretary of Labor under the Labor Code expansion, reorganization or as a result of
that is attended by grave abuse of discretion operational exigencies. The law is that only
is subject to review by the Court, particularly managerial employees are excluded from
when constitutional norms are cited as the any collective bargaining unit and
applicable yardsticks since this Court is the supervisors are now allowed to form their
final interpreter of the meaning and intent of own union (Art. 254 of the Labor Code as
the Constitution. amended by R.A. 6715). The Court grants
the union demand.
In consideration of the parties’
positions and the evidence on record, the We have established on the
Secretary of Labor disregarded and exclusion of confidential employees from
misappreciated evidence, particularly with the rank and file bargaining unit. In Pier 8
respect to the wage award. The Court Arrastre vs. Confesor and General Maritime
emphasizes that a collective bargaining and Stevedore Union, we ruled that: Put
dispute such as this one requires due another way, the confidential employee
consideration and proper balancing of the does not share in the same community of

21
| Labor Relations-2A

interests that might otherwise make him nature of a co-management control of the
eligible to join his rank and file co-workers, business of MERALCO. What is granted by
precisely because of a conflict in those the Secretary
interest. Thus, in Metrolab Industries vs. is participation and representation. Thus,
Roldan-Confesor, We ruled: ...that the there is no impairment of management
Secretary’s order should exclude the prerogatives.
confidential employees from the regular
rank and file employees qualified to become CHECK OFF OF UNION DUES
members of the MEWA bargaining unit. In any increase of union dues or
Hence, employees holding a confidential contributions for mandatory activities, the
position are prohibited from joining the union must submit to the Company a copy
union of the rank and file employees. of its board resolution increasing the union
dues or authorizing such contributions. If a
UNION REPRESENTATION IN board resolution is submitted, the Company
COMMITTEES shall deduct union dues from all union
Article 211 (A)(g) of the Labor Codes members after a majority of the union
provides: To ensure the participation of members have submitted their individual
workers in decision and policy-making written authorizations. Only those check-off
processes affecting their rights, duties and authorizations submitted by the union shall
welfare. Denying this opportunity to the be honored by the Company. With respect
Union is to lay the claim that only to special assessments, attorney’s fees,
management has the monopoly of ideas negotiation fees or any other extraordinary
that may improve management strategies in fees, individual authorizations shall be
enhancing the Company’s growth. What necessary before the company may so
every company should remember is that deduct the same.
there might be one among the Union
members who may offer productive and
viable ideas on expanding the Company’s
business horizons.

All the Union demands and what the


Secretary’s order granted is that the Union
be allowed to participate in policy
formulation and decision-making process on
matters affecting the Union members right,
duties and welfare as required in Article 211
(A)(g) of the Labor Code. And this can only
be done when the Union is allowed to have
representatives in the Safety Committee,
Uniform Committee and other committees of
a similar nature and purpose involving
personnel welfare, rights and benefits as
well as duties. Such participation by the
Union in the said committees is not in the

22
| Labor Relations-2A

DUQUILLA capacity, (2) to persons who formulate,


determine, and effectuate management
policies in the field of labor relations. The
SAN MIGUEL CORP. UNION vs.
exclusion from bargaining units of
LAGUESMA
employees who, in the normal course of
G.R. No. 110399 August 15, 1997
their duties, become aware of management
policies relating to labor relations is a
FACTS:
principal objective sought to be
Petitioner union filed before the DOLE a
accomplished by the ''confidential employee
petition for Direct Certification or
rule." The broad rationale behind this rule is
Certification Election among the supervisors
that employees should not be placed in a
and exempt employees of the SMC
position involving a potential conflict of
Magnolia Poultry Products Plants of
interests.
Cabuyao, San Fernando and Otis. An Order
issued by Undersecretary Laguesma,
Here, supervisors 3 and above may
excluded the employees under supervisory
not be considered confidential employees
levels 3 and 4 and the so-called exempt
merely because they handle "confidential
employees from the proposed bargaining
data". The information they handle are
unit and ruled out their participation in the
properly classifiable as technical and
certification election because they were
internal business operations data, relating to
admittedly confidential employees who, like
product formulation, product standards and
managerial employees, are not allowed to
product specification, which by no means
form, join or assist a labor union for
relate to labor relations. Since the
purposes of collective bargaining. Moreover,
employees are not classifiable under the
Laguesma directed the conduct of separate
confidential type, they may appropriately
certification elections in each of the three
form a bargaining unit for purposes of
plants.
collective bargaining.

ISSUES:
(2) Yes, the employees of the plants of
(1) Whether or not Supervisory
Cabuyao, San Fernando and Otis shall be
employees 3 and 4 and the exempt
recognized as one bargaining unit.
employees of the company are
considered confidential employees,
A unit to be appropriate must effect
hence ineligible from joining a union.
a grouping of employees who have
(2) Whether or not the employees of the
substantial, mutual interests in wages,
three plants constitute an appropriate
hours, working conditions and other
single bargaining unit.
subjects of collective bargaining. It is readily
seen that the employees in the instant case
HELD:
have "community or mutuality of interests,"
(1) No, said employees do not fall within the
which is the standard in determining the
term “confidential employees” who may
proper constituency of a collective
be prohibited from joining a union.
bargaining unit. It is undisputed that they all
belong to the Magnolia Poultry Division of
Confidential employees are those
San Miguel Corporation. This means that,
who (1) assist or act in a confidential

23
| Labor Relations-2A

although they belong to three different EISMA


plants, they perform work of the same
nature, receive the same wages and SUGBUANON RURAL BANK, INC. vs.
compensation, and most importantly, share Usec. BIENVENIDO LAGUESMA
a common stake in concerted activities. G.R. No. 116194 February 2, 2000

When the employee does not have access


to confidential labor relations information,
there is no legal prohibition against
confidential employees from forming,
assisting, or joining a union.

FACTS:
Petitioner is a duly-registered banking
institution in Cebu, while private respondent
APSOTEU-TUCP was a labor organization
duly-registered with the Labor Department.
APSOTEU filed a petition for certification
election of the 5 supervisory employees.
SRBI filed a motion to dismiss the union's
petition on two grounds. First, that the
members of union were in fact managerial
or confidential employees and were to be
disqualified from forming or joining unions.
Second, the Association of Labor Unions-
TUCP was representing the union. Since
ALU-TUCP also sought to represent the
rank-and-file employees of SRBI, there was
a violation of the principle of separation of
unions. The union argued that its members
were not managerial employees but merely
supervisory employees.

Med-Arbiter: denied petitioner's motion to


dismiss.
Secretary of Labor and Employment: same
was denied.
DOLE Undersecretary: denied SRBI's
appeal.

ISSUE: WON the members of the


respondent union are
managerial/confidential employees, hence
prohibited by law from joining labor
organizations and engaging in union
activities.

24
| Labor Relations-2A

FABON
HELD: No.

The petitioner bank failed to show that their DLSU vs. DLSU EMPLOYEES
Cashiers, Accountant, and Acting Chief of ASSOCIATION (DLSUEA)
the Loans Department possessed G.R. No. 109002 April 12, 2000
managerial powers and duties. At best, they
only had recommendatory powers subject to FACTS:
evaluation, review, and final decision by the
bank's management. The job description DLSU (the University) and DLSU
forms submitted by petitioner clearly show Employees Association—National
that the union members in question may not Federation of Teachers and Employees
transfer, suspend, lay-off, recall, discharge, Union (UNION) entered into a CBA. Before
assign, or discipline employees. Moreover, the expiration of the CBA, the Union
the forms also do not indicate that said initiated negotiations with the University for
officers could formulate and execute
a new CBA. After several conciliation-
management policies which are normally
expected of management officers. Neither mediation meetings, a partial CBA was
could the union members be treated as thereafter executed by the parties.
confidential employees. Although the
cashier serves the bank's management, it There arose an issue regarding the
could not be deemed to have access to propriety of the inclusion of a union shop
confidential information specifically relating clause in the CBA, in addition to the existing
to SRBI's labor relations policies.
maintenance of membership clause.
As regards the issue on the violation of the
principle of separation of unions, records The University avers that ". . . it is in the
show that respondent union was initially spirit of the exercise of the constitutional
assisted by ALU during its preliminary right to self-organization that every
stages. A local union maintains its separate individual should be able to freely choose
personality despite affiliation with a larger
whether to become a member of the Union
national federation. APSOTEU-TUCP had
separate legal personality from ALU and or not. The right to join a labor organization
TUCP. should carry with it the corollary right not to
join the same. This position of the University
The law frowns on a union where the is but in due recognition of the individual's
membership is composed of both free will and capability for judgment." The
supervisors and rank-and-file employees, University assails the Union's demand for a
for fear that conflicts of interest may arise in
union shop clause as ". . . definitely unjust
the areas of discipline, collective bargaining,
and strikes. However, in the present case, and amounts to oppression. Moreover, such
none of the members of the respondent a demand is repugnant to democratic
union came from the rank-and-file principles and the constitutionally
employees of the bank. guaranteed freedom of individuals to join or
not to join an association as well as their
right to security of tenure, particularly, on
the part of present employees."

The Union, on the other hand, counters that


the Labor Code, as amended, recognizes

25
| Labor Relations-2A

the validity of a union shop agreement in FABON


Article 248 thereof which reads:
Art. 248. Unfair labor practices of PROGRESSIVE DEVELOPMENT
employers. — CORPORATION-PIZZA HUT vs. HON.
xxx xxx xxx BIENVENIDO LAGUESMA
(e) To discriminate in regard to hire or G.R. No. 115077 April 18, 1997
tenure of employment or any term or
condition of employment in order to FACTS:
encourage or discourage membership in Nagkakaisang Lakas ng Manggagawa
(NLM)-Katipunan (respondent Union) filed a
any labor organization. Nothing in this
petition for certification election with the
Code or in any other law shall prevent Department of Labor in behalf of the rank
the parties from requiring membership in and file employees of the Progressive
a recognized collective bargaining agent Development Corporation (Pizza Hut).
as a condition for employment, except of
those employees who are already Petitioner filed a Motion to Dismiss the
members of another union at the time of petition alleging fraud, falsification and
misrepresentation in the respondent Union's
the signing of the CBA…”
registration. Petitioner alleged, among
others, that respondent Union's registration
ISSUE: was tainted with false, forged, double or
W/N a union shop clause should be multiple signatures.
included in the parties' CBA, in addition to
the existing maintenance of membership Petitioner filed a Petition seeking the
clause in the CBA. (YES.) cancellation of the Union's registration on
the grounds of fraud and falsification.
Motion was filed by petitioner with the Med-
HELD: Arbiter requesting suspension of
The SC ruled that there should be an proceedings in the certification election case
inclusion of a union shop provision in until after the prejudicial question of the
addition to the existing maintenance of Union's legal personality is determined in
membership clause in the CBA. A union the proceedings for cancellation of
registration.
shop clause “is not a restriction on the
employee's right of (sic) freedom of However Med-Arbiter Abdullah directed
association but rather a valid form of union the holding of a certification election among
security while the CBA is in force and in petitioner's rank and file employees. The
accordance with the Constitutional policy to Order explained:
promote unionism and collective bargaining
and negotiations. The parties therefore x x x Sumasaklaw sa
Manggagawa ng Pizza Hut is a
should incorporate such union shop clause
legitimate labor organization in
in their CBA." contemplation of law and shall
remain as such until its very
charter certificate is canceled or
otherwise revoked by competent
authority. The alleged
misrepresentation, fraud and false
statement in connection with the
issuance of the charter certificate

26
| Labor Relations-2A

are collateral issues which could application for registration is vitiated by


be properly ventilated in the falsification and serious irregularities,
cancellation proceedings. especially those appearing on the face of
the application and the supporting
ISSUE: W/N after the necessary papers and documents, a labor organization should be
documents have been filed by a labor denied recognition as a legitimate labor
organization, recognition by the Bureau of organization. And if a certificate of
Labor Relations merely becomes a recognition has been issued, the propriety
ministerial function. (NO.) of the labor organization's registration could
be assailed directly through cancellation of
HELD: registration proceedings in accordance with
Articles 238 and 239 of the Labor Code, or
Art. 234. Requirements of registration. - Any indirectly, by challenging its petition for the
applicant labor organization, association or issuance of an order for certification
group of unions or workers shall acquire election.
legal personality and shall be entitled to the
rights and privileges granted by law to Registration requirements specifically
legitimate labor organizations upon afford a measure of protection to
issuance of the certificate of registration unsuspecting employees who may be lured
based on the following requirements: into joining unscrupulous or fly-by-night
unions whose sole purpose is to control
(a) P50.00 registration fee; (b) The union funds or use the labor organization for
names of its officers, their illegitimate ends. Such requirements are a
addresses, the principal address valid exercise of the police power, because
of the labor organization, the the activities in which labor organizations,
minutes of the organizational associations and unions of workers are
meetings and the list of the engaged directly affect the public interest
workers who participated in and should be protected.
such meetings; (c) The names
of all its members comprising at The employer needs the assurance that
least twenty percent (20%) of all the union it is dealing with is a bona fide
the employees in the bargaining organization, one which has not submitted
unit where it seeks to operate; false statements or misrepresentations to
(d) If the applicant union has the Bureau. Clearly, fraud, falsification and
been in existence for one or misrepresentation in obtaining recognition
more years, copies of its annual as a legitimate labor organization are
financial reports; and (e) 4 contrary to the Med-Arbiter’s conclusion not
copies of the constitution and merely collateral issues. The invalidity of
by-laws of the applicant union, respondent Union’s registration would
minutes of its adoption or negate its legal personality to participate in
ratification, and the list of the certification election.
members who participated in it.
Once a labor organization attains the
The requirements are intended as status of a legitimate labor organization it
preventive measures against the begins to possess all of the rights and
commission of fraud. After a labor privileges granted by law to such
organization has filed the papers and organizations.
documents for registration, it becomes
mandatory for the Bureau of Labor Inasmuch as the legal personality of
Relations to check if the requirements under respondent Union had been seriously
Article 234 have been complied with. If its challenged, it would have been more

27
| Labor Relations-2A

prudent for the Med-Arbiter and public FAJARDO


respondent to have granted petitioner's
request for the suspension of proceedings JOHNSON AND JOHNSON LABOR
in the certification election case, until the UNION-FFW, ET. AL. vs
issue of the legality of the Union's DIRECTOR OF LABOR RELATIONS AND
registration shall have been OSCAR PILI
resolved. Failure of the Med-Arbiter and
public respondent to heed the request G.R. No. 76427 February 21, 1989
constituted a grave abuse of discretion.
FACTS:
Private respondent Oscar Pili was
dismissed from his employment for violation
of company policy, which is for non-
disclosure in his job application form of the
fact that he had a relative in the company.
Upon his dismissal, Pili opt to receive the
financial aid under the By-Laws of petitioner
union. Financial aid is afforded to union
members who have been suspended or
terminated from work without reasonable
cause.

Petitioner union refused to give Pili


the financial aid, it alleged that the union’s
executive board has the prerogative to
determine whether or not the dismissal was
for a reasonable cause or not. They ruled,
together with the general membership, not
to extend financial aid to Pili.

Meanwhile, Pili filed a case for illegal


dismissal against his employer.

Pili also filed a complaint against


petitioner union with the Med-Arbiter which
ordered the union to pay Pili the
corresponding financial aid in case the
complaint for illegal dismissal was ruled in
Pili’s favour provided that if it is decided
otherwise, the union will be absolved in its
obligations.

The union assailed the said order


positing that the Med-Arbiter, in granting the

28
| Labor Relations-2A

financial aid, has substituted its decision on FERRER


whether or not the dismissal is reasonable
PHILIPPINE SKYLANDERS, INC.,
MARILES C. ROMULO and FRANCISCO
ISSUE: DAKILA vs. NATIONAL LABOR
Whether or not the public RELATIONS COMMISSION, LABOR
respondent committed grave abuse of ARBITER EMERSON TUMANON,
discretion in substituting the decision of PHILIPPINE ASSOCIATION OF FREE
petitioner-union. LABOR UNIONS (PAFLU) SEPTEMBER
(now UNIFIED PAFLU) and SERAFIN
AYROSO
HELD: [G.R. No. 127374. January 31, 2002]
NO.
Facts:
A union’s constitution is covenant In November 1993 the Philippine
between the union and its members. There Skylanders Employees Association (PSEA),
is nothing in the constitution which leaves a local labor union affiliated with the
Philippine Association of Free Labor Unions
the legal interpretation of its terms
(PAFLU) September (PAFLU), won in the
unilaterally to the union or its officers or certification election conducted among the
even the general membership. Hence, the rank and file employees of Philippine
union’s own interpretation that Skylanders, Inc. (PSI). Its rival union,
reasonableness of dismissal is the Philippine Skylanders Employees
prerogative of union’s executive board is Association-WATU (PSEA-WATU)
immediately protested the result of the
untenable.
election before the Secretary of Labor.
Several months later, pending settlement of
The union’s constitution or by-laws the controversy, PSEA sent PAFLU a notice
will clearly show that a member who is of disaffiliation citing as reason PAFLU's
suspended or terminated from employment supposed deliberate and habitual dereliction
without reasonable cause is entitled to of duty toward its members. Attached to the
financial assistance. However, the same by- notice was a copy of the resolution adopted
and signed by the officers and members of
laws do not indicate which body has the
PSEA authorizing their local union to
power to determine whether a suspension disaffiliate from its mother federation.
or dismissal is for reasonable cause or not.
There can be no doubt that the by-laws PSEA subsequently affiliated itself with the
silence on this matter is a clear recognition National Congress of Workers (NCW),
of the labor arbiter’s exclusive jurisdiction changed its name to Philippine Skylanders
over dismissal cases. Employees Association - National Congress
of Workers (PSEA-NCW), and to maintain
continuity within the organization, allowed
To wit, whether or not one is entitle the former officers of PSEA-PAFLU to
to the financial aid, depends on the result of continue occupying their positions as
the dismissal case filed before the proper elected officers in the newly-forged PSEA-
government agency. NCW. On 17 March 1994 PSEA-NCW
entered into a collective bargaining
agreement with PSI which was immediately
registered with the Department of Labor and
Employment.

29
| Labor Relations-2A

Meanwhile, apparently oblivious to PSEA's file the instant complaint. In support of this
shift of allegiance, PAFLU Secretary assertion, PSEA-NCW submitted in
General Serafin Ayroso wrote Mariles C. evidence a Katunayan signed by 111 out of
Romulo requesting a copy of PSI's audited 120 rank and file employees of PSI
financial statement. Ayroso explained that disauthorizing Ayroso or PAFLU from
with the dismissal of PSEA-WATUs election instituting any action in their behalf.
protest the time was ripe for the parties to
enter into a collective bargaining In a Decision rendered on 30 June 1995 the
agreement. On 30 July 1994 PSI through its Labor Arbiter declared PSEA's disaffiliation
personnel manager Francisco Dakila denied from PAFLU invalid and held PSI, PSEA-
the request citing as reason PSEA's PAFLU and their respective officers guilty of
disaffiliation from PAFLU and its unfair labor practice. The Decision
subsequent affiliation with NCW. explained that despite PSEA-PAFLU's
status as the sole and exclusive bargaining
Agitated by PSI's recognition of PSEA- agent of PSI's rank and file employees, the
NCW, PAFLU through Serafin Ayroso filed a company knowingly sanctioned and
complaint for unfair labor practice against confederated with Dakila in actively
PSI, its president Mariles Romulo and assisting a rival union. This, according to
personnel manager Francisco Dakila. the Labor Arbiter, was a classic case of
PAFLU alleged that aside from PSIs refusal interference for which PSI could be held
to bargain collectively with its workers, the responsible. As PSEA-NCW's personality
company through its president and was not accorded recognition, its collective
personnel manager, was also liable for bargaining agreement with PSI was struck
interfering with its employees' union down for being invalid. Ayroso's legal
activities. Two days later or on 6 October personality to file the complaint was
1994 Ayroso filed another complaint in sustained on the ratiocination that under the
behalf of PAFLU for unfair labor practice Labor Code no petition questioning the
against Francisco Dakila. Through Ayroso majority status of the incumbent bargaining
PAFLU claimed that Dakila was present in agent shall be entertained outside of the
PSEA's organizational meeting thereby sixty (60)-day period immediately before the
confirming his illicit participation in union expiry date of such five (5)-year term of the
activities. Ayroso added that the members collective bargaining agreement that the
of the local union had unwittingly fallen into parties may enter into. Accordingly,
the manipulative machinations of PSI and judgment was rendered ordering PSI,
were lured into endorsing a collective PSEA-PAFLU and their officers to pay
bargaining agreement which was PAFLU P150,000.00 in damages.
detrimental to their interests. The two (2)
complaints were thereafter consolidated. PSI, PSEA and their respective officers
appealed to the National Labor Relations
PSI, its president Mariles C. Romulo, and its Commission (NLRC). But the NLRC upheld
personnel manager Dakila moved for the the Decision of the Labor Arbiter and
dismissal of the complaint on the ground conjectured that since an election protest
that the issue of disaffiliation was an inter- questioning PSEA-PAFLU's certification as
union conflict which lay beyond the the sole and exclusive bargaining agent was
jurisdiction of the Labor Arbiter. On the pending resolution before the Secretary of
other hand, PSEA-NCW took the cudgels Labor, PSEA could not validly separate from
for its officers who were being sued in their PAFLU, join another national federation and
capacities as former officers of PSEA- subsequently enter into a collective
PAFLU and asserted that since PSEA was bargaining agreement with its employer-
no longer affiliated with PAFLU, Ayroso or company.
PAFLU for that matter had no personality to

30
| Labor Relations-2A

In the current petitions, petitioner PSEA Held:


together with its officers argued that by Yes, a local union can validly disaffiliate
virtue of their disaffiliation PAFLU as a mere from its mother union as a matter of right,
agent had no authority to represent them even pending an election protest of its
before any proceedings. They further status. The finding of Unfair Labor Practice
asserted that being an independent labor by NLRC and Labor Arbiter against PSI was
union PSEA may freely serve the interest of reversed and set aside.
all its members and readily disaffiliate from
its mother federation when circumstances At the outset, let it be noted that the issue of
so warrant. This right, they averred, was disaffiliation is an inter-union conflict the
consistent with the constitutional guarantee jurisdiction of which properly lies with the
of freedom of association. Bureau of Labor Relations (BLR) and not
with the Labor Arbiter. Nonetheless, with
For their part, petitioners PSI, Romulo and due recognition of this fact, SC deemed it
Dakila alleged that their decision to bargain proper to settle the controversy at this
collectively with PSEA-NCW was actuated, instance since to remand the case to the
to a large extent, by PAFLU's behavior. BLR would only mean intolerable delay for
Having heard no objections or protestations the parties.
from PAFLU relative to PSEA's disaffiliation,
they reckoned that PSEA's subsequent The right of a local union to disaffiliate from
association with NSW was done bona fide. its mother federation is not a novel thesis
unillumined by case law. In the landmark
The Solicitor General filed a Manifestation in case of Liberty Cotton Mills Workers Union
Lieu of Comment recommending that both vs. Liberty Cotton Mills, Inc. SC upheld the
petitions be granted. In his Manifestation, right of local unions to separate from their
the Solicitor General argued against the mother federation on the ground that as
Labor Arbiter's assumption of jurisdiction separate and voluntary associations, local
citing the following as reasons: first, there unions do not owe their creation and
was no employer-employee relationship existence to the national federation to which
between complainant Ayroso and PSI over they are affiliated but, instead, to the will of
which the Labor Arbiter could rightfully their members. The sole essence of
assert his jurisdiction; second, since the affiliation is to increase, by collective action,
case involved a dispute between PAFLU as the common bargaining power of local
mother federation and PSEA as local union, unions for the effective enhancement and
the controversy fell within the jurisdiction of protection of their interests. Admittedly,
the Bureau of Labor Relations; and lastly, there are times when without succor and
the relationship of principal-agent between support local unions may find it hard,
PAFLU and PSEA had been severed by the unaided by other support groups, to secure
local union through the lawful exercise of its justice for themselves.
right of disaffiliation.
Yet the local unions remain the basic units
Issue: of association, free to serve their own
Whether or not PSEA, which is an interests subject to the restraints imposed
independent and separate local union, by the constitution and by-laws of the
validly disaffiliate from PAFLU pending the national federation, and free also to
settlement of an election protest questioning renounce the affiliation upon the terms laid
its status as the sole and exclusive down in the agreement which brought such
bargaining agent of PSI's rank and file affiliation into existence.
employees Upon an application of the aforecited
principle to the issue at hand, the
impropriety of the questioned Decisions

31
| Labor Relations-2A

becomes clearly apparent. There is nothing dismissed at the first instance for failure to
shown in the records nor is it claimed by state a cause of action.
PAFLU that the local union was expressly
forbidden to disaffiliate from the federation Policy considerations dictate that in
nor were there any conditions imposed for a weighing the claims of a local union as
valid breakaway. As such, the pendency of against those of a national federation, those
an election protest involving both the mother of the former must be preferred.
federation and the local union did not Parenthetically though, the desires of the
constitute a bar to a valid disaffiliation. mother federation to protect its locals are
Neither was it disputed by PAFLU that 111 not altogether to be shunned. It will however
signatories out of the 120 members of the be to err greatly against the Constitution if
local union, or an equivalent of 92.5% of the the desires of the federation would be
total union membership supported the claim favored over those of its members. That, at
of disaffiliation and had in fact disauthorized any rate, is the policy of the law. For if it
PAFLU from instituting any complaint in were otherwise, instead of protection, there
their behalf. Surely, this is not a case where would be disregard and neglect of the lowly
one or two members of the local union workingmen.
decided to disaffiliate from the mother
federation, but it is a case where almost all
local union members decided to disaffiliate.

It was entirely reasonable then for PSI to


enter into a collective bargaining agreement
with PSEA-NCW. As PSEA had validly
severed itself from PAFLU, there would be
no restrictions which could validly hinder it
from subsequently affiliating with NCW and
entering into a collective bargaining
agreement in behalf of its members.
There is a further consideration that likewise
argues for the granting of the petitions. It
stands unchallenged that PAFLU instituted
the complaint for unfair labor practice
against the wishes of workers whose
interests it was supposedly protecting. The
mere act of disaffiliation did not divest PSEA
of its own personality; neither did it give
PAFLU the license to act independently of
the local union. Recreant to its mission,
PAFLU cannot simply ignore the demands
of the local chapter and decide for its
welfare. PAFLU might have forgotten that
as an agent it could only act in
representation of and in accordance with
the interests of the local union. The
complaint then for unfair labor practice
lodged by PAFLU against PSI, PSEA and
their respective officers, having been filed
by a party which has no legal personality to
institute the complaint, should have been

32
| Labor Relations-2A

ISSUE: W/N the local union’s declaration of


local autonomy is an act of disloyalty which
HADLOC warrants their disassociation from the union
resulting in the employees’ valid termination
MALAYANG SAMAHAN NG MGA from employment.
MANGGAGAWA SA GREENFIELD v.
RAMOS RULING: No.
G.R. No. 113907 February 28, 2000
A local union, being a separate and
voluntary association, is free to serve the
FACTS: interests of all its members including the
Petitioner MSMG, a local union, is freedom to disaffiliate or declare its
an affiliate of private respondent United autonomy from the federation to which it
Lumber and General Workers of the belongs when circumstances warrant, in
Philippines (the federation). In the Union accordance with the constitutional
Security Clause included in the CBA guarantee of freedom of association. The
between the local union and the federation purpose of affiliation by a local union with a
on one hand, and respondent company M. mother union or a federation is to increase
Greenfield Company Inc. on the other hand, by collective action the bargaining power in
it is provided that: respect of the terms and conditions of labor.
Thus, a local union which has affiliated itself
xxx xxx xxx with a federation is free to sever such
Any…employee …who fails to maintain his affiliation anytime and such disaffiliation
membership in the UNION for non-payment cannot be considered disloyalty. In the
of UNION dues, for resignation and for absence of specific provisions in the
violation of UNION's Constitution and By- federation's constitution prohibiting
Laws and any new employee …shall upon disaffiliation or the declaration of autonomy
written notice of such failure to join or to of a local union, a local may dissociate with
maintain membership in the UNION and its parent union. ULGWP's constitution then
upon written recommendation to the does not have such provision.
COMPANY by the UNION, be dismissed
from the employment by the COMPANY…
xxx xxx xxx

The merits of this case commenced


when 30 employees (petitioners) were
terminated by respondent company without
a prior administrative investigation following
the demand of the federation that they be
terminated. The federation alleged that the
petitioners committed acts of disloyalty
when it "undertook to disaffiliate from the
federation and declared autonomy; wherein
they prohibited the federation from
interfering in any internal and external
affairs of the local union’; and hence,
pursuant to the Union Security Clause, the
Company ought to terminate their
employment.

33
| Labor Relations-2A

LEANO formation of which was a protected activity


and could not be considered as disloyalty,
ASSOCIATED WORKERS UNION- held the suspension or dismissal of
PTGWO vs. NLRC individual respondents as illegal and
G.R. No. 87266-69 July 30, 1990 ordered their reinstatement with backwages,
to be paid solidarily by AWU and
MANILA PORT SERVICES, INC., vs. Hon. respondent Metro. The NLRC also found
Arthur Amansec et.al taht the strike staged by AWU is not illegal,
G.R. Nos. 91223-26 JULY 30, 1990 holding that AWU was of the belief,
although erroneously, that it could validly
FACTS: stage a strike during the pendency of its
motion for reconsideration of the Minister's
Petitioner, the then bargaining Order enjoining a strike or lockout.
representative of the dockworkers at South
Harbor, Port Area, Manila, filed a Notice of Both AWU and Metro filed separate motions
Strike against respondent Metro Port for reconsideration of the consolidated
Service, Inc., the then arrastre contractor in Decision. Meanwhile, petitioner Marina Port
the South Harbor, on the issues, among Services, Inc., started operations as the
others, of unfilled vacancies and union arrastre operator at the Manila South
busting (NLRC Case No. NCR-NS-10-288- Harbor vice Metro. The NLRC in a
84). Minsitry of Labor and Employement Resolution denied AWU's and Metro's
ordered for the compulsary arbitration to the motions for reconsideration and
NLRC and forbade the holding of strikes or ordered Marina to reinstate individual
lock-outs ( Certified NLRC Case No. 0403- respondents with backwages and
85). In the latter case, one of the demands allowances Marina complied with the
raised by AWU was that Metro terminate the Resolution by reinstating individual
employment of 11 repondents for having respondents
organized the Associated Workers Union in
Metroport ("AWUM"). Metro initially resisted AWU thereafter in G.R. Nos. 87266-69 filed
AWU’s request by reason of due proces. with the Court a Petition for certiorari
However, Metro eventually suspended praying for the reversal of the order holding
individual respondents after AWU staged a that Metro could not be compelled to fill up
strike against it, despite the express vacancies with AWU’s recommendees and
prohibition in the Order. To end the strike, of the order reinstating the 11 private
Metro executed an Agreement with AWU. respondents to their former positions with
As a result of Metro’s implementation of the backwages payable solidarily by AWU and
Agreement, individual respondents filed a respondent Metro.
complaint against Metro (NLRC Case No.
NCR-4-1372-85) . Labor Arbiter Diosana in Marina, meantime, had gone to the Court,
an Order directed Metro provisionally to protesting, on grounds of alleged denial of
reinstate individual respondents pending due process, its inclusion by the NLRC as a
resolution of the issues raised therein, with party and its being required to reinstate
which Order Metro complied. individual respondents with backwages.
Metro also went to the court again and
All the above-mentioned cases were claimed that it should not have been held
ordered consolidated before the NLRC en solidary liable with AWU because it had
banc. NLRC rendered a consolidated merely suspended individual respondents
Decision. The NLRC ruled that that AWU pursuant to the Agreement it had executed
was a national union, and that individual with AWU. These cases were dismissed by
respondents have the right to organize the Court and were remanded to the LA of
themselves into a local chapter thereof, the origin for execution. The LA issued a writ of

34
| Labor Relations-2A

execution against Marina to reinstate AWUM. Respondent Metro had about 4,000
individual respondents and to pay them the employees, and around 2,000 of these were
salary adjustments. Marina then once more members of AWU. It is evident that
went to the Court in G.R. Nos. 91223-26 individual respondents had failed to muster
and filed a Petition for certiorari to invalidate the necessary majority in order to justify
the writ of execution. G.R. Nos. 91223-26 their disaffiliation. Thus, in the referendum
were consolidated with G.R. Nos. 87266- held at the PTGWO compound to determine
69. whether individual respondents should be
expelled from AWU, 1,229 members out of
ISSUES: 1,695 members present voted for expulsion
1. WON AWU was justified in expelling its of individual respondents. In sum, the
membership the 11 individual respondents. attempted disaffiliation of the 11 private
2. WON Metro and Marina should be held respondents from the petitioner mother
solidary liable with AWU for the payment of union and the effort to organize either a new
backwages. local of the mother union or an entirely new
and separate union, did not, under the
HELD: circumstances of this case, constitute
protected activities of the 11 individual
1. YES. AWU was justified in expelling respondents.
from its membership the 11 individual
respondents. 2. Strictly speaking, in view of the
conclusion above that AWU was justified in
AWUM as a local union, being an entity expelling individual respondents from its
separate and distinct from AWU, is free to membership, neither AWU nor Metro/Marina
serve the interest of all its members and would be liable to individual respondents for
enjoys the freedom to disaffiliate, such right the backwages accruing during this Second
to disaffiliate may be exercised, and is thus Period.
considered a protected labor activity, only
when warranted by circumstances. In the interest of substantial and expeditious
Generally, a labor union may disaffiliate justice, however, we believe that the
from the mother union to form a local or backwages accruing during the Second
independent union only during the 60-day Period should be paid and shared by AWU
freedom period immediately preceding the and by Metro Marina, on a 50-50 basis. The
expiration of the CBA. Even before the equitable considerations which impel the
onset of the freedom period, disaffiliation Court to hold AWU liable for one-half of the
may still be carried out, but must be backwages during the Second Period
effected by a majority of the members in the include:
bargaining unit. This happens when there is
a substantial shift in allegiance on the part (a) the fact that Metro had been reluctant to
of the majority of the members of the union. comply with the demand of AWU to
In such a case, however, the CBA continues terminate the services of individual
to bind the members of the new or respondents and had wanted to give the
disaffiliated and independent union up to the latter procedural due process, but gave in to
CBA's expiration date. the demands of AWU;
(b) that AWU had pressed Metro very hard
The record does not show that individual and indeed went on strike against Metro
respondents had disaffiliated during the when Metro refused simply to terminate the
freedom period. The record does, however, services of the individual respondents;
show that only 11 members of AWU had (c) that AWU, instead of waiting for final
decided to disaffiliate from AWU and form judicial determination of the legality of its
expulsion of individual respondents, chose

35
| Labor Relations-2A

to importune the NLRC to issue the order LEONARDO


requiring the re-suspension of the individual
respondents on 1 August 1985, with which TOYOTA MOTOR PHIL. CORP vs.
order Metro eventually complied. TOYOTA MOTOR PHIL. CORP LABOR
UNION
The equitable considerations which lead to G.R. No. 121084. February 19, 1997
hold Metro/Marina responsible for one-half
of the backwages accruing during the above FACTS:
Second Period relate to the failure of Metro On Nov. 26, 1992, Toyota Motor Phil
to accord individual respondents procedural Corp. Labor Union (TMPCLU) filed a
due process by giving them reasonable petition for certification election with Dept. of
opportunity to explain their side before Labor, for all rank-and-file employees of the
suspending or dismissing them. Such petitioner Toyota Motor Corp. Petitioner
dismissal was accordingly in violation of the seek the denial of the holding of the
Labor Code. Notwithstanding AWU's closed- certification election on 2 grounds: First, the
shop clause in the CBA, Metro was bound union, being “process of registration” had no
to conduct its own inquiry to determine the legal personality to file the same as it was
existence of substantial basis for not a legitimate labor organization at the
terminating the employment of individual time the petition was file; and Second, that
respondents. That AWU, disregarding the the union was composed of both rank-and-
Minister of Labor and Employment's file and supervisory employees in violation
express order, had threatened to go on of the law. The Med-Arbiter dismissed the
strike, and indeed actually went on strike, if petition for certification election in favor with
Metro had continued with the services of the grounds stated by petitioner. However,
individual respondents, did not relieve Metro on appeal, the Secretary of Labor set aside
from the duty to accord procedural due the decision of the Me-Arbiter and ordered
process to individual respondents. the holding of the certification election
contending that the union was already a
legitimate labor organization at the time of
the filing of the petition evidenced by a
certificate of registration.

ISSUE:
Whether or not TMPCLU is a legitimate
labor organization that can file for petition or
certification election.

HELD:
No. TMPCLU is not a legitimate labor
organization.

The Labor Code (Art. 245) has made it


a clear statutory policy to prevent
supervisory employees from joining labor
organizations consisting of rank-and-file
employees as the concerns which involve
members of either group are normally
disparate and contradictory. A labor
organization composed of both rank-and-file
and supervisory employees is no labor
organization at all. It cannot, for any guise

36
| Labor Relations-2A

or purpose, be a legitimate labor determine manpower requirements, thereby


organization. Not being one, an organization influencing important hiring decisions at the
which carries a mixture of rank-and-file and highest levels. This determination is neither
supervisory employees cannot possess any routine nor clerical but involves the
of the rights of a legitimate labor independent assessment of factors affecting
organization, including the right to file a production, which in turn affect decisions to
petition for certification election for the hire or transfer workers. The use of
purpose of collective bargaining. It becomes independent judgment in making the
necessary, therefore, anterior to the decision to hire, fire or transfer in the
granting of an order allowing a certification identification of manpower requirements
election, to inquire into the composition of would be greatly impaired if the employee's
any labor organization whenever the status loyalties are torn between the interests of
of the labor organization is challenged on the union and the interests of management.
the basis of Article 245 of the Labor Code. A supervisory employee occupying a level
five position would therefore find it difficult to
In the case at bar, as respondent objectively identify the exact manpower
union's membership list contains the names requirements dictated by production
of at least twenty-seven (27) supervisory demands.
employees in Level Five positions. The job-
description for the Level 5 positions is This is precisely what the Labor Code,
quoted below: in requiring separate unions among rank-
and-file employees on one hand, and
LEVEL 5 He is responsible for supervisory employees on the other, seeks
overseeing initial production of new models, to avoid. The rationale behind the Code's
prepares and monitors construction exclusion of supervisors from unions of
schedules for new models, identifies rank-and-file employees is that such
manpower requirements for production, employees, while in the performance of
facilities and equipment, and lay-out supervisory functions, become the alter ego
processes. He also oversees other sections of management in the making and the
in the production process (e.g. assembly, implementing of key decisions at the sub-
welding, painting)." (Annex "V" of managerial level. Certainly, it would be
Respondent TMP's Position Paper, which is difficult to find unity or mutuality of interests
the Job Description for an Engineer holding in a bargaining unit consisting of a mixture
Level 5 position in the Production of rank-and-file and supervisory employees.
Engineering Section of the Production And this is so because the fundamental test
Planning and Control Department). of a bargaining unit's acceptability is
whether or not such a unit will best advance
From a reading of the Labor Code’s to all employees within the unit the proper
definition of supervisory employees, those exercise of their collective bargaining
occupying Level 5 positions are rights. The Code itself has recognized this,
unquestionably supervisory employees. in preventing supervisory employees from
Supervisory employees are those who, in joining unions of rank-and-file employees.
the interest of the employer, effectively
recommend managerial actions if the Hence, the union could not attain the
exercise of such authority is not merely status of a legitimate labor organization. Not
routinary or clerical in nature but require the being one, it cannot possess the requisite
use of independent judgment. Under the job personality to file a petition for certification
description for level five employees, such election.
personnel all engineers having a number of
personnel under them, not only oversee
production of new models but also

37
| Labor Relations-2A

MAGABILIN also by 'non-USTFU members who are


members in good standing of the UST
UST FACULTY UNION (USTFU) et al vs. Academic Community Collective Bargaining
Dir. BENEDICTO ERNESTO R.BITONIO Unit'. On this occasion, appellants were
JR. of the Bureau of Labor Relations et elected as USTFUs new set of officers. The
al. election of the appellants came about upon
G.R. No. 131235 November 16, 1999 a motion of one Atty. Lopez, admittedly not
a member of USTFU, that the USTFU CBL
FACTS: and 'the rules of the election be suspended
Marino, et. al. are duly elected officers and that the election be held on that day.
of the UST Faculty Union (USTFU). The
union has a subsisting five-year Collective The appellees filed the instant petition
Bargaining Agreement with its employer seeking injunctive reliefs and the
(UST). nullification of the results of the
election. Appellees alleged that the holding
Collantes, in her capacity as Secretary of the same violated the temporary
General of USTFU, posted a notice restraining order issued. Accusing
addressed to all USTFU members appellants of usurpation, appellees
announcing a general assembly which was characterized the election as spurious for
called to elect USTFUs next set of being violative of USTFUs CBL.
officers. Some of the appellants filed a
separate petition with the Med-Arbiter, ISSUE: Whether or not the public
alleging that the COMELEC was not respondent committed grave abuse of
constituted in accordance with USTFUs discretion in refusing to recognize the
constitution and by-laws (CBL) and that no officers elected during the general
rules had been issued to govern the assembly.
conduct of the election.
HELD: No.
Thereafter, the secretary general of
UST, upon the request of the various UST Self-organization is a fundamental right
faculty club presidents issued notices guaranteed by the Philippine Constitution
allowing all faculty members to hold a and the Labor Code. Employees have the
convocation. Denominated as a general right to form, join or assist labor
faculty assembly, the convocation was organizations for the purpose of collective
supposed to discuss the state of the bargaining or for their mutual aid and
unratified UST-USTFU CBA and status and protection. Whether employed for a definite
election of USTFU officers. The med-arbiter period or not, any employee shall be
issued a temporary restraining order against considered as such, beginning on his first
herein appellees enjoining them from day of service, for purposes of membership
conducting the election scheduled. in a labor union. Corollary to this right is the
prerogative not to join, affiliate with or assist
The general faculty assembly was held a labor union. Therefore, to become a union
as scheduled. The general assembly was member, an employee must, as a rule, not
attended by members of the USTFU and only signify the intent to become one, but

38
| Labor Relations-2A

also take some positive steps to realize that bargaining unit is entitled to vote in said
intent. The procedure for union membership election. However, the reverse is not always
is usually embodied in the union’s true; an employee belonging to the
constitution and bylaws. An employee who appropriate bargaining unit but who is not a
becomes a union member acquires the member of the union cannot vote in the
rights and the concomitant obligations that union election, unless otherwise authorized
go with this new status and becomes bound by the constitution and bylaws of the
by the union’s rules and regulations. If a union. Verily, union affairs and elections
member of a union dislikes the provisions of cannot be decided in a non-union activity.
the by-laws, he may seek to have them
amended or may withdraw from the union; In both elections, there are procedures
otherwise, he must abide by them. It is not to be followed. Thus, the October 4, 1996
the function of courts to decide the wisdom election cannot properly be called
or propriety of legitimate by-laws of a trade a union election, because the procedure laid
union. The agreement of a member on down in the USTFUs CBL for the election of
joining a union to abide by its laws and officers was not followed. It could not have
comply with the will of the lawfully been a certification election either, because
constituted majority does not require a representation was not the issue, and the
member to submit to the determination of proper procedure for such election was not
the union any question involving his followed. The participation of non-union
personal rights. members in the election aggravated its
irregularity.
Union Election vs. Certification Election

A union election is held pursuant to the


union’s constitution and bylaws, and the
right to vote in it is enjoyed only by
union members. A union election should be
distinguished from a certification election,
which is the process of determining, through
secret ballot, the sole and exclusive
bargaining agent of the employees in the
appropriate bargaining unit, for purposes of
collective bargaining. Specifically, the
purpose of a certification election is to
ascertain whether or not a majority of the
employees wish to be represented by a
labor organization and, in the affirmative
case, by which particular labor organization.

In a certification election, all employees


belonging to the appropriate bargaining unit
can vote. Therefore, a union member who
likewise belongs to the appropriate

39
| Labor Relations-2A

payment for services rendered by union


officers, consultants and others." There was
also an additional proviso stating that the
"matter of allocation ... shall be at the
45_Mallari
discretion of our incumbent Union
President."

PALACOL VS FERRER-CALLEJA This "Authorization and CBA Ratification"


was obtained by the Union through a secret
G.R NO. 85333. February 26, 1990 referendum held in separate local
membership meetings on various dates.
The total membership of the Union was
about 800. 528 of 800 are withdrawing or
FACTS: disauthorizing the deduction of any amount
from their CBA lump sum leaving a
On October 12, 1987, the respondent remainder of 272 as supporters.
Manila CCBPI Sales Force Union On account of the above-mentioned
(hereinafter referred to as the Union), as the disauthorization, the Company filed an
collective bargaining agent of all regular action for interpleader with the Bureau of
salesmen, regular helpers, and relief Labor Relations in order to resolve the
helpers of the Manila Plant and Metro conflicting claims of the parties concerned.
Manila Sales Office of the respondent Coca- Petitioners filed a motion/complaint for
Cola Bottlers (Philippines), Inc. (hereinafter intervention therein in two groups of 161
referred to as the Company) concluded a and 94, respectively. They claimed to be
new collective bargaining agreement with among those union members who either did
the latter. Among the compensation benefits not sign any individual written authorization,
granted to the employees was a general or having signed one, subsequently
salary increase to be given in lump sum withdrew or retracted their signatures
including recomputation of actual therefrom.
commissions earned based on the new
rates of increase.
Petitioners assailed the 10% special
On the same day, the president of the Union assessment as a violation of Article 241(o)
submitted to the Company the ratification by in relation to Article 222(b) of the Labor
the union members of the new CBA and Code and cited Galvadores v. Trajano
authorization for the Company to deduct wherein it was ruled that no check-offs from
union dues equivalent to P10.00 every any amount due employees may be
payday or P20.00 every month and, in effected without individual written
addition, 10% by way of special authorizations duly signed by the
assessment, from the CBA lump-sum pay employees specifically stating the amount,
granted to the union members. The last one purpose, and beneficiary of the deduction.
among the aforementioned is the subject of
the instant petition. In its answer, the Union countered that the
deductions not only have the popular
As embodied in the Board Resolution of the indorsement and approval of the general
Union dated September 29, 1987, the membership, but likewise complied with the
purpose of the special assessment sought
legal requirements of Article 241 (n) and (o)
to be levied is "to put up a cooperative and
credit union; purchase vehicles and other of the Labor Code in that the board
items needed for the benefit of the officers resolution of the Union imposing the
and the general membership; and for the questioned special assessment had been

40
| Labor Relations-2A

duly approved in a general membership purpose. In addition, the secretary of the


meeting and that the collection of a special organization must record the minutes of
fund for labor education and research is the meeting which, in turn, must include,
among others, the list of all the members
mandated.
present as well as the votes cast.

ISSUE:
It submitted only minutes of the local
Can a special assessment be validly
deducted by a labor union from the lump- membership meetings when what is
sum pay of its members, granted under a required is a written resolution adopted at
collective bargaining agreement (CBA), the general meeting. Worse still, the
notwithstanding a subsequent minutes of three of those local meetings
disauthorization of the same by a majority of held were recorded by a union director and
the union members?
not by the union secretary. The minutes
submitted to the Company contained no list
HELD: of the members present and no record of
the votes cast. Since it is quite evident that
No. the Union did not comply with the law at
every turn, the only conclusion that may be
After a careful review of the records of this
made therefrom is that there was no valid
case, We are convinced that the deduction
of the 10% special assessment by the levy of the special assessment pursuant to
Union was not made in accordance with the paragraph (n) of Article 241 of the Labor
requirements provided by law. Code.

Petitioners are correct in citing the ruling of


this Court in Galvadores which is applicable
to the instant case. The principle "that Paragraph (o) on the other hand requires an
individual written authorization duly signed
employees are protected by law from
by every employee in order that a special
unwarranted practices that diminish their
compensation without their known edge and assessment may be validly checked-off.
Even assuming that the special assessment
consent" is in accord with the constitutional
was validly levied pursuant to paragraph (n),
principle of the State affording full protection
and granting that individual written
to labor.
authorizations were obtained by the Union,
nevertheless there can be no valid check-off
considering that the majority of the union
The applicable provisions are clear. The members had already withdrawn their
Union itself admits that both paragraphs individual authorizations. A withdrawal of
(n) and (o) of Article 241 apply.
Paragraph (n) refers to "levy" while individual authorizations is equivalent to no
paragraph (o) refers to "check-off" of a authorization at all. Hence, the ruling in
special assessment. Both provisions Galvadores that "no check-offs from any
must be complied with. Under paragraph amounts due employees may be effected
(n), the Union must submit to the without an individual written authorization
Company a written resolution of a signed by the employees ... " is applicable.
majority of all the members at a general
membership meeting duly called for the

41
| Labor Relations-2A

Moreover, it is well-settled that "all 46_Panganiban


doubts in the implementation and
interpretation of the provisions of the
Labor Code ... shall be resolved in favor ABS CBN Supervisors Employees Union
of labor." And as previously stated, Member v. ABS CBN Broadcasting Corp.,
labor in this case refers to the union et al.
members, as employees of the
Company. Their mere desire to establish GR No.106518 March 11,1999
a separate bargaining unit, albeit
unproven, cannot be construed against
them in relation to the legality of the
FACTS:
questioned special assessment. On the
contrary, the same may even be taken to ABS CBN Broadcasting Corporation
reflect their dissatisfaction with their and the union officers of the petitioner
bargaining representative, the concluded a Collective Bargaining
respondent-Union, as shown by the Agreement with a check- off provision which
circumstances of the instant petition, provides for a 10% deduction from salary
and with good reason. increases and signing bonuses granted to
supervisors. This is in view of the advance
made by the corporation to the union to
cover union’s incidental expenses,
attorney’s fees, and representation
expenses for its organization. Petitioner filed
a complaint against the corporation and
union officers praying for suspension of
further deductions. The union alleged that
the check off provision was illegal for failure
to comply with Article 241, paragraphs (g),
(n), and (o) of the Labor Code and with the
By-Laws of the Union, particularly its
submission to all the members for
consideration and approval at a general
membership meeting called for that
purpose.

On the other hand, respondent union


officers and corporation contended that the
provision was in accordance with law as
majority of the union members individually
executed a written authorization giving the
union officers and corporation a blanket
authority to make deductions.

42
| Labor Relations-2A

ISSUE: Whether or not the check off 47_Questin


provision is legal and valid
HOLY CROSS OF DAVAO COLLEGE,
HELD: INC. v. HON. JEROME JOAQUIN, in his
capacity as Voluntary Arbitrator
Yes, because all of the 3 requisites
in order that the special assessment for G.R. No. 110007, October 18, 1996
union’s incidental expenses, attorney’s fees,
and representation expenses were complied
with. Check- off is a process whereby the DOCTRINE OF THE CASE
employer, by agreement with the union,
deducts union dues or agency fees from the The employer’s failure to make the
wages and remits them directly to the union. requisite deductions may constitute a
It is primarily for the benefit of union as it violation of a contractual commitment for
assures continuous funding. Its statutory which it may incur liability for unfair labor
limitations can be found in Article 241 of practice. But it does not by that omission,
Labor Code which deals with rights and incur liability to the union for the aggregate
conditions of membership in a labor of dues or assessments uncollected from
organization. The provision speaks of 3 the union members, or agency fees for non-
requisites: a) authorization by a written union employees.
resolution of majority of all members at the
general membership meeting; b) secretary’s
record of minutes of the meeting; and c)
FACTS:
individual written authorization duly signed
by employee concerned. First, the union A collective bargaining agreement
held a general meeting where they agreed (CBA) was entered into between Holy Cross
on the 10% check off to cover their of Davao College, Inc. (Holy Cross), an
expenses. Second, the union’s secretary educational institution, and the affiliate labor
recorded the minutes of the said meeting organization representing its employees,
and was noted by the president. Third, 85 of Holy Cross of Davao College Union-
the members of the union executed KAMAPI (KAMAPI). Shortly before the
individual written authorizations for check- expiration of the agreement, KAMAPI
off and records did not indicate that they President Jose Lagahit, wrote Holy Cross
were executed under influence of force or expressing his union’s desire to renew the
compulsion nor withdrawn by them. The agreement, withal seeking its extension for
Court affirmed validity of check off two months, on the ground that the teachers
provision but, ruled that deductions were still on summer vacation and union
must be made only to workers who gave activities necessary or incident to the
individual written check- off negotiation of a new agreement could not
authorization. yet be conducted. The Holy Cross President
replied that he had no objection to the
extension sought, it being allowable under
the collective bargaining agreement.

43
| Labor Relations-2A

sent a letter to the School asking for its


counter-proposals. In response, Holy Cross
A meeting of the KAMAPI declared that it would take no action
membership for the purpose of electing a towards a new CBA without a definitive
new set of union officers was conducted. ruling on the proper interpretation of Article I
Rodolfo Gallera won election as president. of the old CBA. Said Article provides inter
Gallera forthwith initiated discussions for the alia for the automatic extension of the CBA
union’s disaffiliation from the KAMAPI for another period of three (3) years counted
Federation. Gallera’s group subsequently from its expiration, if the parties fail to agree
formed a separate organization known as on a renewal, modification or amendment
the Holy Cross of Davao College Teachers thereof.
Union, and elected its own officers.

There has been a dispute regarding


Holy Cross thereafter stopped the proper interpretation of items 1 and 2 of
deducting from the salaries and wages of its Article I of the CBA. After several failed
teachers and employees the corresponding conciliation meetings, KAMAPI and Holy
union dues and special assessment, and Cross referred the case to voluntary
agency fees, in accordance with the check- arbitration. Voluntary Arbitrator Joaquin
off clause of the CBA. ruled in favor of KAMAPI. He ruled that the
request for extension filed by KAMAPI
constituted seasonable notice of its intention
In the meantime, there ensued to renew, modify or amend the agreement,
between the two unions a full-blown action which it could not however pursue because
on the basic issue of representation, which of the absence of the teachers who were
was to last for some two years. It began then on summer vacation. Further, he
with the filing by the new union of a petition ordered Holy Cross to pay to KAMAPI the
for certification election in the Office of the amount equivalent to the uncollected union
Med-Arbiter. KAMAPI responded by filing a dues, it appearing that Holy Cross stopped
motion asking the Med-Arbiter to dismiss complying with the CBA’s check-off
the petition. The Med-Arbiter denied provisions.
KAMAPI’s motion to dismiss, and ordered
the holding of a certification election. On
appeal, however, the Secretary of Labor Aggrieved, Holy Cross filed the
reversed the Med-Arbiter’s ruling and instant petition.
ordered the dismissal of the petition for
certification election, which action was
eventually sustained by the Supreme Court
in appropriate proceedings. ISSUE: Is Holy Cross liable to pay to
KAMAPI, the amounts it failed to deduct
from KAMAPI’s members as union dues, or
agency fees as regards those not union
Thereafter, KAMAPI presented its members, in accordance with the check-off
bargaining proposals to Holy Cross; and it provisions of the CBA?

44
| Labor Relations-2A

In this aspect, the legal basis of the union’s


right to agency fees is neither contractual
RULING: No. nor statutory, but quasi-contractual, deriving
from the established principle that non-
union employees may not unjustly enrich
A check-off is a process or device themselves by benefiting from employment
whereby the employer, on agreement with conditions negotiated by the bargaining
the union recognized as the proper union.
bargaining representatives, or on prior
authorization from its employees, deducts
union dues or agency fees from the latter's No provision of law makes the
wages and remits them directly to the union. employer directly liable for the payment to
Its desirability to a labor organization is the labor organization of union dues and
quite evident; by it, it is assured of assessments that the former fails to deduct
continuous funding. Indeed, this Court has from its employees salaries and wages
acknowledged that the system of check-off pursuant to a check-off stipulation. The
is primarily for the benefit of the union and, employer’s failure to make the requisite
only indirectly, of the individual laborers. deductions may constitute a violation of a
The legal basis of check-off is thus found in contractual commitment for which it may
statute or in contract. Statutory limitations incur liability for unfair labor practice. But it
on check-offs generally require written does not by that omission, incur liability to
authorization from each employee to deduct the union for the aggregate of dues or
wages; however, a resolution approved and assessments uncollected from the union
adopted by a majority of the union members members, or agency fees for non-union
at a general meeting will suffice when the employees.
right to check-off has been recognized by
the employer, including collection of
reasonable assessments in connection with
mandatory activities of the union, or other Where the employer fails or refuses
special assessments and extraordinary to implement a check-off agreement, logic
fees. and prudence dictate that the union itself
undertake the collection of union dues and
assessments from its members (and agency
fees from non-union employees); this, of
On the other hand, the collection of course, without prejudice to suing the
agency fees in an amount equivalent to employer for unfair labor practice.
union dues and fees, from employees who
are not union members, is recognized by
Article 248 (e) of the Labor Code. No
requirement of written authorization from the Therefore, there was no basis for the
non-union employee is imposed. The Voluntary Arbitrator to require Holy Cross to
employees’ acceptance of benefits resulting assume liability for the union dues and
from a collective bargaining agreement assessments, and agency fees that it had
justifies the deduction of agency fees from failed to deduct from its employees’
his pay and the union’s entitlement thereto. salaries.

45
| Labor Relations-2A

48_Reonico negotiate as representatives. The CA


affirmed the NLRC decision, but it ordered
the reinstatement with back wages of union
Philippine Diamond Hotel vs. Manila members. Hence, this present case.
Diamond Hotel Union

G.R. No. 158075, June 30, 2006 ISSUE

Whether or not there is a violation of


FACTS the constitutional right to self-organization
(when the Hotel intimidated some workers
The Diamond Hotel Union sought for from joining the Union before, during or after
a certification as the exclusive bargaining the strike)
representative of its members. However,
DOLE-NCR denied for failure to comply with RULING
the requirements. The Union informed the NO. The Court observed that the
hotel of its intention to negotiate for petition of the Union contained general
collective bargaining agreement. The Hotel allegations that the Hotel committed unfair
rejected such because the Union was not labor practices by refusing to bargain with
certified by DOLE as the exclusive the union and y alleged acts of union
bargaining agent. Hence, the Union filed a interference, coercion and discrimination
notice to strike due to unfair labor practice tantamount to union-busting. The burden to
against the Hotel who refused to negotiate prove these allegations by substantial
with the Union. evidence lies with the Union. While the
Union continues to accuse the Hotel of
violating their constitutional right to organize
Suddenly, the Union went on a strike by busting the Union, the Court cannot
despite several conferences. The entrances overlook the events that transpired prior to
were blocked. The NLRC issued a the strike that the Union staged on
temporary restraining order pursuant to the November 29, 1997. It is beyond argument
ocular inspection of the NLRC that a conciliatory meeting was still
representative who confirmed that the scheduled to be held on December 1, 1997
strikers obstructed the free ingress to and before the NCMB. In this conciliatory
egress from the Hotel. The TRO directed meeting, the Union could have
the strikers to cease and desist from substantiated and presented additional
obstructing the entrances. In its evidence. Thus, as held by the Supreme
implementation, the strikers resisted and Court in the case of Tiu vs. National Labor
some were injured. NLRC ruled that the Relations Commission:
strike was illegal; and the termination of
union officers was based on their
participation in an illegal strike. The Union "The Court is not unmindful of this
averred that the termination equates to rule, but in the case at bar the facts
union busting and unfair labor practice and the evidence did not establish
because in fact they had the capacity to

46
| Labor Relations-2A

events, least a rational basis why the Labor Code is misplaced for not every
union would wield a strike based on legitimate labor organization possesses the
alleged unfair labor practices; it did rights mentioned therein. Article 242 (a)
not even bother to substantiate must be read in relation to Article 255.
during the conciliation proceedings. Finally, the Court affirmed the decision of
It is not enough that the union the NLRC and CA but modified in terms that
believed that the employer only those members of the Union who did
committed acts of unfair labor not commit illegal acts during the course of
practice when the circumstances the illegal strike should be reinstated but
clearly negate even prima facie without back wages.
showing to warrant such a belief."

It is also evident from the records of


the instant petition, specifically from the
Notice of Strike, that their principal ground
for the strike was the "refusal of the Hotel
Management to bargain collectively with the
Union for the benefit of the latter’s
members." In the instant case, it is not
disputed that the UNION is not a certified
bargaining unit to negotiate a collective
bargaining agreement with the Hotel.
Hence, the Court notes that the Union
violated Article 264 which proscribes the
staging of a strike on the ground of ULP
during the pendency of cases involving the
same grounds for the strike.

The Court finds the strike illegal as


the Union is admittedly not the exclusive
representative of the employees of the
Hotel; hence, it could not demand from the
Hotel the right to bargain collectively in
behalf of the employees. Article 255 of the
Labor Code declares that only the labor
organization designated or selected by the
majority of the employees in an appropriate
collective bargaining unit is the exclusive
representative of the employees in such unit
for the purpose of collective bargaining. The
reliance of the Union to Article 242 of the

47
| Labor Relations-2A

49_Terre Employees Union, has already signed a


CBA with the Corporation, making the
ME-SHURN CORP. v. ME-SHURN UNION certification election moot and academic.
G.R. No. 156292 January 11, 2005 LA dismissed Union’s petitions, NLRC
FACTS: reversed the decision and denied the MR,
while CA likewise dismissed the
Rank and file employees organized the Me- Corporation’s petition.
Shurn Workers Union-FSM (February Six
ISSUES:
Movement) in July 1998. While their
application for registration was pending 1. W/N the dismissal of employees
before the Bureau of Labor Relations, the constitute unfair labor practice.
Corporation placed the members of the 2. W/N respondent union is legitimate.
union’s bargaining unit on forced leave. The
Union filed a Petition for Certification
HELD:
Election before the Med-Arbiter Department
of DOLE. Instead of filing an Answer, the 1. YES. The Corporation has not
Corporation filed a comment stating that satisfactorily explained why the workers
they will lay off employees and cease dismissal was effected only after the
operations due to its inability to meet the formation of respondent union in September
export quota required by the Board of 1998. We also take note of the allegation
Investment. Thus, the Union withdrew its that after several years of attempting to
organize a union, the employees finally
petition. But later on the Union’s appeal was
succeeded on June 7, 1998. Ten days later,
granted by the DOLE Usec, and they were without any valid notice, all of them were
allowed to hold the certification election. placed on forced leave, allegedly because
of lack of quota. All these considerations
give credence to their claim that the closure
of the corporation was a mere subterfuge, a
The Corporation rehired the employees on systematic approach intended to dampen
the condition that they sign an agreement the enthusiasm of the union members.
that no union will be organized upon their
return. After signing the agreement, Furthermore, as a condition for the
operations resumed, but the rank and file rehiring of the employees, the union officers
were made to sign an agreement that they
employees organized the union once more
would not form any union upon their return
and elected Rosalina Cruz as president. to work. This move was contrary to law.
They filed two cases for compulsory Notwithstanding the Petition for Certification
arbitration, citing unfair labor practice, illegal Election filed by respondents and despite
dismissal, underpayment of wages and knowledge of the pendency thereof,
deficiency in separation pay. petitioners recognized a newly formed union
and hastily signed with it an alleged
Collective Bargaining Agreement. Their
preference for the new union was at the
The Corporation reiterated they had to close expense of respondent union.
down business due to losses. They also
questioned the legality of the Union, alleging All these factors strongly give credence
to the contention of respondents that the
that another one, Me-Shurn Independent

48
| Labor Relations-2A

real reason behind the shutdown of the 50_Santos


corporation was the formation of their union.
Note that, to constitute an unfair labor ACEDERA ET AL. V. INTERNATIONAL
practice, the dismissal need not entirely and CONTAINER TERMINAL SERVICES, INC.,
exclusively be motivated by the unions NLRC, & CA
activities or affiliations. It is enough that the
discrimination was a contributing factor. If GR No. 146073, 13 January 2003
the basic inspiration for the act of the
employer is derived from the affiliation or Facts:
activities of the union, the formers
assignment of another reason, no matter Jerry Acedera, et al. are employees of
how seemingly valid, is unavailing. International Container Terminal Services,
Inc. (ICTSI) and are members of Associated
2. YES. It should be pointed out that it
Port Checkers & Workers Union-
filed a Petition for Certification Election.
While this Petition was initially dismissed by International Container Terminal Services,
the med-arbiter on the basis of a supposed Inc.(APCWU-ICTSI), a duly registered labor
retraction, note that the appeal was granted. organization. ICTSI went on a retrenchment
The DOLE would not have entertained the program and laid off its on-call employees.
Petition if the union were not a legitimate This prompted the APCWU to file a notice of
labor organization within the meaning of the strike which included as cause of action not
Labor Code. Under this Code, in an
unorganized establishment, only a only the retrenchment of the employees but
legitimate union may file a petition for also ICTSI's use of 365 days as divisor in
certification election. the computation of wages, even if the
Verily, the union has the requisite employees' work week consisted only of five
personality to sue in its own name in order days as agreed upon in the Collective
to challenge the unfair labor practice Bargaining Agreement (CBA).
committed by petitioners against it and its
members. It would be an unwarranted
impairment of the right to self-organization
The dispute respecting the retrenchment
through formation of labor associations if was resolved by a compromise settlement,
thereafter such collective entities would be
while that respecting the computation of
barred from instituting action in their
wages was referred to the Labor Arbiter.
representative capacity.
Subsequently, APCWU, on behalf of its
members and other employees similarly
situated, filed with the Labor Arbiter a
Finally, in view of the discriminatory complaint against ICTSI which was
acts committed by petitioners against dismissed. Petitioners filed with the Labor
respondent union -- acts that included their Arbiter a Complaint-in-Intervention with
immediate grant of exclusive recognition to Motion to Intervene, but the same was
another union as a bargaining agent despite denied upon finding that they are already
the pending Petition for certification election well represented by APCWU. The denial of
-- the results of that election cannot be said petitioners' intervention was affirmed by the
to constitute a repudiation by the affected NLRC. Petitioners filed a petition for
employees of the unions right to represent certiorari with the Supreme Court which
them in the present case. referred the petition to the CA. The CA

49
| Labor Relations-2A

dismissed the petition. Hence, this present faith on the part of the representative must
petition. be proven. It must be based on facts borne
on record. Mere assertions, as what
petitioners-appellants proffer, do not suffice.
Issue: Whether petitioners have a legal right
to intervene and pursue the case INOCENTES_51

Held: Golden Donuts, Inc. v. NLRC [G.R. Nos.


113666-68, January 19, 2000.]

Wednesday, January 21, 2009 Posted


No. A labor union is one such party by Coffeeholic Writes
authorized to represent its members under Labels: Case Digests, Labor Law
Article 242 (a) of the Labor Code which
provides that a union may act as the
representative of its members for the FACTS: Private respondents were the
purpose of collective bargaining. This complainants in three consolidated cases
authority includes the power to represent its submitted with the Labor Arbiter.
members for the purpose of enforcing the Complainants were members of the KMDD-
provisions of the CBA. CFW whose CBA with the
corporation expired. During the
negotiations, the management panel arrived
late causing the union panel to walk out.
While a party acting in a representative
The management addressed a letter of
capacity, such as a union, may be permitted
apology to the union and requested for
to intervene in a case, ordinarily, a person
negotiations to resume. The union panel did
whose interests are already represented will
not show up despite letters from
not be permitted to do the same except
management advising the former of the
when there is a suggestion of fraud or
CBA meetings. The union struck. A
collusion or that the representative will not
compliant was filed by Golden Donuts to
act in good faith for the protection of all
declare the strike illegal. Counsel for the
interests represented by him.
union and strikers pleaded for a
compromise whereupon both parties would
desist from continuing their cases against
Petitioners cite the dismissal of the case each other. The Labor Arbiter rendered a
filed by APCWU, first by the Labor Arbiter, decision upholdingthe dismissal of private
and later by the CA. The dismissal of the respondents and ruling that they were
case does not, however, by itself show the bound by the compromise agreement
existence of fraud or collusion or a lack of entered into by the union with petitioners.
good faith on the part of APCWU. Private respondents appealed to the NLRC,
claiming that the union had no
To reiterate, for a member of a class to be authority to waive or compromise their
permitted to intervene in a representative individual rights and they were not bound by
action, fraud or collusion or lack of good

50
| Labor Relations-2A

the compromise agreement entered into by


the union with petitioners.

ISSUE: Whether or not a union may


compromise or waive the right to security
of tenure and money claims of its minority
members, without the latter’s consent.

HELD: No. Absent a showing of the union’s


special authority to compromise the
individual claims of private respondents for
reinstatement and backwages, there is no
valid waiver of the aforesaid rights. The
judgment of the Labor Arbiter based on the
compromise agreement does not have the
effect of res judicata upon private
respondents who did not agree thereto
since the requirement of identity of parties is
not satisfied. A judgment upon a
compromise agreement has all the force
and effect of any other judgment and is
conclusive only upon parties thereto and
their privies. Private respondents have not
waived their right to security oftenure nor
can they be barred from entitlement of their
individual claims. Since there was no
evidence that private respondents
committed any illegalact, petitioner’s failure
to reinstate them after the settlement of the
strike amounts to illegal dismissal.

51
| Labor Relations-2A

RIGHTS OF LEGITIMATE strike. On December 4, 1997, former Labor


Secretary Leonardo A. Quisumbing, now
LABOR ORGS
Associate Justice of this Court, issued an
52_Palacios Order assuming jurisdiction over the labor
dispute and ordering all striking workers to
return to work and the management to
resume normal operations.
CAPITOL MEDICAL CENTER, INC. vs.
HON. CRESENCIANO B. TRAJANO and
CAPITOL MEDICAL CENTER
EMPLOYEES ASSOCIATION-AFW Meantime, on October 1, 1998, the
Regional Director issued an Order
denying the petition for cancellation of
respondent union’s certificate of registration.
Facts: Capitol Medical Center,
On September 20, 2001, the Appellate
Inc., petitioner, is a hospital with address at
Court rendered a Decision affirming the
Panay Avenue corner Scout Magbanua
Orders of the Secretary of Labor. The Court
Street, Quezon City. Upon the other hand,
of Appeals held: In order to allow
Capitol Medical Center Employees
an employer to validly suspend the
Association-Alliance of Filipino Workers,
bargaining process, there must be a
respondent, is a duly registered labor union
valid petition for certification election. The
acting as the certified collective bargaining
mere filing of a petition does not ipso
agent of the rank-and-file employees of
facto justify the suspension of negotiation by
petitioner hospital. On October 2, 1997,
the employer.
respondent union, through its president
Jaime N. Ibabao, sent petitioner a letter If pending a petition for certification, the
requesting a negotiation of their Collective collective bargaining is allowed by the
Bargaining Agreement (CBA).In its reply Supreme Court to proceed, with more
dated October 10, 1997, petitioner, reason should the collective bargaining (in
challenging the union’s legitimacy, refused this case) continue since the High Court had
to bargain with respondent. Subsequently or recognized the respondent as the certified
on October15, 1997, petitioner filed with the bargaining agent in spite of several petitions
Bureau of Labor Relations (BLR), for cancellation filed against it.
Department of Labor and Employment, a
petition for cancellation of respondent’s Issue: Whether the petition for
certificate of registration. the cancellation of respondent union’s
certificate of registration involves a
For its part, on October 29, 1997, prejudicial question that should first be
respondent filed with the NCMB, National settled before the Secretary of Labor
Capital Region, a notice of strike. could order the parties to bargain
Respondent alleged that petitioner’s refusal collectively.
to bargain constitutes unfair labor practice.
Despite several conferences and efforts of
the designated conciliator-mediator, the
Held: No. As aptly stated by the Solicitor
parties failed to reach an amicable
General in his comment on the petition, the
settlement. Thus, respondent staged a
52
| Labor Relations-2A

Secretary of Labor correctly ruled that the


pendency of a petition for cancellation of
union registration does not preclude 53_Talens
collective bargaining, thus: "That there is
a pending cancellation proceedings against
the respondent Union is not a bar to set
in motion the mechanics of collective
bargaining. If a certification election may still
be ordered despite the pendency of
a petition to cancel the union’s registration
certificate, more so should the collective
bargaining process continue despite its
pendency.

We must emphasize that the majority status


of the respondent Union is not affected by
the pendency of the Petition for Cancellation
pending against it. Unless its certificate of
registration and its status as the certified
bargaining agent are revoked, the Hospital
is, by express provision of the law, duty
bound to collectively bargain with the Union.
Indeed, no less than the Supreme Court
already ordered the Hospital to collectively
bargain with the Union when it affirmed the
resolution of this Office dated November 18,
1994 directing the management of the
Hospital to negotiate a collective bargaining
agreement with the Union.

Moreover, as mentioned earlier, during the


pendency of this case before the Court of
Appeals, the Regional Director issued an
Order on October 1, 1998 denying the
petition for cancellation of respondent’s
certificate of registration. This Order
became final and executory and recorded in
the BLR’s Book of Entries of Judgments on
June 3,1999.

53
| Labor Relations-2A

54_Acosta filed a complaint for ULP and Damages against


the Union alleging that the union did not bargain
STANDARD CHARTERED BANK in good faith; that the Union demanded "sky high
EMPLOYEES UNION V. CONFESOR AS economic demands," indicative of blue-sky
bargaining. It contended further, that the Union
SOLE AND STANDARD CHARTERED
violated its no strike- no lockout clause by filing
BANK
a notice of strike before the NCMB; that
considering the filing of notice of strike was an
FACTS: Standard Chartered Bank (the Bank) is
illegal act, the Union officers should be
a foreign banking corporation doing business in
dismissed; and that the bank suffered nominal
the Philippines. The exclusive bargaining agent
and actual damages and was forced to litigate
is the Standard Chartered Bank Employees
and hire the services of a lawyer.
Union (the Union).

The SOLE assumed jurisdiction over the labor


In August 1990, the Bank and the Union signed
dispute and ordered the parties to execute a
a five year CBA with a provision to renegotiate
CBA incorporating the dispositions. The SOLE
after 3 years. Prior to the expiration of the three
dismissed the charges of ULP of both the Union
year period and before the 60 day freedom
and the Bank. Both parties filed MFR but to no
period, the union initiated the negotiations
avail. On March 22, 1994, the parties signed the
followed by a counterproposal of the Bank. The
CBA. On April 28, 1994, the Union filed this
parties agreed to settle the differences in a
petition for certiorari under Rule 65 of the Rules
meeting.
of Procedure.

Before the commencement of the negotiation,


ISSUE: Whether or not the Bank is guilty of
the Union, through Divinagracia, suggested to
the Bank’s Human Resource Manager and head ULP because of its alleged: “interference”
of the negotiating panel, Cielito Diokno, that the with its choice of negotiator, surface
bank lawyers should be excluded from the bargaining, and refusal to furnish the Union
negotiating team. The Bank acceded. with copies of the relevant data.
Meanwhile, Diokno suggested to Divinagracia
that Jose P. Umali, Jr., the President of the HELD: NO.
National Union of Bank Employees (NUBE), the
federation to which the Union was affiliated, be Interference
excluded from the Union’s negotiating panel.
However, Umali was retained as a member The circumstances that occurred during the
thereof. negotiation do not show that the suggestion
made by Diokno to Divinagracia is an anti-
Both parties agreed to place the notation union conduct from which it can be inferred
“DEFERRED/DEADLOCKED” on the proposed
that the Bank consciously adopted such act
non-economic provisions of the CBA as they
to yield adverse effects on the free exercise
were unable to reach an agreement.
of the right to self-organization and
Days after, the negotiation for economic collective bargaining of the employees,
provisions commenced. Except for the especially considering that such was
provisions on signing bonus and uniforms, the undertaken previous to the commencement
Union and the Bank failed to agree on the of the negotiation and simultaneously with
remaining economic provisions of the CBA. The Divinagracia’s suggestion that the bank
Union, then declared deadlock and filed a notice lawyers be excluded from its negotiating
of strike before NCMB. Consequently, the Bank panel. It is clear that such ULP charge was

54
| Labor Relations-2A

merely an afterthought. The accusation practice, and also supports the inference of
occurred after the arguments and surface bargaining, in the case at bar,
differences over the economic provisions Umali, in a meeting dated May 18, 1993,
became heated and the parties had become requested the Bank to validate its
frustrated. guestimates on the data of the rank and file.
However, Umali failed to put his request in
Surface bargaining writing as provided for in Article 242(c) 1 of
Surface bargaining is defined as going the Labor Code.
through the motions of negotiating without
any legal intent to reach an agreement. The The Union, did not, as the Labor Code
determination of whether a party has requires, send a written request for the
engaged in unlawful surface bargaining is issuance of a copy of the data about the
usually a difficult one because it involves Banks rank and file employees. Moreover,
the question of whether an employer’s as alleged by the Union, the fact that the
conduct demonstrates an unwillingness to Bank made use of the aforesaid
bargain in good faith or is merely hard guestimates, amounts to a validation of the
bargaining. data it had used in its presentation.

The minutes of meetings from March 12, *1Article 242. Rights of Legitimate Labor
1993 to June 15, 1993 do not show that the Organization
Bank had any intention of violating its duty
to bargain with the Union. The minutes of (c) To be furnished by the employer, upon
the meetings show that both the Bank and written request, with the annual audited
the Union exchanged economic and non- financial statements, including the balance
economic proposals and counter-proposals. sheet and the profit and loss statement,
within thirty (30) calendar days from the
The Union has not been able to show that date of receipt of the request, after the
the Bank had done acts, both at and away union has been duly recognized by the
from the bargaining table, which tend to employer or certified as the sole and
show that it did not want to reach an exclusive bargaining representatives of the
agreement with the Union or to settle the employees in the bargaining unit, or within
differences between it and the Union. sixty (60) calendar days before the
Admittedly, the parties were not able to expiration of the existing collective
agree and reached a deadlock. However, it bargaining agreement, or during the
is herein emphasized that the duty to collective negotiation;collective bargaining
bargain does not compel either party to agreement, or during the collective
agree to a proposal or require the making of negotiation;
a concession. Hence, the parties failure to
agree did not amount to ULP under Article
248(g) for violation of the duty to bargain.

Refusal to furnish copies of relevant data


While the refusal to furnish requested
information is in itself an unfair labor

55
| Labor Relations-2A

55_Barrietta he has both jurisdiction and expertise to


make them - are valid.
MANILA ELECTRIC COMPANY V. HON.
SEC. OF LABOR AND MEWA

January 27, 1999 ISSUE

Whether or not the extent of judicial review


over the Secretary of Labor is limited only to
PONENTE: AUSTRIA-MARTINEZ, J.: a determination of grave abuse in the
manner of the secretary’s exercise of his
statutory powers.
FACTS

Meralco and MEWA, renegotiating their


CBA for the remaining two of its five-year HELD
term, deadlocked on such issues as wage The Secretary of Labor’s statutory power
increase, coverage of bargaining unit, under Art. 263 (g) of the Labor Code to
Christmas bonus and so forth. Meralco assume jurisdiction over a labor dispute in
management petitioned the DOLE secretary an industry indispensable to the national
to assume jurisdiction over the deadlock interest, and, to render an award on
dispute. The Secretary rendered an award compulsory arbitration, does not exempt the
and directed the parties to incorporate it in exercise of this power from the judicial
their renegotiated CBA. Objecting to review that Sec. 1, Art. 8 of the Constitution
particular items in the award as excessive mandates. This constitutional provision
or arbitrary, Meralco moved for states:
reconsideration, and the Secretary did
reconsider some of the items. Still
dissatisfied, Meralco charged the Secretary
with grave abuse of discretion before the “Judicial power includes the duty of the
Supreme Court. The Union opposed the courts of justice to settle actual
petition. It disputed the allegation of controversies involving rights which are
MERALCO that the Secretary abused his legally demandable and enforceable, and to
discretion in issuing the assailed orders determine whether or not there has been a
arguing that he acted within the scope of the grave abuse of discretion amounting to lack
powers granted him by law and by the or excess of jurisdiction on the part of any
Constitution. The union contends that any branch or instrumentality of the
judicial review is limited to an examination government.”
of the Secretary’s decision-
making/discretion - exercising process to
determine if this process was attended by Under this constitutional mandate, every
some capricious or whimsical act that legal power of the Secretary of Labor under
constitutes “grave abuse”; in the absence of the Labor Code, or, for that matter, any act
such abuse, his findings - considering that of the Executive, that is attended by grave
abuse of discretion is subject to review by

56
| Labor Relations-2A

this Court in an appropriate proceeding. To 56_Batalier


be sure, the existence of an executive
power alone - whether granted by statute or CREA vs. BRILLANTES and CALTEX
by the Constitution - cannot exempt the (PHILIPPINES), Inc.
executive action from judicial oversight,
interference or reversal when grave abuse G.R. No. 123782, September 16 1997
of discretion is, or is alleged to be, present.
This is particularly true when constitutional
norms are cited as the applicable yardsticks FACTS:
since this Court is the final interpreter of the
meaning and intent of the Constitution. Anticipating the expiration of their CBA on
July 31, 1995, petitioner and private
respondent negotiated the terms and
conditions of employment to be contained in
The extent of judicial review over the a new CBA. The negotiation between the
Secretary of Labor’s arbitral award is not two parties was participated in by the NCMB
limited to a determination of grave abuse in and the Office of the Secretary of Labor and
the manner of the secretary’s exercise of his Employment. Some items in the new CBA
statutory powers. This Court is entitled to, were amicably arrived at and agreed upon,
and must - in the exercise of its judicial but others were unresolved.
power - review the substance of the
Secretary’s award when grave abuse of
discretion is alleged to exist in the award,
i.e., in the appreciation of and the To settle the unresolved issues, eight
conclusions the Secretary drew from the meetings between the parties were
evidence presented. conducted. Because the parties failed to
reach any significant progress in these
meetings, petitioner declared a deadlock.
On July 24, 1995, petitioner filed a notice of
strike. 6 conciliation meetings conducted by
the NCMB failed, failed. Marathon meetings
at the plant level, but this remedy proved
also unavailing.

Secretary assumed jurisdiction and ordered


“Accordingly, any strike or lockout, whether
actual or intended, is hereby enjoined.”xxx
But the members of petitioner defied them
and continued their mass action (despite
repeated orders).

57
| Labor Relations-2A

Thereafter, the contending parties filed their adjustment should the increase fall short of
position papers pertaining to unresolved the inflation rate.
issues. Because of the strike, private
respondent terminated the employment of The alleged “similarity” in the situation of
some officers of petitioner union. The Caltex and Shell cannot be considered a
legality of these dismissals brought valid ground for a demand of wage
additional contentious issues. increase, in the absence of a showing that
the two companies are also similar in
“substantial aspects,” as discussed above.

Again, the parties tried to resolve their True, union members have the right to
differences through conciliation. Failing to demand wage increases through their
come to any substantial agreement, the collective force; but it is equally cogent
parties decided to refer the problem to the that they should also be able to justify an
secretary of labor and employment. appreciable increase in wages. We
observe that private respondent’s detailed
allegations on productivity are unrebutted. It
ISSUE: is noteworthy that petitioner ignored this
argument of private respondent and based
1. Petitioner questions public its demand for wage increase not on the
respondent’s resolution of five ground that they were as productive as the
issues in the CBA, specifically on Shell employees. Thus, we cannot attribute
wage increase, union security grave abuse of discretion to public
clause, retirement benefits or
respondent.
application of the new retirement
plan, signing bonus and grievance
and arbitration machineries; and
2. Whether or not the Honorable 2. Union Security Clause. Petitioner
Secretary of Labor and Employment
committed grave abuse of discretion argues that in spite of the provisions on the
in resolving the instant labor dispute. “union security clause,” it may expel a
member only on any of three grounds: non-
HELD: payment of dues, subversion, or conviction
for a crime involving moral turpitude. If the
The petition is partly meritorious. employee’s act does not constitute any of
these three grounds, the member would
1. Wage Increase. Petitioner maintains that
continue to be employed by private
the salaries of Shell Refinery employees be
respondent. Thus, the disagreement
used as a “reference point” in upgrading the
between petitioner and private respondent
compensation of private respondent’s
on this issue is not only “procedural” but
employees because these two companies
also “substantial.”
are in the “same industry and their refineries
are both in Batangas.” Thus, the wage We agree with petitioner. The
increase of petitioner’s members should be disagreement between petitioner and
“15%/15%/15%.” Private respondent private respondent on the union security
counters with a “proposed 9% 7% 7% clause should have been definitively
increase for the same period with automatic resolved by public respondent. The labor

58
| Labor Relations-2A

secretary should take cognizance of an 4. Grievance Machinery and


issue which is not merely incidental to Arbitration. Petitioner contends that public
but essentially involved in the labor respondent “derailed the grievance and
dispute itself, or which is otherwise arbitration scheme proposed by the
submitted to him for resolution. The Union.” Petitioner’s recommendation for a
secretary of labor assumed jurisdiction over “single arbitrator is based on the proposition
this labor dispute in an industry that if voluntary arbitration should be
indispensable to national interest, precisely resorted to at all, this recourse should entail
to settle once and for all the disputes over the least possible expense.”
which he has jurisdiction at his level. In not
performing his duty, the secretary of labor No particular setup for a grievance
committed a grave abuse of discretion. machinery is mandated by law. Rather,
Article 260 of the Labor Code, as
incorporated by RA 6715, provides for only
a single grievance machinery in the
3. New Retirement Plan. Petitioner company to settle problems arising from
contends that “40 of its members who are “interpretation or implementation of their
still covered by the Old Retirement Plan CBA and those arising from the
because they were not able to exercise the interpretation or enforcement of company
option to shift to the New Retirement Plan, personnel policies.”
for one reason or another, when such option
was given in the past” are included in the We believe that the procedure described by
New Retirement Plan. public respondent sufficiently complies with
the minimum requirement of the law. Public
We hold that public respondent did not respondent even provided for two steps in
commit grave abuse of discretion in hearing grievances prior to their referral to
respecting the free and voluntary decision of arbitration. The parties will decide on the
the employees in regard to the Provident number of arbitrators who may hear a
Plan and the irrevocable one-time option dispute only when the need for it
provided for in the New Retirement arises. Even the law itself does not
Plan. Although the union has every right specify the number of arbitrators. . In
to represent its members in the this matter, cost is not the only
negotiation regarding the terms and consideration; full deliberation on the
conditions of their employment, it cannot issues is another, and it is best
negate their wishes on matters which are accomplished in a hearing conducted by
purely personal and individual to three arbitrators. In effect, the parties are
them. In this case, the forty employees afforded the latitude to decide for
freely opted to be covered by the Old Plan; themselves the composition of the
their decision should be respected. The grievance machinery as they find
company gave them every opportunity to appropriate to a particular situation. At
choose, and they voluntarily exercised their bottom, we cannot really impute grave
choice. The union cannot pretend to know abuse of discretion to public respondent on
better; it cannot impose its will on them. this issue.

59
| Labor Relations-2A

5. Signing Bonus. Petitioner asseverates issues to the secretary of labor for his
that the “signing bonus is an existing benefit resolution, they should not expect their
embodied in the old CBA.” 42 It explains that positions to be adopted in toto. It is
public respondent erred in removing the understood that they defer to his wisdom
award of a signing bonus xxx and objectivity in insuring industrial
peace. And unless they can clearly
Although proposed by petitioner, 45 the demonstrate bias, arbitrariness,
signing bonus was not accepted by private capriciousness or personal hostility on
respondent. 46 Besides, a signing bonus is the part of such public officer, the Court
not a benefit which may be demanded will not interfere or substitute the said
under the law. Rather, it is now claimed by officer’s judgment with its own.
petitioner under the principle of
“maintenance of existing benefits” of the old
CBA. However, as clearly explained by
private respondent, a signing bonus may
not be demanded as a matter of right. If it
is not agreed upon by the parties or
unilaterally offered as an additional
incentive by private respondent, the
condition for awarding it must be duly
satisfied. In the present case, the
condition sine qua non for its grant — a
non-strike — was not complied with. In
fact, private respondent categorically sated
in its counter-proposal — to the exclusion of
those agreed upon before — that the new
CBA would constitute the only agreement
between the parties.

II. In the present case, the foregoing


requirement has been sufficiently met.
Petitioner’s claim of grave abuse of
discretion is anchored on the simple fact
that public respondent adopted largely the
proposals of private respondent. It should
be understood that bargaining is not
equivalent to an adversarial litigation where
rights and obligations are delineated and
remedies applied. It is simply a process of
finding a reasonable solution to a conflict
and harmonizing opposite positions into a
fair and reasonable compromise. When
parties agree to submit unresolved

60
| Labor Relations-2A

57_Canape Petitioner filed a motion asking the


Secretary of Labor to assume jurisdiction
UNION OF FILIPRO EMPLOYEES vs. over the dispute of deadlock in collective
NLRC and NESTLE PHILIPPINES, INC. bargaining between the parties. On October
28, 1988, Labor Secretary Franklin Drilon
G.R. No. 91025 : December 19, 1990. “certified” to the NLRC the said dispute
between the UFE and Nestle, Philippines..
FACTS: On June 22, 1988, the petitioner which reads as follows: xxx “The NLRC is
Union of the Filipro Employees, the sole and further directed to call all the parties
exclusive bargaining agent of all rank-and- immediately and resolve the CBA deadlock
file employees of Nestle Philippines, (private within twenty (20) days from submission of
respondent) filed a Notice of Strike at the the case for resolution.” Second Division of
DOLE raising the issues of CBA deadlock the NLRC promulgated a resolution granting
and unfair labor practice. Private wage increase and other benefits to
respondent assailed the legal personality of Nestle’s employees, ruling on non-economic
the proponents of the said notice of strike to issues, as well as absolving the private
represent the Nestle employees, before the respondent of the Unfair Labor Practice
NCMB. This notwithstanding, the NCMB charge. Petitioner finds said resolution to be
proceeded to invite the parties to attend the inadequate and accordingly, does not agree
conciliation meetings and to which private therewith. It filed a motion for
respondent failed to attend contending that reconsideration, denied. Hence, this
it will deal only with a negotiating panel duly petition.
constituted and mandated in accordance
with the UFE Constitution and By-laws. ISSUE: (relative to the topic) WON
Thereafter, Company terminated from WHETHER OR NOT THE SECOND
employment all UFE Union officers, and all DIVISION OF THE NLRC ACTED
the members of the negotiating panel for WITHOUT JURISDICTION IN RENDERING
instigating and knowingly participating in a THE ASSAILED RESOLUTION, THE SAME
strike staged at the Makati, Alabang, BEING RENDERED ONLY BY A DIVISION
Cabuyao and Cagayan de Oro on OF THE PUBLIC RESPONDENT AND NOT
September 11, 1987 without any notice of BY EN BANC;
strike filed and a strike vote obtained for the
purpose. The union filed a complaint for HELD: This case was certified on October
illegal dismissal. LA upheld the validity of 28, 1988 when existing rules prescribed
the dismissal; NLRC en banc affirmed. that, it is incumbent upon the Commission
Subsequently, company concluded en banc to decide or resolve a certified
separate CBAs with the general dispute. However, R.A. 6715 took effect
membership of the union at Cebu/Davao during the pendency of this case. Aside
and Cagayan de Oro units; Assailing the from vesting upon each division the power
validity of these agreements, the union filed to adjudicate cases filed before the
a case of ULP against the company with the Commission, said Act further provides that
NLRC-NCR Arbitration Branch Efforts to the divisions of the Commission shall have
resolve the dispute amicably were taken by exclusive appellate jurisdiction over cases
the NCMB but yielded negative result. within their respective territorial jurisdiction.

61
| Labor Relations-2A

Section 5 of RA 6715 provides as follows: 58_Carillo


xxxx The Commission may sit en banc or in
five (5) divisions, each composed of three SAMAHANG MANGGAGAWA SA TOP
(3) members. The Commission shall sit en FORM MANUFACTURING-UNITED
banc only for purposes of promulgating WORKERS OF THE PHILIPPINES
rules and regulations governing the hearing (SMTFM-UWP) VS NLRC
and disposition of cases before any of its
divisions and regional branches and FACTS:
formulating policies affecting its
Petitioner Samahang Manggagawa sa Top
administration and operations. The
Form Manufacturing — United Workers of
Commission shall exercise its adjudicatory
the Philippines (SMTFM) was the certified
and all other powers, functions and duties
collective bargaining representative of all
through its divisions. xxxx In view of the
regular rank and file employees of private
enactment of Republic Act 6715, the
respondent Top Form Manufacturing
aforementioned rules requiring the
Philippines, Inc. At the collective bargaining
Commission en banc to decide or resolve a
negotiation held in Makati, the parties
certified dispute have accordingly been
agreed to discuss unresolved economic
repealed. Confirmed in Administrative Order
issues. According to the minutes of the
No. 36 (Series of 1989) promulgated by the
meeting, Article VII of the collective
Secretary under his delegated rule-making
bargaining agreement was discussed
power. Moreover, it is to be emphasized
wherein the Union proposed that any future
and it is a matter of judicial notice that since
wage increase given by the government
the effectivity of R.A. 6715, many cases
should be implemented by the company
have already been decided by the 5
across-the-board or non-conditional. The
divisions of the NLRC. We find no legal
justification in entertaining petitioner’s claim RTWPB-NCR later on issued Wage Order
No. 01 and 02 providing for salary
considering that the clear intent of the
increases.an across-the-board basis.
amendatory provision is to expedite the
Private respondent refused to accede to
disposition of labor cases filed before the
that demand. Instead, it implemented a
Commission. To rule otherwise would not
scheme of increases purportedly to avoid
be congruous to the proper administration of
wage distortion. The union, through its legal
justice. ACCORDINGLY, PREMISES
counsel, wrote private respondent a letter
CONSIDERED, the petition is DISMISSED.
demanding that it should "fulfill its pledge of
The Resolutions of the NLRC, dated June 5,
sincerity to the union by granting an across-
1989 and August 8, 1989 are AFFIRMED,
the-board wage increases (sic) to all
except insofar as the ruling absolving the
employees under the wage orders." The
private respondent of unfair labor practice
which is declared SET ASIDE union reiterated that it had agreed to "retain
the old provision of CBA" on the strength of
private respondent's "promise and
assurance" of an across-the-board salary
increase should the government mandate
salary increases.

ISSUE:

62
| Labor Relations-2A

WON Top Form's promise at the collective provisions should be "construed liberally
bargaining conferences to implement any rather than narrowly and technically, and
government-mandated wage increases on the courts must place a practical and
an across-the-board basis is binding upon realistic construction upon it, giving due
the parties considering it was not part of the consideration to the context in which it is
CBA though contained in the Minutes of the negotiated and purpose which it is intended
Meeting to serve." This is founded on the dictum that
a CBA is not an ordinary contract but one
impressed with public interest. It goes
HELD: without saying, however, that only
provisions embodied in the CBA should be
To start with, if there was indeed a promise so interpreted and complied with. Where a
or undertaking on the part of private proposal raised by a contracting party does
respondent to obligate itself to grant an not find print in the CBA, it is not a part
automatic across-the-board wage increase, thereof and the proponent has no claim
petitioner union should have requested or whatsoever to its implementation. Hence,
demanded that such "promise or petitioner union's contention that the
undertaking" be incorporated in the CBA. Minutes of the collective bargaining
After all, petitioner union has the means negotiation meeting forms part of the entire
under the law to compel private respondent agreement is pointless. The Minutes reflects
to incorporate this specific economic the proceedings and discussions
proposal in the CBA. It could have invoked undertaken in the process of bargaining for
Article 252 of the Labor Code defining "duty worker benefits in the same way that the
to bargain," thus, the duty includes minutes of court proceedings show what
"executing a contract incorporating such transpired therein. At the negotiations, it is
agreements if requested by either party." but natural for both management and labor
Petitioner union's assertion that it had to adopt positions or make demands and
insisted on the incorporation of the same offer proposals and counterproposals.
proposal may have a factual basis However, nothing is considered final until
considering the allegations in the the parties have reached an agreement. In
aforementioned joint affidavit of its fact, one of management's usual negotiation
members. However, Article 252 also states strategies is to ". . . agree tentatively as you
that the duty to bargain "does not compel go along with the understanding that
any party to agree to a proposal or make nothing is binding until the entire agreement
any concession." Thus, petitioner union may is reached." If indeed private respondent
not validly claim that the proposal embodied promised to continue with the practice of
in the Minutes of the negotiation forms part granting across the-board salary increases
of the CBA that it finally entered into with ordered by the government, such promise
private respondent. The CBA is the law could only be demandable in law if
between the contracting parties — the incorporated in the CBA. Moreover, by
collective bargaining representative and the making such promise, private respondent
employer company. Compliance with a CBA may not be considered in bad faith or at the
is mandated by the expressed policy to give very least, resorting to the scheme of
protection to labor. In the same vein, CBA feigning to undertake the negotiation

63
| Labor Relations-2A

proceedings through empty promises. As 59_Castro


earlier stated, petitioner union had, under
the law, the right and the opportunity to Standard Chartered Bank Employees Union
insist on the foreseeable fulfillment of the vs. Confesor, G. R. No. 114974, June 16,
private respondent's promise by demanding 2004
its incorporation in the CBA. Because the
proposal was never embodied in the CBA, FACTS
the promise has remained.
The petitioner asserts that the private
respondent committed ULP, i.e.,
interference in the selection of the Unions
negotiating panel, when Cielito Diokno, the
Banks Human Resource Manager,
suggested to the Unions President Eddie L.
Divinagracia that Jose P. Umali, Jr.,
President of the NUBE, be excluded from
the Unions negotiating panel. In support of
its claim, Divinagracia executed an affidavit,
stating that prior to the commencement of
the negotiation, Diokno approached him and
suggested the exclusion of Umali from the
Unions negotiating panel, and that during
the first meeting, Diokno stated that the
negotiation be kept a family affair.

ISSUE

1. whether or not the Union was able


to substantiate its claim of unfair labor
practice against the Bank arising from the
latters alleged interference with its choice of
negotiator;

2. whether or not the Union was able to


substantiate its claim of unfair labor practice
against the Bank arising from the latters
surface bargaining

HELD.

1. No. Article 248(a) of the Labor Code,


considers it an unfair labor practice when an
employer interferes, restrains or coerces
employees in the exercise of their right to

64
| Labor Relations-2A

self-organization or the right to form The records show that after the initiation of
association. The right to self-organization the collective bargaining process, with the
necessarily includes the right to collective inclusion of Umali in the Unions negotiating
bargaining. panel, the negotiations pushed through. The
complaint was made only on August 16,
Parenthetically, if an employer interferes in 1993 after a deadlock was declared by
the selection of its negotiators or coerces the Union on June 15, 1993.
the Union to exclude from its panel of
negotiators a representative of the Union, It is clear that such ULP charge was merely
and if it can be inferred that the employer an afterthought. The accusation occurred
adopted the said act to yield adverse effects after the arguments and differences over
on the free exercise to right to self- the economic provisions became heated
organization or on the right to collective and the parties had become frustrated. It
bargaining of the employees, ULP under happened after the parties started to involve
Article 248(a) in connection with Article 243 personalities. As the public respondent
of the Labor Code is committed. noted, passions may rise, and as a result,
suggestions given under less adversarial
In order to show that the employer situations may be colored with unintended
committed ULP under the Labor Code, meanings.[49] Such is what appears to have
substantial evidence is required to support happened in this case.
the claim. Substantial evidence has
been defined as such relevant evidence as If at all, the suggestion made by Diokno to
a reasonable mind might accept as Divinagracia should be construed as part of
adequate to support a conclusion.[48] In the the normal relations and innocent
case at bar, the Union bases its communications, which are all part of the
claim of interference on the alleged friendly relations between the Union and
suggestions of Diokno to exclude Umali Bank.
from the Unions negotiating panel.
2. The Union alleges that the Bank violated
The circumstances that occurred during the its duty to bargain; hence, committed ULP
negotiation do not show that the suggestion under Article 248(g) when it engaged in
made by Diokno to Divinagracia is an anti- surface bargaining. It alleged that the Bank
union conduct from which it can be inferred just went through the motions of bargaining
that the Bank consciously adopted such act without any intent of reaching an
to yield adverse effects on the free exercise agreement, as evident in the Banks counter-
of the right to self-organization and proposals. It explained that of the 34
collective bargaining of the employees, economic provisions it made, the Bank only
especially considering that such was made 6 economic
undertaken previous to the commencement counterproposals. Further, as borne by the
of the negotiation and simultaneously with minutes of the meetings, the Bank, after
Divinagracias suggestion that the bank indicating the economic provisions it had
lawyers be excluded from its negotiating rejected, accepted, retained or were open
panel. for discussion, refused to make a list of

65
| Labor Relations-2A

items it agreed to include in the economic 60_Dela Rosa


package.
G.R. No. 75321 June 20, 1988
Surface bargaining is defined as going
through the motions of negotiating without ASSOCIATED TRADE UNIONS
any legal intent to reach an (ATU), petitioner,
[50]
agreement. The resolution of surface vs.
bargaining allegations never presents an HON. CRESENCIO B. TRAJANO, in his
easy issue. The determination of whether a capacity as Director of the Bureau of
party has engaged in unlawful surface Labor Relations, MOLE, BALIWAG
bargaining is usually a difficult one because TRANSIT, INC. and TRADE UNIONS OF
it involves, at bottom, a question of the THE PHILIPPINES AND ALLIED
intent of the party in question, and usually SERVICES (TUPAS)-WFTU,respondents.
such intent can only be inferred from the
totality of the challenged partys conduct Facts: On March 25, 1986, the private
both at and away from the bargaining respondent union (TUPAS) filed with the
table.[51] It involves the question of whether Malolos labor office of the MOLE a petition
an employers conduct demonstrates an for certification election at the Baliwag
unwillingness to bargain in good faith or is Transit, Inc. among its rank-and-file
merely hard bargaining workers. Despite opposition from the herein
petitioner, Associated Trade Unions (ATU),
The Union has not been able to show that the petition was granted by the med-arbiter,
the Bank had done acts, both at and away and a certification election was ordered.
from the bargaining table, which tend to
show that it did not want to reach an ATU claims that the private respondent's
agreement with the Union or to settle the petition for certification election is defective
differences between it and because (1) at the time it was filed, it did not
the Union. Admittedly, the parties were not contain the signatures of 30% of the
able to agree and reached a workers, to signify their consent to the
deadlock. However, it is herein emphasized certification election; and (2) it was not
that the duty to bargain does not compel allowed under the contract-bar rule because
either party to agree to a proposal or require a new collective bargaining agreement had
the making of a concession.[53] Hence, the been entered into by ATU with the company
parties failure to agree did not amount to on April 1, 1986. ATU insists that its
ULP under Article 248(g) for violation of the collective bargaining agreement concluded
duty to bargain. by it with Baliwag Transit, Inc, on April 1,
1986, should bar the certification election
sought by TUPAS as this would disturb the
said new agreement. Moreover, the
agreement had been ratified on April 3,
1986, by a majority of the workers and is
plainly beneficial to them because of the
many generous concessions made by the
management.

66
| Labor Relations-2A

TUPAS for its part, states that the collective workers of the benefits of the said
bargaining agreement, besides being agreement, it shall be recognized and given
vitiated by certain procedural defects, was effect on a temporary basis, subject to the
concluded by ATU with the management results of the certification election. The
only on April 1, 1986 after the filing of the agreement may be continued in force if ATU
petition for certification election on March is certified as the exclusive bargaining
25, 1986. representative of the workers or may be
rejected and replaced in the event that
Issue: WON TUPAS was barred from filing TUPAS emerges as the winner.
a petition for certification election under the
contract-bar rule under Section 3, Rule 5,
Book V of the Implementing Rules and
Regulations. 61_De Leon

Ruling: No.

Section 3, Rule 5, Book V of the


Implementing Rules and Regulations simply
provides that a petition for certification
election or a motion for intervention can only
be entertained within sixty days prior to the
expiry date of an existing collective
bargaining agreement. Otherwise put, the
rule prohibits the filing of a petition for
certification election during the existence of
a collective bargaining agreement except
within the freedom period, as it is called,
when the said agreement is about to expire.
The purpose, obviously, is to ensure
stability in the relationships of the workers
and the management by preventing
frequent modifications of any collective
bargaining agreement earlier entered into
by them in good faith and for the stipulated
original period.

The said CBA was entered into at a time


when the petition for certification election
had already been filed by TUPAS and was
then pending resolution. The CBA cannot
be deemed permanent, precluding the
commencement of negotiations by another
union with the management. In the
meantime however, so as not to deprive the

67
| Labor Relations-2A

62_Del Rosario others are not entitled for the benefits of the
CBA for they were not yet employed much
New Pacific Timber & Supply Company less were members of the bargaining unit
vs NLRC during the term of the CBA.

GR No. 124224 March 17, 2000 Issue:

a. Whether or not Collective Bargaining


Agreement (CBA) as to its economic
Facts: National Federation of Labor (NFL), provisions be extended beyond the term
the sole and exclusive bargaining agent of expressly stipulated therein, and, in the
the rank- and-file employees of petitioner absence of a new CBA, even beyond the
Company, filed a complaint for unfair labor three-year period provided by law
practice (ULP) against the latter on the
b. whether or not employees hired after the
ground of refusal to bargain collectively.
NFL had received a favorable decision from stipulated term of a CBA entitled to the
benefits provided thereunder
the NLRC and SC (upon appeal); hence
petitioner Company was directed to pay the Held:
142 employees entitled to different
monetary benefits under the CBA. Petitioner a. Yes, the CBA will extend beyond
the term expressly stipulated
Company complied; and, the corresponding
therein in the absence of a new
quitclaims were executed. The case was CBA
considered closed following NFL's According to petitioner, the provision
manifestation that it will no longer appeal on wage increase in the 1981 to 1984 CBA
the October 18, 1993 Order of Labor Arbiter between petitioner Company and NFL
Villena. provided for yearly wage increases.
However, notwithstanding such Logically, these provisions ended in the
manifestation, a "Petition for Relief" was year 1984 - the last year that the economic
filed in behalf of 186 of the private provisions of the CBA were, pursuant to
respondents "Mariano J. Akilit and 350 contract and law, effective. Petitioner claims
others" on May 12, 1994. In their petition, that there is no contractual basis for the
they claimed that they were wrongfully grant of CBA benefits such as wage
excluded from enjoying the benefits under increases in 1985 and subsequent years,
the CBA since the agreement with NFL and since the CBA stipulates only the increases
petitioner Company limited the CBA's for the years 1981 to 1984.
implementation to only the 142 rank-and-file Anent the issue of whether or not the
employees enumerated. They claimed that term of an existing CBA, particularly as to its
NFL's misrepresentations had precluded economic provisions, can be extended
them from appealing their exclusion. beyond the period stipulated therein, and
Treating the petition for relief as an appeal, even beyond the three-year period
the NLRC entertained the same and issued prescribed by law, in the absence of a new
a Resolution ordering the Company to pay agreement, Article 253 of the Labor Code
these 350 other complainants. Hence this explicitly provides:
petition seeking to annul such Resolution on
the ground that Mariano Akilit and 350

68
| Labor Relations-2A

ART. 253. Duty to bargain ceased to have force and effect in the year
collectively when there exists a 1984, would be to create a gap during which
collective bargaining agreement. - no agreement would govern, from the time
When there is a collective bargaining the old contract expired to the time a new
agreement, the duty to bargain agreement shall have been entered into.
collectively shall also mean that
b. Yes, the employees hired after the
neither party shall terminate nor
stipulated term of a CBA are
modify such agreement during its entitled to the benefits provided
lifetime. However, either party can thereunder
serve a written notice to terminate or When a collective bargaining contract is
modify the agreement at least sixty entered into by the union representing the
(60) days prior to its expiration employees and the employer, even the non-
date. It shall be the duty of both member employees are entitled to the
parties to keep the status quo and to benefits of the contract. To accord its
continue in full force and effect the benefits only to members of the union
terms and conditions of the existing without any valid reason would constitute
agreement during the 60-day period undue discrimination against nonmembers.
and/or until a new agreement is It is even conceded, that a laborer can claim
reached by the parties. benefits from a CBA entered into between
It is clear from the above provision the company and the union of which he is a
of law that until a new Collective Bargaining member at the time of the conclusion of the
Agreement has been executed by and agreement, after he has resigned from said
between the parties, they are duty-bound to union. his is in consonance with our ruling
keep the status quo and to continue in full that the terms and conditions of a collective
force and effect the terms and conditions of bargaining agreement continue to have
the existing agreement. The law does not force and effect beyond the stipulated term
provide for any exception nor qualification when no new agreement is executed by and
as to which of the economic provisions of between the parties to avoid or prevent the
the existing agreement are to retain force situation where no collective bargaining
and effect; therefore, it must be understood agreement at all would govern between the
as encompassing all the terms and employer company and its employees.
conditions in the said agreement. Sccal r

In the case at bar, no new


agreement was entered into by and
between petitioner Company and NFL
pending appeal of the decision in NLRC
Case No. RAB-IX-0334-82; nor were any of
the economic provisions and/or terms and
conditions pertaining to monetary benefits in
the existing agreement modified or altered.
Therefore, the existing CBA in its entirety,
continues to have legal effect. To rule
otherwise, i.e., that the economic provisions
of the existing CBA in the instant case

69
| Labor Relations-2A

63_Diola The UFE assigns several errors of the NLRC’s


judgment. the power of the Secretary of Labor
UNION OF FILIPRO EMPLOYEES (UFE) under Art. 263(g) of the Labor Code to assume
vs. NESTLE PHILIPPINES INC. jurisdiction over a labor dispute tainted with
national interests, or to certify the same for
(G.R. Nos 88710-13, December 19, 1990) compulsory arbitration. UFE contends that Arts.
263 and 264 are based on the 1973
FACTS: Constitution, specifically Sec. 9 of Art. II thereof,
the pertinent portion of which reads:
Petitioners in this case are the various
"Sec. 9. . . . The State may provide for
employees of Nestle Philippines under their
compulsory arbitration." (p. 801, Rollo)
Union UFE. The first incident happened when
UFE filed a notice of strike with the Bureau of
UFE argues that since the aforecited provision
Labor Relations against Filipro (now NESTLE).
of Sec. 9 is no longer found in the 1987
UFE likewise filed a complaint for unfair labor
Constitution, Arts. 263(g) and 264 of the Labor
practice (ULP) against Nestle for violation of the
Code are now "unconstitutional and must be
Labor Code on Holiday Pay and non-
ignored.
implementation of the CBA, among others.
ISSUE: WON the strike was legal and WON
Through a petition, Nestle sought the
assumption of jurisdiction over the dispute of the
the aforecited provisions of the Labor Code
NLRC for compulsory arbitration. Then Minister are applicable.
of Labor Blas Ople assumed jurisdiction over the
dispute and strictly enjoined concerted actions of RULING: NO, the strike is NOT legal and
strikes, lockouts, noise barrages and other acts YES, the aforecited provisions of the Labor
which tend to disrupt company operations during code are STILL APPLICABLE.
office hours.
Article 7 of the New Civil Code declares
Notwithstanding the said resolution against any that:
concerted activity, and an absence of a
restraining order, the UFE still proceeded with a Laws are repealed only by
strike. The UFE officers likewise distributed subsequent ones, and their violation
leaflets to employees of Nestle and passers-by or non-observance shall not be
advocating a boycott of company products.
excused by disuse or custom or
Nestle filed a petition to declare the strike illegal
practice to the contrary.
premised on the violation of the CBA provisions
on “no strike/no lockout” clause and the
In the case at bar, no law has ever been
grievance machinery provisions on settlement of passed by Congress expressly repealing
disputes. Articles 263 and 264 of the Labor Code.
Neither may the 1987 Constitution be
Despite subsequent orders of Labor Minister considered to have impliedly repealed the
Ople and the then new Labor Minister Sanchez, said Articles considering that there is no
the UFE refused to stop the strikes. A series of showing that said articles are inconsistent
petitions and strikes later ensued.
with the said Constitution. Moreover, no
After trial on the merits, the NLRC issued a
court has ever declared that the said articles
decision declaring the strikes of UFE illegal and are inconsistent with the 1987 Constitution.
that the UFE is guilty of unfair labor practice.

70
| Labor Relations-2A

Articles 263 (g) and 264 of the Labor Code maintain the status quo while the
have been enacted pursuant to the police determination is being made.
power of the State to promote the order,
safety, health, morals and general welfare The return to work order does not so much
of society confer a right as it imposes a duty; and
while as a right it may be waived, it must be
On the issue of the legality of the strike discharged as a duty even against the
committed, UFE seeks to absolve itself by worker's will. Returning to work in this
pointing out qualifying factors such as situation is not a matter of option or
motives, good faith, absence of findings on voluntariness but of obligation. The worker
specific participation and/or liability, and must return to his job together with his co-
limiting the no-strike provision to economic workers so the operations of the company
strikes. can be resumed and it can continue serving
the public and promoting its interest.":
UFE completely misses the underlying
principle embodied in Art. 264(g) on the We also wish to point out that an
settlement of labor disputes and this is, that assumption and/or certification order of the
assumption and certification orders are Secretary of Labor automatically results in a
executory in character and are to be strictly return-to-work of all striking workers,
complied with by the parties even during the whether or not a corresponding order has
pendency of any petition questioning their been issued by the Secretary of Labor.
validity. This extraordinary authority given to Thus, the striking workers erred when they
the Secretary of Labor is aimed at arriving continued with their strike alleging absence
at a peaceful and speedy solution to labor of a return-to-work order. Article 264(g) is
disputes, without jeopardizing national clear. Once an assumption/certification
interests. order is issued, strikes are enjoined, or if
one has already taken place, all strikers
Regardless therefore of their motives, or the shall immediately return to work.
validity of their claims, the striking workers
must cease and/or desist from any and all A strike that is undertaken despite the
acts that tend to, or undermine this authority issuance by the Secretary of Labor of an
of the Secretary of Labor, once an assumption or certification order becomes a
assumption and/or certification order is prohibited activity and thus illegal.
issued. They cannot, for instance, ignore
return-to-work orders, citing unfair labor
practices on the part of the company, to
justify their actions.

One other point that must be underscored is


that the return-to-work order is issued
pending the determination of the legality or
illegality of the strike. It is not correct to say
that it may be enforced only if the strike is
legal and may be disregarded if the strike is
illegal, for the purpose precisely is to

71
| Labor Relations-2A

64_Dizon (February and August 2000) agency and does not automatically result
from a mere increase in the wages of
MANILA ELECTRIC COMPANY VS. SEC. petitioner's employees. Collective
OF LABOR LEONARDO QUISUMBING bargaining disputes particularly those
and MERALCO EMPLOYEES AND affecting the national interest and public
WORKERS ASSOC. service "requires due consideration and
proper balancing of the interests of the
G.R. No. 127598 February 22, 2000 parties to the dispute and of those who
FACTS: might be affected by the dispute. It should
be noted that the relations between labor
In 1999, the Supreme Court promulgated a and capital is impressed with public interest
decision directing the parties to execute a which must yield to the common
CBA which provided for increase in wages good. Neither party should act oppressively
and retroactive application of arbitral against the other or impair the interest or
awards. MERALCO filed this petition convenience of the public. Besides, matters
arguing that an increase in wages will result of salary increases are part of management
in higher rates of electricity which will be prerogative.
passed to the consumers. The Union
likewise asks for reconsideration insofar as
the 1999 decision which denied them the 2. ON THE RETROACTIVITY OF
benefit of being granted loans to set up a ARBITRAL AWARDS
cooperative. Finally, the Union questions
the right given to MERALCO in contracting Labor laws are silent as to when an arbitral
out jobs without need to consult the Union. award in a labor dispute where the
Secretary had assumed jurisdiction by virtue
ISSUES: of Article 263 (g) of the Labor Code shall
1. W/N the increase in wages will result in retroact. In general, a CBA negotiated within
higher prices of electricity. six months after the expiration of the
existing CBA retroacts to the day
2. W/N the grant of Collective Bargaining immediately following such date and if
Agreement (CBA) arbitral awards agreed thereafter, the effectivity depends on
retroactive the agreement of the parties. On the other
hand, the law is silent as to the retroactivity
3. W/N the cooperative may demand for the of a CBA arbitral award or that granted not
loan for a cooperative by virtue of the mutual agreement of the
parties but by intervention of the
4. W/N contracting without need to consult
government. Despite the silence of the law,
the Union is a valid provision
the Court rules herein that CBA arbitral
RULING: awards granted after six months from the
expiration of the last CBA shall retroact to
1. ON THE INCREASE IN WAGES such time agreed upon by both employer
and the employees or their union. Absent
No, it doesn’t follow. An increase in the
such an agreement as to retroactivity, the
prices of electric current needs the approval
award shall retroact to the first day after the
of the appropriate regulatory government

72
| Labor Relations-2A

six-month period following the expiration of management prerogative subject only to


the last day of the CBA should there be one. special laws and agreements on the matter
In the absence of a CBA, the Secretary's and the fair standards of justice.24
determination of the date of retroactivity as
part of his discretionary powers over arbitral
awards shall control. The management cannot be denied the
faculty of promoting efficiency and attaining
economy by a study of what units are
3. ON COOPERATIVE’S DEMAND FOR essential for its operation. It has the ultimate
LOAN determination of whether services should be
performed by its personnel or contracted to
On the allegation concerning the grant of outside agencies. While there should be
loan to a cooperative, there is no merit in mutual consultation, eventually deference is
the union's claim that it is no different from to be paid to what management
housing loans granted by the employer. The decides.25 Contracting out of services is an
award of loans for housing is justified exercise of business judgment or
because it pertains to a basic necessity of 26
management prerogative. Absent proof
life. It is part of a privilege recognized by the that management acted in a malicious or
employer and allowed by law. In contrast, arbitrary manner, the Court will not interfere
providing seed money for the establishment with the exercise of judgment by an
of the employee's cooperative is a matter in employer.
which the employer has no business
interest or legal obligation. Courts should
not be utilized as a tool to compel any
person to grant loans to another nor to force
parties to undertake an obligation without
justification.

4. ON CONTRACTING WITHOUT
CONSULTING THE UNION

The employer is allowed to contract out


services for six months or more. However, a
line must be drawn between management
prerogatives regarding business
operations per se and those which affect the
rights of employees, and in treating the
latter, the employer should see to it that its
employees are at least properly informed of
its decision or modes of action in order to
attain a harmonious labor-management
relationship and enlighten the workers
concerning their rights. Hiring of workers is
within the employer's inherent freedom to
regulate and is a valid exercise of its

73
| Labor Relations-2A

Second, petitioner contends that


this Court erred in holding that the
MANILA ELECTRIC CO. VS. HON. SEC. effectivity of CBA provisions are
OF LABOR LEONARDO QUISUMBING automatically retroactive. Petitioner
AND MERALCO EMPLOYEES AND invokes, rather, this Court’s ruling in
WORKERS ASSOCIATION (MEWA) the Decision dated January 27,
G.R. No. 127598 August 1, 2000 1999, which was modified in the
assailed Resolution, that in the
FACTS: absence of an agreement between
the parties, an arbitrated CBA takes
Petitioner Manila Electric Company filed on the nature of any judicial or
with this Court, a "Motion for Partial quasi-judicial award; it operates and
Modification (Re: Resolution Dated 22 may be executed only prospectively
February 2000)" anchored on the following unless there are legal justifications
grounds: for its retroactive application.

I. Honorable Court’s ruling on the Third, petitioner contends that the


retroactivity issue: (a) fails to Resolution is internally flawed
account for previous rulings of the because when it held that the award
Court on the same issue; (b) fails to shall retroact to the first day after the
indicate the reasons for reversing six-month period following the
the original ruling in this case on the expiration of the last day of the CBA,
retroactivity issue; and (c) is the reckoning date should have
internally inconsistent. been June 1, 1996, not December 1,
1995, which is the last day of the
II. Honorable Court’s ruling on the three-year lifetime of the economic
retroactivity issue does not take into provisions of the CBA.
account the huge cost that this
award imposes on petitioner, The petitioner also prays that the two-year
estimated at no less than P800 term of the CBA be fixed from December
Million. 28, 1996 to December 27, 1998 since the
application of the arbitral award will cost it
Petitioner specifically assails the Resolution no less than P800 Million. Petitioner also
as being logically flawed, arguing, seeks this Court’s declaration that the
award of P2,000.00 be paid to petitioner’s
First, that while it alludes to the rank-and-file employees during this two-
Secretary’s discretionary powers year period. In the alternative, petitioner
only in the absence of a CBA, Article prays that the award of P2,000.00 be made
253-A of the Labor Code always to retroact to June 1, 1996 as the effectivity
presupposes the existence of a prior date of the CBA.
or subsisting CBA; hence the
exercise by the Secretary of his ISSUE:
discretionary powers will never come
to pass.

74
| Labor Relations-2A

W/N Art. 253-A of the Labor Code retroactivity thereof. In case of a


applies in this case. deadlock in the renegotiation of the
collective bargaining agreement, the
W/N collective bargaining agreement parties may exercise their rights
shall take effect only upon its signing and under this Code.
shall remain in full force and effect for a
period of five years. Under the circumstances of the case, Article
253-A cannot be properly applied to herein
RULING: case. As correctly stated by public
respondent in his assailed Order of April 12,
1. No. Article 253-A is hereunder 1991 dismissing petitioner’s Motion for
reproduced for ready reference: Reconsideration –

ART. 253-A. Terms of a collective Anent the alleged lack of basis for the
bargaining agreement. --- Any retroactivity provisions awarded, we would
Collective Bargaining Agreement stress that the provision of law invoked by
that the parties may enter into shall, the Hospital, Article 253-A of the Labor
insofar as the representation aspect Code, speaks of agreements by and
is concerned, be for a term of five (5) between the parties, and not arbitral awards
years. No petition questioning the . . . (p. 818 Rollo).
majority status of the incumbent
bargaining agent shall be Therefore, in the absence of a specific
entertained and no certification provision of law prohibiting retroactivity of
election shall be conducted by the the effectivity of arbitral awards issued by
Department of Labor and the Secretary of Labor pursuant to Article
Employment outside of the sixty-day 263(g) of the Labor Code, such as herein
period immediately before the date involved, public respondent is deemed
of expiry of such five year term of vested with plenary and discretionary
the Collective Bargaining powers to determine the effectivity
Agreement. All other provisions of thereof (223 SCRA 779, 792-793 [1993];
the Collective Bargaining Agreement reiterated in Philippine Airlines, Inc. v.
shall be renegotiated not later than Confessor 231 SCRA 41 [1994]).
three (3) years after its
execution. Any agreement on such Indeed, petitioner has not shown that the
other provisions of the Collective question of effectivity was not included in
Bargaining Agreement entered into the general agreement of the parties to
within six (6) months from the date submit their dispute for arbitration. To the
of expiry of the term of such other contrary, as the order of the labor arbiter
provisions as fixed in such Collective states, this question was among those
Bargaining Agreement, shall retroact submitted for arbitration by the parties:
to the day immediately following
such date. If any such agreement is
entered into beyond six months, the
parties shall agree on the duration of

75
| Labor Relations-2A

2.As regards the "Effectivity and Duration" day of the CBA. In the dispositive portion,
clause, the company proposes that the however, the period to which the award
collective bargaining agreement shall take shall retroact was inadvertently stated as
effect only upon its signing and shall remain beginning on December 1, 1995 up to
in full force and effect for a period of five November 30, 1997.
years. The union proposes that the
agreement shall take effect retroactive to In resolving the motions for reconsideration
March 15, 1989, the expiration date of the in this case, this Court took into account the
old CBA. fact that petitioner belongs to an industry
imbued with public interest. As such, this
And after an evaluation of the parties’ Court cannot ignore the enormous cost that
respective contention and argument thereof, petitioner will have to bear as a
it is believed that that of the union is fair and consequence of the full retroaction of the
reasonable. It is the observation of this arbitral award to the date of expiry of the
Arbitrator that in almost subsequent CBAs, CBA, and the inevitable effect that it would
the effectivity of the renegotiated CBA, have on the national economy. On the other
usually and most often is made effective hand, under the policy of social justice, the
retroactive to the date when the immediately law bends over backward to accommodate
preceding CBA expires so as to give a the interests of the working class on the
semblance of continuity. Hence, for this humane justification that those with less
particular case, it is believed that there is privilege in life should have more in law.
nothing wrong adopting the stand of the Balancing these two contrasting interests,
union, that is that this CBA be made this Court turned to the dictates of fairness
retroactive effective March 15, 1989. and equitable justice and thus arrived at a
formula that would address the concerns of
Parenthetically, the Decision rendered in the both sides. Hence, this Court held that the
case at bar on January 27, 1999 ordered arbitral award in this case be made to
that the CBA should be effective for a term retroact to the first day after the six-month
of two years counted from December 28, period following the expiration of the last
1996 (the date of the Secretary of Labor’s day of the CBA, i.e., from June 1, 1996 to
disputed Order on the parties’ motion for May 31, 1998.
reconsideration) up to December 27,
1998. That is to say, the arbitral award was This Court, therefore, maintains the
given prospective effect. foregoing rule in the assailed Resolution pro
hac vice. It must be clarified, however, that
Upon a reconsideration of the Decision, this consonant with this rule, the two-year
Court issued the assailed Resolution which effectivity period must start from June 1,
ruled that where an arbitral award granted 1996 up to May 31, 1998, not December 1,
beyond six months after the expiration of 1995 to November 30, 1997.1âwphi1
the existing CBA, and there is no agreement
between the parties as to the date of
effectivity thereof, the arbitral award shall
retroact to the first day after the six-month
period following the expiration of the last

76
| Labor Relations-2A

65_Dona SMC management informed its


employees that the company which was
SAN MIGUEL CORP. EMPLOYEES composed of four operating divisions would
UNION- PTGWO, et al. v. SECRETARY undergo a restructuring for its business
OF LABOR HON. CONFESOR, SAN expansion. Two divisions, Magnolia and
MIGUEL CORP., MAGNOLIA CORP. and Feeds and Livestock Division were spun-off
SAN MIGUEL FOODS, INC. and became two separate and distinct
G.R. No. 111262 September 19, 1996 corporations: Magnolia Corporation
(Magnolia) and San Miguel Foods, Inc.
FACTS: (SMFI). Notwithstanding the spin-offs, the
CBA remained in force and effect.
Petitioner-union entered into CBA
with private respondent San Miguel After June 30, 1992, the CBA was
Corporation (SMC) to take effect upon the renegotiated in accordance with the terms
expiration of the previous CBA or on June of the CBA and Article 253-A of the Labor
30, 1989. This CBA provided, among Code.
others, that:
PETITIONER-UNION’S CLAIM
ARTICLE XIV
The bargaining unit of SMC
DURATION OF AGREEMENT should still include the employees of the
spun-off corporations; and that the
SECTION 1. This Agreement which shall be renegotiated terms of the CBA shall be
binding upon the parties hereto and their effective only for the remaining period of two
respective successors-in-interest, shall years or until June 30, 1994. Petitioner-
become effective and shall remain in force union contends that the duration for the
and effect until June 30, 1992. non-representation provisions of the CBA
should be coterminous with the term of the
SEC. 2. In accordance with Article 253-A of
bargaining agency which in effect shall be
the Labor Code as amended, the term of
for the remaining two years of the current
this Agreement insofar as the
CBA.
representation aspect is concerned, shall be
for five (5) years from July 1, 1989 to June RESPONDENT’S CONTENTION
30, 1994. Hence, the freedom period for
purposes of such representation shall be SMC contended that the
sixty (60) days prior to June 30, 1994. members/employees who had moved to
Magnolia and SMFI, automatically ceased
SEC. 3. Sixty (60) days prior to June 30, to be part of the bargaining unit at the SMC.
1992 either party may initiate negotiations of Furthermore, the CBA should be effective
all provisions of this Agreement, except for three years in accordance with Art. 253-
insofar as the representation aspect is A of the Labor Code.
concerned. If no agreement is reached in
such negotiations, this Agreement shall Petitioner-union declared a deadlock
nevertheless remain in force up to the time and consequently filed a Notice of Strike
a subsequent agreement is reached by the against SMC. No settlement was arrived at
parties. despite several meetings held between the

77
| Labor Relations-2A

parties through NCMB. As prayed for by Bargaining Agreement that the parties may
private respondents, the Secretary of Labor enter into shall, insofar as the
assumed jurisdiction over the labor dispute representation aspect is concerned, be for a
on November 10, 1992. Several conciliation term of five (5) years … All other provisions
meetings were held but still no of the Collective Bargaining Agreement
agreement/settlement was arrived at by shall be renegotiated not later than three (3)
both parties. years after its execution.

The Secretary of Labor resolved the Article 253-A is a new


labor dispute and issued the assailed Order provision. This was incorporated by Section
directing, among others, that the 21 of Republic Act No. 6715 (the Herrera-
renegotiated terms of the CBA shall be Veloso Law) which took effect on March 21,
effective for the period of three (3) years 1989. This new provision states that the
from June 30, 1992; and that such CBA CBA has a term of five (5) years instead of
shall cover only the employees of SMC three years, before the amendment of the
and not of Magnolia and SMFI. law as far as the representation aspect is
concerned. The representation aspect
ISSUE/S: refers to the identity and majority status of
1) Whether or not the duration of the the union that negotiated the CBA as the
exclusive bargaining representative of the
renegotiated terms of the CBA is to be
effective for three years or for only two appropriate bargaining unit concerned. All
years; and other provisions simply refers to the rest of
the CBA, economic as well as non-
2) Whether or not the bargaining unit of economic provisions, except
SMC includes also the employees of representation.The law is clear and definite
Magnolia and SMFI. on the duration of the CBA insofar as the
representation aspect is concerned, but is
RULING: quite ambiguous with the terms of the other
provisions of the CBA.
The petition is DISMISSED for lack of
merit. There is no grave abuse of discretion If we look into the deliberations
on the part of the Secretary of Labor in leading to the passage of the law, the
rendering the assailed Order. legislators were more inclined to have
the period of effectivity for three (3)
HELD:
years insofar as the economic as well as
1. The Court agrees with the Secretary of non-economic provisions are concerned,
Labor that the renegotiated terms of the except representation. The framers of the
CBA at SMC should run for a period of three law wanted to maintain industrial peace and
(3) years. stability by having both management and
labor work harmoniously together without
Art. 253-A of the Labor Code as any disturbance. Thus, no outside union can
amended reads: enter the establishment within five (5) years
and challenge the status of the incumbent
ART. 253-A. Terms of a Collective
union as the exclusive bargaining
Bargaining Agreement. Any Collective
agent. Likewise, the terms and conditions of

78
| Labor Relations-2A

employment (economic and non-economic) incorporating a term of 3-years on the


cannot be questioned by the employers or renegotiated provisions.
employees during the period of effectivity of
the CBA. The CBA is a contract between The issue as to the term of the non-
the parties and the parties must respect the representation provisions of the CBA must
terms and conditions of the agreement. be resolved in the Memorandum of the
Notably, the framers of the law did not give Secretary of Labor dated February 24, 1994
a fixed term as to the effectivity of the terms which clarified that the parties are
and conditions of employment. It was left to encouraged to enter into a renegotiated
the parties to fix the period. In the instant CBA with a term which would coincide with
case, it is not difficult to determine the the aforesaid five (5) year term of the
period of effectivity for the non- bargaining representative. In the event
representation provisions of the however, that the parties, by mutual
CBA. Taking it from the history of their agreement, enter into a renegotiated
CBAs, SMC intended to have the terms of contract with a term of three (3) years or
the CBA effective for three (3) years one which does not coincide with the said 5-
reckoned from the expiration of the old year term, and said agreement is ratified by
or previous CBA which was on June 30, majority of the members in the bargaining
1989. unit, the subject contract is valid and legal
and therefore, binds the contracting
The company voluntary recognizes parties. The same will however not
the continuing representative status of the adversely affect the right of another union to
unions after the aforementioned spin-offs challenge the majority status of the
and the stand of the company for a 3-year incumbent bargaining agent within sixty (60)
renegotiated cycle when the economic days before the lapse of the original five (5)
provisions of the existing CBAs year term of the CBA.
expired. Precisely, the company conceded
to face the same union notwithstanding the 2. The bargaining unit of SMC DOES NOT
spin-offs in order to preserve industrial include employees of Magnolia and SMFI.
peace during the infancy of the two As a result of the spin-offs,
corporations. If the union would insist on a Magnolia and SMFI became distinct
shorter (2-year) renegotiated term, then all entities with separate juridical
the advantages gained by both parties in personalities. Thus, they cannot belong to
this regard would become futile. In order to a single bargaining unit. In determining an
effect a smooth transition, the companies appropriate bargaining unit, the test of
concerned continued to recognize the grouping is mutuality or commonality of
existing unions as the bargaining agents of interests affecting a grouping of employees
their respective bargaining units. In the
who have substantial, mutual interests in
meantime, the other unions in these wages, hours, working conditions and other
companies eventually concluded their CBA subjects of collective bargaining.
negotiations on the remaining term and all Considering the spin-offs, the companies
of them agreed on a 3-year cycle. Notably, would consequently have their respective
the following CBAs were forged and distinctive concerns in terms of the

79
| Labor Relations-2A

nature of work, wages, hours of work and 66_Duquila


other conditions of employment.
RIVERA, ET AL. vs. ESPIRITU, ET AL.
SMC is engaged in the business of
beer manufacturing. Magnolia is involved in G.R. No. 135547 January 23, 2002
the manufacturing and processing of dairy FACTS: Faced with bankruptcy, PAL
products while SMFI is involved in the adopted a rehabilitation plan and downsized
production of feeds and the processing of its labor force by more than one-third,
chicken. The nature of their products and prompting PALEA to go on strike to protest
scales of business may require different the retrenchment measures adopted by the
skills which must necessarily be airline. A Task Force composed of herein
commensurated by different compensation public respondents was created to hold
packages. The different companies may conciliation meetings between PAL
have different volumes of work and different management and the unions representing
working conditions. Hence, the employees
the airline employees, among them PALEA.
of the different companies see the need
PAL eventually ceased its operations and
to group themselves together and sent notices of termination to its employees.
organize themselves into distinctive and
However, PALEA offered a 10-year
different groups. It would then be best to
moratorium on strikes and similar actions
have separate bargaining units for the
and a waiver of some of the economic
different companies where the benefits in the existing CBA. PAL
employees can bargain separately management accepted the PALEA proposal
according to their needs and according and the necessary referendum was
to their own working conditions.
scheduled. Of the votes cast, 61% were in
favor of accepting the PAL-PALEA
agreement, while 34% rejected it. On the
same date PAL resumed domestic
operations, seven officers and members of
PALEA filed this instant petition to annul the
agreement entered into between PAL and
PALEA on the ground that public
respondents gravely abused their discretion
and exceeded their jurisdiction in actively
pursuing the conclusion of the PAL-PALEA
agreement as the constitutional rights to
self-organization and collective bargaining,
being founded on public policy, may not be
waived, nor the waiver, ratified.

ISSUE: Whether or not the agreement


stipulating the suspension of the PAL-
PALEA CBA is unconstitutional and contrary
to public policy.

HELD: NO.

80
| Labor Relations-2A

There is no conflict between said abeyance the limit on the representation


agreement and Article 253-A of the Labor period.
Code. Article 253-A has a two-fold purpose.
One is to promote industrial stability and
predictability. Inasmuch as the agreement PALEA PROPOSAL (Terms and
sought to promote industrial peace at PAL Conditions)
during its rehabilitation, said agreement
satisfies the first purpose of Article 253-A. 1. Each PAL employee shall be granted
The other is to assign specific timetables 60,000 shares of stock with a par value of
wherein negotiations become a matter of P5.00, from Mr. Lucio Tan’s shareholdings,
right and requirement. Nothing in Article with three (3) seats in the PAL Board and
253-A, prohibits the parties from waiving or an additional seat from government shares
suspending the mandatory timetables and as indicated by His Excellency;
agreeing on the remedies to enforce the
2. Likewise, PALEA shall, as far as
same.
practicable, be granted adequate
In the instant case, it was PALEA, as representation in committees or bodies
the exclusive bargaining agent of PAL’s which deal with matters affecting terms and
ground employees, that voluntarily entered conditions of employment;
into the CBA with PAL. It was also PALEA
that voluntarily opted for the 10-year 3. To enhance and strengthen labor-
suspension of the CBA. Either case was the management relations, the existing Labor-
union’s exercise of its right to collective Management Coordinating Council shall be
bargaining. The right to free collective reorganized and revitalized, with adequate
bargaining includes the right to suspend it. representation from both PAL management
and PALEA;
Further, the acts of public
respondents in sanctioning the 10-year 4. To assure investors and creditors of
suspension of the PAL-PALEA CBA did not industrial peace, PALEA agrees, subject
contravene the "protection to labor" policy of to the ratification by the general
the Constitution. The agreement afforded membership, (to) the suspension of the
full protection to labor; promoted the shared PAL-PALEA CBA for a period of ten (10)
responsibility between workers and years, provided the following safeguards
employers; and exercised the voluntary are in place:
modes in settling disputes, including
a. PAL shall continue recognizing
conciliation to foster industrial peace.
PALEA as the duly certified
Moreover, the agreement does not bargaining agent of the regular rank-
violate the five-year representation limit and-file ground employees of the
mandate by Article 253-A. Under said Company;
article, the representation limit for the
exclusive bargaining agent applies only b. The ‘union shop/maintenance of
when there is an extant CBA in full force membership’ provision under the
and effect. In the instant case, the parties PAL-PALEA CBA shall be
agreed to suspend the CBA and put in respected.

81
| Labor Relations-2A

c. No salary deduction, with full 67_Eisma


medical benefits.

5. PAL shall grant the benefits under the 26


July 1998 Memorandum of Agreement PHILIPPINE AIRLINES, INC. vs. NLRC,
PAL EMPLOYEES ASSOCIATION
forged by and between PAL and PALEA, to
(PALEA) et al
those employees who may opt to retire or G.R. No. 85985 August 13, 1993
be separated from the company.
Management prerogative is not boundless.
6. PALEA members who have been Line must be drawn between policies which
retrenched but have not received separation are purely business-oriented and those
benefits shall be granted priority in the which affect rights of employees.
hiring/rehiring of employees. Employee’s right to participate in
7. In the absence of applicable Company policymaking is vital in promoting industrial
rule or regulation, the provisions of the peace.
Labor Code shall apply.

FACTS:

PAL completely revised its 1966 Code of


Discipline. The Code was circulated among
the employees and was immediately
implemented, and some employees were
instantly subjected to the disciplinary
measures. Respondent PALEA filed a
complaint before the NLRC contending that
PAL is guilty of unfair labor practice due to
its unilateral implementation of the Code,
specifically violating par. E and G of Article
249 and Article 253 of the Labor Code.
PALEA alleged that copies of the Code
had been circulated in limited numbers; that
being penal in nature the Code
must conform with the requirements of
sufficient publication, and that the Code was
arbitrary, oppressive, and prejudicial to the
rights of the employees.

Respondent prayed that implementation of


the Code be held in abeyance; that PAL
should first discuss the substance of the
Code; that employees dismissed under the
Code be reinstated and their cases

82
| Labor Relations-2A

subjected to further hearing; and that PAL adopted cannot be properly implemented in
should pay damages. the absence of full cooperation of the
employees. Such cooperation cannot be
PAL asserts its management prerogative attained if the employees are left out in the
and further alleged that it had not violated determination of cardinal and fundamental
the CBA or any provision of the Labor Code. matters affecting their employment.
It maintained that Article 253 of the Labor
Code cited by PALEA was inapplicable
since the current CBA had been negotiated.

ISSUE: Whether management may be


compelled to share with the union or its
employees its prerogative of formulating a
code of discipline.

HELD: Yes.

PAL asserts that when it revised its Code on


March 15, 1985, there was no law which
mandated the sharing of responsibility
therefor between employer and employee. It
was only on March 2, 1989, with the
approval of Republic Act No. 6715,
amending Article 211 of the Labor Code,
that the law explicitly considered it a State
policy "to ensure the participation of workers
in decision and policy-making processes
affecting the rights, duties and welfare."
However, even in the absence of said clear
provision of law, the exercise of managerial
prerogatives is not unlimited. It is
circumscribed by limitations found in law, a
collective bargaining agreement, or the
general principles of fair play and justice.

The objectionable provisions of the Code


reveal that they are not purely business-
oriented nor do they concern the
management aspect of the business.
Management should see to it that its
employees are at least properly informed of
its decisions. A provision in the CBA may
not be interpreted as cession of employees'
rights to participate in the deliberation of
matters which may affect their rights and the
formulation of policies such as the
formulation of a code of discipline.
Whatever disciplinary measures are

83
| Labor Relations-2A

68_Fabon Indophil, on the other hand, submits that it is


a juridical entity separate and distinct from
Rights of legitimate labor organizations Acrylic. It cited jurisprudence which ruled
that two corporations cannot be treated as a
INDOPHIL TEXTILE MILL WORKERS single bargaining unit even if their
UNION-PTGWO vs. VOLUNTARY businesses are related. It submits that the
ARBITRATOR TEODORICO P. CALICA fact that there are as many bargaining units
and INDOPHIL TEXTILE MILLS, INC. as there are companies in a conglomeration
of companies is a positive proof that a
G.R. No. 96490 February 3, corporation is endowed with a legal
1992 personality distinctly its own, independent
and separate from other corporations.
FACTS:
Petitioner notes that the ff. evidence
Petitioner Indophil Textile Mill Workers establish that Acrylic is an extension or
Union-PTGWO (petitioner Union) is a expansion of Indophil:
legitimate labor organization and the
exclusive bargaining agent of all the rank- (a) the two corporations have their physical
and-file employees of Indophil Textile Mills, plants, offices and facilities situated in the
Inc (Indophil). Private respondent Indophil is same compound at Bulacan; (b) many of
a corporation engaged in the manufacture, Indophil's own machineries were transferred
sale and export of yarns and of materials of to and are now installed and being used in
similar character. the Acrylic plant; and (c) the employees of
Indophil are the same persons manning and
Petitioner Union and Indophil executed a servicing the units of Acrylic.
CBA.
ISSUE:
Indophil Acrylic Manufacturing Corporation
(Acrylic) was formed and registered with the W/N Acrylic is an extension or expansion of
SEC. It became operational and it hired private respondent. (NO)
workers according to its own criteria and
standards. The workers of Acrylic unionized W/N the rank-and-file employees working at
and a CBA was executed. Acrylic should be recognized as part of,
and/or within the scope of the bargaining
Petitioner Union contends that the plant unit. (NO)
facilities built and set up by Acrylic should
be considered as an extension or expansion HELD:
of the facilities of Indophil.
Under the doctrine of piercing the veil of
In other words, it is petitioner Union's corporate entity, when valid grounds
contention that Acrylic is part of the Indophil therefore exist, the legal fiction that a
bargaining unit, and that the creation of corporation is an entity with a juridical
Acrylic is a device of Indophil to evade the personality separate and distinct from its
application of the CBA (between petitioner members or stockholders may be
Union and Indophil) to Acrylic.

84
| Labor Relations-2A

disregarded. In such cases, the corporation Jurisprudence also dictates that it is grave
will be considered as a mere association of abuse of discretion to treat two companies
persons. The members or stockholders of as a single bargaining unit when these
the corporation will be considered as the companies are indubitably distinct entities
corporation, that is liability will attach directly with separate juridical personalities.
to the officers and stockholders. The
doctrine applies when the corporate fiction Acrylic not being an extension or expansion
is used to defeat public convenience, justify of Indophil, the rank-and-file employees
wrong, protect fraud, or defend crime, or working at Acrylic should not be recognized
when it is made as a shield to confuse the as part of, and/or within the scope of the
legitimate issues, or where a corporation is petitioner, as the bargaining representative
the mere alter ego or business conduit of a of Indophil.
person, or where the corporation is so
organized and controlled and its affairs are
so conducted as to make it merely an
instrumentality, agency, conduit or adjunct
of another corporation.

In the case at bar, petitioner seeks to pierce


the veil of corporate entity of Acrylic,
alleging that the creation of the corporation
is a devise to evade the application of the
CBA between petitioner Union and Indophil.

SC did not apply the doctrine invoked by


petitioner. The fact that the businesses of
Indophil and Acrylic are related, that some
of the employees of Indophil are the same
persons manning and providing for auxilliary
services to the units of Acrylic, and that the
physical plants, offices and facilities are
situated in the same compound, these facts
are not sufficient to justify the piercing of the
corporate veil of Acrylic.

Jurisprudence dictates that "the legal


corporate entity is disregarded only if it is
sought to hold the officers and stockholders
directly liable for a corporate debt or
obligation." In the instant case, petitioner
does not seek to impose a claim against the
members of the Acrylic.

85
| Labor Relations-2A

69_Fajardo certificate election, also among the regular


rank-and-file employees of CMC. They
anchor their petition on Section 3, Rule V
CAPITOL MEDICAL CENTER OF Book V of the Rules Implementing the
CONCERNED EMPLOYEES-UNIFIED Labor Code on when a certificate election
FILIPINO SERVICE WORKERS (CMC- should be conducted. They alleged that 1
ACE-UFSW) year had already lapsed since respondent
union was certified as the exclusive
vs bargaining agent and no CBA was as yet
concluded or bargaining deadlock to which
HON. BIENVENIDO E. LAGUESMA, the incumbent or certified bargaining agent
CAPITOL MEDICAL CENTER is a party had been submitted to conciliation
EMPLOYEES ASSOCIATION-ALLIANCE or had become the subject of a valid notice
OF FILIPINO WORKERS AND CAPITOL of strike or lockout, hence, there is no bar to
MEDICAL CENTER INCORPORATED the holding of a certificate election.
G.R. No. 118915 February 4, 1997

ISSUE:
FACTS: Whether or not petitioner union’s
petition for certificate election shall be
Respondent union applied for
granted.
certificate of election among the rank-and-
file employees of CMC. The Med-Arbiter
granted the same. Although CMC
questioned the former’s legal status, the HELD:
union was able to register as an
NO.
independent union and secure another
certificate of election. Elections were finally It is true that the certificate was filed
held, and the union became the sole and after the lapse of one year from the time of
exclusive bargaining representative of the declaration of a final certification result and
rank and file employees at CMC. However, that no bargaining deadlock had been
CMC refuse to negotiate with respondent submitted for conciliation or arbitration,
union but instead file a petition for the however respondent union was not remiss
cancellation of the certificate of registration on its rights to enter into a CBA for it was
of respondent union. Due to CMC’s refusal the CMC which refused to bargain
to negotiate, respondent union stage a collectively.
strike, to which the SOLE assume
jurisdiction. Meanwhile, the case for the Deadlock is the counteraction of
cancellation of respondent union’s things producing entire stoppage. It
registration reached the Supreme Court presupposes reasonable effort at good faith
which declared with finality the legality of bargaining which, despite noble intentions,
respondent union. The Court issued an does not conclude in agreement between
Entry of Judgment on March 23, 1994. The the parties.
next day, petitioner union filed a petition for

86
| Labor Relations-2A

Although in the present case, there 70_Ferrer


is no “deadlock” in its strict sense, what
happened is worse than a bargaining
deadlock, for CMC employed all legal SAMAHANG MANGGAGAWA SA
means to block the certification of PERMEX (SMP-PIILU-TUCP) vs. THE
respondent union as the bargaining agent of SECRETARY OF LABOR, NATIONAL
the rank-and-file; and use it as leverage for FEDERATION OF LABOR, PERMEX
its failure to bargain with respondent union. PRODUCER AND EXPORTER
CMC was unwilling to negotiate and reach CORPORATION
an agreement with respondent union. To
grant petitioner union’s application, taking [G.R. No. 107792; March 2, 1998]
advantage of the delay caused by CMC,
would give premium on the failure of the
hospital to perform its duty to bargain
Facts:
collectively as mandated by Article 252 of
the Labor Code. On January 15, 1991, a certification election
was conducted among employees of
----- ----- ---- ---- ---- ----- 
respondent Permex Producer and Exporter
The case was compared to the case Corporation (hereafter referred to as
of Kaisahan ng Manggagagawang Pilipino v Permex Producer). The results of the
Trajano. In this case, the subsequent elections were as follows:
petition for certificate of election was
National Federation of Labor (NFL) - 235
granted although there is no bargaining
deadlock to speak of. The obiter advanced No Union- 466
the idea that the certification would not have
been granted if only the union is not remiss Spoiled Ballots - 18
in its obligation to compel the employer of
its duty to bargain collectively. Marked Ballots - 9

Unlike in the case of CMC, the union Challenged Ballots- 7


took action to legally coerce the hospital to
However, some employees of Permex
bargain when it staged a strike.
Producer formed a labor organization
known as the Samahang Manggagawa sa
Permex (SMP) which they registered with
the Department of Labor and Employment
on March 11, 1991. The union later affiliated
with the Philippine Integrated Industries
Labor Union (PIILU).

On August 16, 1991, Samahang


Manggagawa sa Permex-Philippine
Integrated Industries Labor Union (SMP-
PIILU), wrote the respondent company
requesting recognition as the sole and

87
| Labor Relations-2A

exclusive bargaining representative of employer to bargain collectively, the


employees at the Permex Producer. On employer may do one of two things. First, if
October 19, 1991 Permex Producer the employer is satisfied with the employees
recognized SMP-PIILU and, on December claim the employer may voluntarily
1, entered into a collective bargaining recognize the union by merely bargaining
agreement with it. The CBA was ratified collectively with it. The formal written
between December 9 and 10, 1991 by the confirmation is ordinarily stated in the
majority of the rank and file employees of collective bargaining agreement. Second, if
Permex Producer. On December 13, 1991, on the other hand, the employer refuses to
it was certified by the DOLE. recognize the union voluntarily, it may
petition the Bureau of Labor Relations to
conduct a certification election. If the
On February 25, 1992, respondent NFL filed employer does not submit a petition for
a petition for certification election, but it was certification election, the union claiming to
represent the employees may submit the
dismissed by Med-Arbiter Edgar B.
Gongalos in an order dated August 20, petition so that it may be directly certified as
1992. Respondent NFL then appealed the the employees’ representative or a
order to the Secretary of Labor and certification election may be held.
Employment. On October 8, 1992, the
Secretary of Labor, through Undersecretary
Bienvenido Laguesma, set aside the order Issue:
of the Med-Arbiter and ordered a
certification election to be conducted among Whether or not Samahang Manggagawa ng
the rank and file employees at the Permex Permex is the sole and exclusive bargaining
Producer, with the following choices: agent of Permex Producer employees even
if not elected through certification election
1. National Federation of Labor

2. Samahang Manggagawa sa Permex


Held:
3. No union
No, Samahang Manggagawa ng Permex
Petitioner moved for a reconsideration but cannot be the sole and exclusive bargaining
its motion was denied in an order dated agent of Permex Producer employees
November 12, 1992. Hence, this petition. without winning a certification election,
despite the fact that Permex Producer
Two arguments are put forth in support of already entered a CBA with the former. The
the petition. First, it is contended that Decision of Secretary of Labor ordering a
petitioner has been recognized by the new certification election was upheld.
majority of the employees at Permex
Producer as their sole collective bargaining
agent. Petitioner argues that when a group
of employees constituting themselves into The case of Ilaw at Buklod ng Manggagawa
an organization and claiming to represent a v. Ferrer-Calleja, cited by the Solicitor
majority of the work force requests the General in his comment filed in behalf of the
NLRC, is particularly apropos. There, the

88
| Labor Relations-2A

union also requested voluntary recognition previously allowed under the Labor Code
by the company. Instead of granting the had been discontinued as a method of
request, the company petitioned for a selecting the exclusive bargaining agents of
certification election. The union moved to the workers. Certification election is the
dismiss on the ground that it did not ask the most effective and the most democratic way
company to bargain collectively with it. As of determining which labor organization can
its motion was denied, the union brought the truly represent the working force in the
matter to this Court. In sustaining the appropriate bargaining unit of a company.
company’s stand, this Court ruled:

Petitioner argues that of the 763 qualified


...Ordinarily, in an unorganized employees of Permex Producer, 479
establishment like the Calasiao Beer supported its application for registration with
Region, it is the union that files a petition for the DOLE and that when petitioner signed
a certification election if there is no certified
the CBA with the company, the CBA was
bargaining agent for the workers in the
establishment. If a union asks the employer ratified by 542 employees. Petitioner
to voluntarily recognize it as the bargaining contends that such support by the majority
agent of the employees, as the petitioner of the employees justifies its finding that the
did, it in effect asks the employer to certify it CBA made by it is valid and binding. But it is
as the bargaining representative of the not enough that a union has the support of
employees A CERTIFICATION WHICH the majority of the employees. It is equally
THE EMPLOYER HAS NO AUTHORITY
important that everyone in the bargaining
TO GIVE, for it is the employees prerogative
(not the employers) to determine whether unit be given the opportunity to express
they want a union to represent them, and, if himself.
so, which one it should be. (emphasis
supplied) This is especially so because, in this case,
the recognition given to the union came
barely ten months after the employees had
In accordance with this ruling, Permex voted No Union in the certification election
Producer should not have given its conducted in the company. As pointed out
voluntary recognition to SMP-PIILU-TUCP by respondent Secretary of Labor in his
when the latter asked for recognition as decision, there can be no determination of a
exclusive collective bargaining agent of the bargaining representative within a year of
employees of the company. The company the proclamation of the results of the
did not have the power to declare the union certification election. Here the results, which
the exclusive representative of the workers showed that 61% of the employees voted
for the purpose of collective bargaining. for no union, were certified only on February
25, 1991 but on December 1, 1991 Permex
Producer already recognized the Samahang
Indeed, petitioner’s contention runs counter Manggagawa ng Permex and entered into a
to the trend towards the holding of CBA with it.
certification election. By virtue of Executive
Order No. 111, which became effective on
March 4, 1987, the direct certification

89
| Labor Relations-2A

There is something dubious about the fact freedom of choice because it does not
that just ten months after the employees establish the kind of industrial peace
had voted that they did not want any union contemplated by the law. Such situation
to represent them, they would be obtains in this case. The petitioner entered
expressing support for petitioner. The doubt into a CBA with Permex Producer when its
is compounded by the fact that in sworn status as exclusive bargaining agent of the
affidavits some employees claimed that they employees had not been established yet.
had either been coerced or misled into
signing a document which turned out to be
in support of petitioner as its collective
bargaining agent. Although there were
retractions, we agree with the Solicitor
General that retractions of statements by
employees adverse to a company (or its
favored union) are oftentimes tainted with
coercion and intimidation. For how could
one explain the seeming flip-flopping of
position taken by the employees? The
figures claimed by petitioner to have been
given to it in support cannot readily be
accepted as true.

Second. Petitioner invokes the contract-bar


rule. They contend that under Arts. 253,
253-A and 256 of the Labor Code and Book
V, Rule 5, 3 of its Implementing Rules and
Regulations, a petition for certification
election or motion for intervention may be
entertained only within 60 days prior to the
date of expiration of an existing collective
bargaining agreement. The purpose of the
rule is to ensure stability in the relationships
of the workers and the management by
preventing frequent modifications of any
collective bargaining agreement earlier
entered into by them in good faith and for
the stipulated original period. Excepted from
the contract-bar rule are certain types of
contracts which do not foster industrial
stability, such as contracts where the
identity of the representative is in doubt.
Any stability derived from such contracts
must be subordinated to the employees

90
| Labor Relations-2A

71_Hadloc effective only for the remaining period of two


years or until June 30, 1994. SMC, on the
San Miguel Corp. Employees Union- other hand, contended that the employees
PGTWO V. Confessor who had moved to Magnolia and SMFI,
automatically ceased to be part of the
FACTS: bargaining unit at the SMC. Furthermore,
Petitioner-union entered into a CBA the CBA should be effective for three years
with private respondent San Miguel in accordance with Art. 253-A of the Labor
Corporation (SMC) to take effect upon the Code.
expiration of their previous CBA on June 30,
1989.This CBA provided, among others, ISSUES: 1) W/N the duration of the
that: renegotiated terms of the CBA is to be
XXX XXX XXX effective for 2 years
Sec. 1. This Agreement which shall be 2) W/N the bargaining unit of SMC
binding upon the parties hereto and their includes also the employees of the
respective successors-in-interest, shall Magnolia and SMFI.
become effective and shall remain in force
and effect until June 30, 1992. RULING:
Sec. 2. In accordance with Article 253-A of 1) NO. Art. 253-A of the Labor Code
the Labor Code as amended, the term of as amended reads:
this Agreement insofar as the Art. 253-A. Terms of a CBA. — Any
representation aspect is concerned, shall be CBA that the parties may enter into shall,
for five (5) years from July 1, 1989 to June insofar as the representation aspect is
30, 1994. Hence, the freedom period for concerned, be for a term of 5 years. No
purposes of such representation shall be petition questioning the majority status of
sixty (60) days prior to June 30, 1994. the incumbent bargaining agent shall be
Sec. 3. Sixty (60) days prior to June 30, entertained and no certification election
1992 either party may initiate negotiations of shall be conducted by the Department of
all provisions of this Agreement, except Labor and Employment outside of the sixty-
insofar as the representation aspect is day period immediately before the date of
concerned. If no agreement is reached in expiry of such five year term of the CBA. All
such negotiations, this Agreement shall other provisions of the CBA shall be
nevertheless remain in force up to the time renegotiated not later than 3 years after its
a subsequent agreement is reached by the execution. Any agreement on such other
parties. provisions of the CBA entered into within 6
months from the date of expiry of the term
Meanwhile, Magnolia and Feeds and of such other provisions as fixed in such
Livestock Division were spun-off and CBA, shall retroact to the day immediately
became two separate and distinct following such date. If any such agreement
corporations: Magnolia Corporation is entered into beyond six months, the
(Magnolia) and San Miguel Foods, Inc. parties shall agree on the duration of
(SMFI). Notwithstanding the spin-offs, the retroactivity thereof. In case of a deadlock in
CBA remained in force and effect. After the renegotiation of the CBA, the parties
June 30, 1992, the CBA was renegotiated in may exercise their rights under this
accordance with the terms of the CBA and Code.xxx The “representation aspect” refers
Article 253-A of the Labor Code. During the to the identity and majority status of the
negotiations, the petitioner-union insisted union that negotiated the CBA as the
that the bargaining unit of SMC should still exclusive bargaining representative of the
include the employees of the spun-off appropriate bargaining unit concerned. “All
corporations: Magnolia and SMFI; and that other provisions” simply refers to the rest of
the renegotiated terms of the CBA shall be

91
| Labor Relations-2A

the CBA, economic as well as non- representatives of the employees


economic provisions, except representation. concerned.
The law is clear and definite on the duration Therefore, Magnolia and SMFI became
of the CBA insofar as the representation distinct entities with separate juridical
aspect is concerned, but is quite ambiguous personalities. Thus, they can not belong to a
with the terms of the other provisions of the single bargaining unit. Moreover, in
CBA. It is the framers of the law’s intention determining an appropriate bargaining unit,
to maintain industrial peace and stability by the test of grouping is mutuality or
having both management and labor work commonality of interests. The employees
harmoniously together without any sought to be represented by the collective
disturbance. Thus, no outside union can bargaining agent must have substantial
enter the establishment within 5 years and mutual interests in terms of employment
challenge the status of the incumbent union and working conditions as evinced by the
as the exclusive bargaining agent. Likewise, type of work they performed. Considering
the terms and conditions of employment the spin-offs, the companies would
(economic and non-economic) can not be consequently have their respective and
questioned by the employers or employees distinctive concerns in terms of the nature of
during the period of effectivity of the CBA. work, wages, hours of work and other
The CBA is a contract between the parties conditions of employment. Interests of
and the parties must respect the terms and employees in the different companies
conditions of the agreement. Notably, the perforce differ. The nature of their products
framers of the law did not give a fixed term and scales of business may require different
as to the effectivity of the terms and skills which must necessarily be
conditions of employment. It can be gleaned commensurated by different compensation
from their discussions that it was left to the packages. The different companies may
parties to fix the period. have different volumes of work and different
The issue as to the term of the non- working conditions. For such reason, the
representation provisions of the CBA need employees of the different companies see
not belaboured. The parties, by mutual the need to group themselves together and
agreement, enter into a renegotiated organize themselves into distinctive and
contract with a term of three (3) years or different groups. It would then be best to
one which does not coincide with the said 5- have separate bargaining units for the
year term, and said agreement is ratified by different companies where the employees
majority of the members in the bargaining can bargain separately according to their
unit, the subject contract is valid and legal needs and according to their own working
and therefore, binds the contracting parties. conditions.

2) NO. The transformation of the companies


was a management prerogative and
business judgment which the courts can not
look into unless it is contrary to law, public
policy or morals. Ever mindful of the
employees’ interests, management has
assured the concerned employees that they
will be absorbed by the new corporations
without loss of tenure and retaining their
present pay and benefits according to the
existing CBAs. They were advised that upon
the expiration of the CBAs, new agreements
will be negotiated between the management
of the new corporations and the bargaining

92
| Labor Relations-2A

72_Leano 2) whether or not a chapters legal


personality may be collaterally attacked in a
LAGUNA AUTOPARTS petition for certification election; and
MANUFACTURING CORPORATION 3)whether or not the petitioner, as the
vs. OFFICE OF THE SOLE [G.R. No. employer, has the legal standing to oppose
157146. April 29, 2005] the petition for certification election.

FACTS: HELD:
The respondent union filed a petition for First of all, in a petition for review
certification election before the DOLE, on certiorari, a petitioner can raise only
alleging that Obrero Pilipino and its chapter questions of law. The SC is not the proper
affiliate, LAMCOR Chapter were legitimate venue to consider a factual issue as it is not
labor organizations. It averred that the said a trier of facts. Findings of fact of
bargaining unit, composed of more or less administrative agencies and quasi-judicial
160 rank-and-file employees, is bodies, which have acquired expertise
unorganized and that there has been no because their jurisdiction is confined to
certification election conducted for the past specific matters, are generally accorded not
12 months prior to the filing of the petition. only great respect but even finality.
The petitioner company moved to dismiss
the petition for certification election and 1. YES, based on the finding of the SOLE,
further asserted that even if the respondent the respondent union is a legitimate labor
union was issued a certificate of organization. As correctly stressed by the
registration, it could not file a petition for CA, a local or chapter need not be
certification election since its legal registered to become a legitimate labor
personality was at question. organization because a local or chapter
acquires legal personality as a labor
Med-Arbiter Bactin dismissed the petition organization from the date of filing of the
for certification election for the respondent complete documents enumerated in Section
unions lack of legal personality because it 1of Rule VI of the Implementing Rules of
failed to indicate its principal office on the Book V (as amended by Department Order
documents it submitted to the Regional [D.O.] No. 9).
Office. The respondent union appealed the
case to the SOLE Sto. Tomas, who granted The task of determining whether the
the appeal and remanded the case to the local or chapter has submitted the complete
regional office of origin for the immediate documentary requirements is lodged with
conduct of a certification election. Finding the Regional Office or the BLR. The records
no cogent reason to alter her decision, the of the case show that the respondent union
SOLE denied the motion for submitted the said documents to Regional
reconsideration thereof. Office No. IV and was subsequently issued
certificates. It, therefore, declared that the
Not convinced, the petitioner filed a petition respondent union has acquired legal
for certiorari with the CA, who ruled in favor personality as a labor organization. Absent
of the respondnet union. The petitioner’s any pronouncement to the contrary, such
MR was denied. Hence, this petition for determination of the Labor Relations
review on certiorari. Division Chief will stand, on the presumption
that the duty of determining whether the
ISSUES: respondent union submitted the complete
1)whether or not the respondent union documentary requirements has been
is a legitimate labor organization; regularly performed.

93
| Labor Relations-2A

2. NO. Legal personality may not be subject 73_Leonardo


to a collateral attack but only through a
separate action instituted particularly for the
purpose of assailing it, as prescribed by
Section 5, Rule V of the Implementing Rules REYES, et al vs. CRESENCIANO B.
of Book V. . What applies in this case is the TRAJANO, et al. and Tri-union
principle that once a union acquires
Employees Union
legitimate status as a labor organization, it
continues as such until its certificate of G.R. No. 84433 June 2, 1992
registration is cancelled or revoked in an
independent action for cancellation.
FACTS:
Equally important is Section 11, Paragraph
II, Rule IX of D.O. 9, which provides for the
dismissal of a petition for certification The Bureau of Labor Relations
election based on the lack of legal
(BLR ) authorized the conduct of
personality of a labor organization only in
the following instances: (1) appellant is not certification election among the employees
listed by the Regional Office or the BLR in of Tri-Union Industries Corporation. The
its registry of legitimate labor organizations; competing unions were the TUEU-OLALIA
or (2) appellants legal personality has been and TUPAS. Of the 384 workers initially
revoked or cancelled with finality. deemed to be qualified voters, only 240
actually took part in the election. Among the
3. Petitioner does not have the legal 240 who cast their votes, 141 were
standing to oppose the petition for
members of the Iglesia ni Kristo (INK). The
certification election. The choice of a
collective bargaining agent is the sole ballots provided for three (3) choices. They
concern of the employees. The only provided for votes to be cast, of course, for
exception to this rule is where the employer either of the two (2) contending labor
has to file the petition for certification organizations, (a) TUPAS and (b) TUEU-
election pursuant to Article 258 of the LC OLALIA; and, conformably with established
because it was requested to bargain
rule and practice, 1 for (c) a third
collectively. An employer’s role in a
certification election has aptly been choice: "NO UNION."
described in Trade Unions of the Philippines
and Allied Services (TUPAS) v. Trajano, as
that of a mere bystander. It has no legal
The challenged votes were those
standing in a certification election as it
cannot oppose the petition or appeal the cast by the 141 INK members. They were
Med-Arbiters orders related thereto. segregated and excluded from the final
count because the competing unions
agreed earlier that the INK members should
not be allowed to vote “because they are
not members of any union and refused to
participate in the previous certification
election.”

94
| Labor Relations-2A

The INK employees protested the a labor union or determine which of two or
exclusion of their votes. They filed a petition more unions in an establishment to join, and
to cancel the election alleging that it “was to engage in concerted activities with co-
not fair” and the result thereof did “not workers for purposes of collective
reflect the true sentiments of the majority of bargaining through representatives of their
the employees.” own choosing, or for their mutual aid and
protection. Logically, the right NOT to join,
affiliate with, or assist any union, and to
TUEU-OLALIA opposed the petition. disaffiliate or resign from a labor
It contended that petitioners “do not have organization, is subsumed in the right to
legal personality to protest the results of the join, affiliate with, or assist any union, and to
election because they are not members of maintain membership therein. The right to
either the contending unions, but of the INK form or join a labor organization necessarily
which prohibits its followers to, on religious includes the right to refuse or refrain from
exercising said right. It is self-evident that
grounds, from joining or forming any labor
organization. just as no one should be denied the
exercise of a right granted by law, so also,
no one should be compelled to exercise
such a conferred right. The fact that a
The Med-Arbiter, seeing no merit in person has opted to acquire membership in
the INK employees’ petition, certified the a labor union does not preclude his
TUEU-OLALIA as the sole and exclusive subsequently opting to renounce such
bargaining agent of the rank-and-file membership.
employees. The officer-in-charge of BLR
(Hon. Cresenciano Trajano) sustained the The purpose of a certification
denial by the Med Arbiter of the right to vote election is precisely the ascertainment of
of one hundred forty-one (141) members of the wishes of the majority of the employees
the "Iglesia ni Kristo" (INK). in the appropriate bargaining unit: to be or
not to be represented by a labor
organization, and in the affirmative case, by
which particular labor organization. If the
ISSUE: results of the election should disclose that
the majority of the workers do not wish to be
represented by any union, then their wishes
must be respected, and no union may
Whether or not union properly be certified as the exclusive
membership is a prerequisite in order representative of the workers in the
to vote in a Certification Election. bargaining unit in dealing with the employer
regarding wages, hours and other terms
and conditions of employment. In the
Certification Election, all members of the
HELD: No unit, whether union members or not, have
the right to vote. Union membership is not
prerequisite. If majority of the unit members
do not want a union, as expressed in the
The right of self-organization certification election, such majority decision
includes the right to organize or affiliate with

95
| Labor Relations-2A

must be respected. Hence, the INK 74_Magabilin


members may vote.
AIRTIME SPECIALISTS, INC.VS
Moreover, the contention that the
petitioners are disqualified to vote because DIRECTOR OF LABOR RELATIONS
they "are not constituted into a duly FERRER-CALLEJA
organized labor union" — "but members of
the INK which prohibits its followers, on G.R. No. 80612-16 December 29, 1989
religious grounds, from joining or forming
any labor organization" — and "hence, not
FACTS:
one of the unions which vied for certification
Respondent Samahan ng mga
as sole and exclusive bargaining
Manggagawa sa Asia-FFW Chapter
representative," is specious. Neither law,
(SAMA-ASIA, for short) filed with the
administrative rule nor jurisprudence
National Capital Region, Ministry of Labor
requires that only employees affiliated with
and Employment, two separate petitions for
any labor organization may take part in a
direct certification and/or certification electio
certification election. Neither does the
n on behalf of the regular rank-and-
contention that petitioners should be denied
file employees of the petitioners Airtime
the right to vote because they "did not
Specialists and Absolute Sound, Inc. The
participate in previous certification elections
other respondent Pinagbuklod ng
in the company for the reason that their
Manggagawa sa Ataco-FFW Chapter (PMA
religious beliefs do not allow them to form,
for short) also filed with the same office, on
join or assist labor organizations," persuade
the same day, similar separate petitions in
acceptance. No law, administrative rule or
behalf of the regular rank
precedent prescribes forfeiture of the right
and file employees of petitioners Country-
to vote by reason of neglect to exercise the
Wealth Development, Ad Planner and
right in past certification elections.
Marketing Counsellors and Atlas
Resources.

All these five cases were consolidated.


Petitioners filed their position paper with
motion to dismiss on the following grounds
— disaffiliation of the rank
and file employees, ineligibility of some
signatories because they had less than one
(1) year of service resulting in the non-
compliance with the 30% requirement. The
Med-Arbiter issued an Order mandating
a certification election to be conducted
among the rank and file employees of the
Airtime Specialists, Inc.; Absolute Sound,
Inc.; Commonwealth Development
Corporation; Ad Planners & Mktg. Corp.;
and Atlas Resources & Management Group,
within 20 days from receipt of the Order.

Petitioners’ motion for reconsideration


having been denied they filed the instant
petition for “Certiorari and Prohibition with
Preliminary Injunction” with a Prayer for the

96
| Labor Relations-2A

issuance of a temporary restraining order In a certification election all rank-and-file


enjoining public respondents from employees in the appropriate bargaining
conducting any further proceedings in the unit are entitled to vote. This principle is
said five cases. clearly stated in Art. 255 of the Labor Code
which states that the "labor organization
designated or selected by the majority of the
ISSUE: employees in an appropriate bargaining unit
WON the Bureau of Labor Relations (public shall be the exclusive representative of the
respondents) committed grave abuse of employees in such unit for the purpose of
discretion when they considered (a) collective bargaining." Collective bargaining
employees with less than one year of covers all aspects of the employment
service and even (b) probationary relation and the resultant CBA negotiated by
employees as qualified participants in the the certified union binds all employees in
certification election process. the bargaining unit. Hence, all rank-and-file
employees, probationary or permanent,
HELD: have a substantial interest in the selection
No. The Court had made it clear that the of the bargaining representative. The Code
Court of Industrial Relations, (or in this makes no distinction as to their employment
case), the Bureau of Labor Relations in status as basis for eligibility in supporting
deciding whether or not to grant a petition the petition for certification election. The law
for certification election considering the refers to "all" the employees in the
facts and circumstances of which it has bargaining unit. All they need to be eligible
intimate knowledge. Moreover, a perusal of to support the petition is to belong to the
Art. 258 of the Labor Code as amended by "bargaining unit.".
Presidential Decree No. 442 reveals that
compliance with the 30% requirement (now
20%) makes it mandatory upon the Bureau
of Labor Relations to order the holding of a
certification election in order to determine
the exclusive-bargaining agent of the
employees. Stated otherwise, it means that
with such, the Bureau is left without any
discretion but to order the holding of
certification election. Parenthetically, where
the petition is supported by less than 30%
(now 20%) the Bureau of Labor Relations
has discretion whether or not to order the
holding of certification election depending
on the circumstances of the case. Thus, it is
the Court’s holding in LVN Pictures vs.
Musicians Guild, et al. (1 SCRA 132) that in
connection with certification election, the
Court of Industrial Relations enjoys a wide
discretion in determining the procedure
necessary to insure a fair and free choice of
bargaining representatives by employees,
and having exercised its sound discretion,
this Court cannot interfere. (Arguelles v.
Young, 153 SCRA 690).

97
| Labor Relations-2A

STRIKES AND LOCKOUTS respondents to file on October 31, 1990 a


petition to declare the work stoppage illegal
75_Mallari for failure to comply with procedural
requirements.
BIFLEX UNION VS. FILFLEX INDUSTRIAL
On November 13, 1990, respondents
MANUFACTURING
resumed their operations. Petitioners,
G.R. No. 155679. December 19, 2006 claiming that they were illegally locked out
by respondents, assert that aside from the
FACTS: fact that the welga ng bayan rendered it
Petitioners Patricia Villanueva, Emilia difficult to get a ride and the apprehension
Bandola, Raquel Cruz, Delia Relato, Regina that violence would erupt between those
Castillo, Lolita delos Angeles, Marissa
participating in the welga and the
Villoria, Marita Antonio, Lolita Lindio, Eliza
Caraulia, and Liza Sua were officers of authorities, respondents workers were
Biflex (Phils.) Inc. Labor Union. prevented from reporting for work.
Petitioners Myrna dela Torre, Avelina
Aonuevo, Bernice Borcelo, Narlie Yagin, On their putting up of tents, tables and
Evelyn Santillan, Leony Serdoncilo, Trinidad chairs in front of the main gate of
Cuya, Andrea Lumibao, Gynie Arneo, respondents premises, petitioners, who
Elizabeth Capellan, Josephine Detosil, claim that they filed a notice of strike on
Zenaida Francisco, and Florencia Anago October 31, 1990, explain that those were
were officers of Filflex Industrial and for the convenience of union members who
Manufacturing Labor Union. reported every morning to check if the
management would allow them to report for
The two petitioner-unions, which are work.
affiliated with National Federation of Labor
Unions (NAFLU), are the respective Respondents, on the other hand, maintain
collective bargaining agents of the that the work stoppage was illegal since the
employees of corporations. following requirements for the staging of a
valid strike were not complied with: (1) filing
Respondents Biflex (Phils.) Inc. and Filflex of notice of strike; (2) securing a strike vote,
Industrial and Manufacturing Corporation and (3) submission of a report of the strike
(respondents) are sister companies vote to the Department of Labor and
engaged in the garment business. Situated Employment.
in one big compound along with another
sister company, General Garments Labor Arbiter – strike was illegal.
Corporation (GGC), they have a common NLRC- reversed the ruling of the LA. No
entrance. strike to speak of as no labor or
On October 24, 1990, the labor sector industrial dispute existed between the
staged a welga ng bayan to protest the parties.
accelerating prices of oil. On even date, CA – reinstated the ruling of the LA.
petitioner-unions, led by their officers,
herein petitioners, staged a work stoppage
which lasted for several days, prompting

98
| Labor Relations-2A

ISSUE: purposes, or obstruct public


Whether or not the union officers engaged thoroughfares."
in an illegal strike.
Even the NLRC, which ordered their
HELD: reinstatement, took note of petitioners’ act
YES. of "physically blocking and preventing the
Stoppage of work due to welga ng bayan is
entry of complainant’s customers, supplies
in the nature of a general strike, an
and even other employees who were not on
extended sympathy strike. It affects
strike."
numerous employers including those who
do not have a dispute with their employees In fine, the legality of a strike is determined
regarding their terms and conditions of not only by compliance with its legal
employment. formalities but also by the means by which it
is carried out.
Employees who have no labor dispute with
their employer but who, on a day they are Petitioners, being union officers, should thus
scheduled to work, refuse to work and bear the consequences of their acts of
instead join a welga ng bayan commit an knowingly participating in an illegal strike,
illegal work stoppage. conformably with the third paragraph of
Article 264 (a) of the Labor Code which
There being no showing that petitioners
provides:
notified respondents of their intention, or
that they were allowed by respondents, . . . Any union officer who knowingly
to join the welga ng bayan on October participates in an illegal strike and any
24, 1990, their work stoppage is beyond worker or union officer who knowingly
legal protection. participates in the commission of illegal acts
during a strike may be declared to have lost
Petitioners, nonetheless, assert that when
his employment status: Provided, That mere
they returned to work the day following the
participation of a worker in a lawful strike
welga ng bayan on October 24, 1990, they
shall not constitute sufficient ground for
were refused entry by the management,
termination of his employment, even if a
allegedly as punishment for their joining the
replacement had been hired by the
welga. Hence, they claim that they were
employer during such lawful strike.
illegally locked out by respondents.

Even assuming arguendo that in staging


the strike, petitioners had complied with
legal formalities, the strike would just the
same be illegal, for by blocking the free
ingress to and egress from the company
premises, they violated Article 264(e) of
the Labor Code which provides that
"[n]o person engaged in picketing shall
… obstruct the free ingress to or egress
from the employer’s premises for lawful

99
| Labor Relations-2A

76_Panganiban sent letters to the employees requiring them


to explain why they should not be
dismissed. Union explained that they only
exercised their constitutional right of
Toyota Motor Phils. Corp. Workers peaceably assemble and to petition
Association v. NLRC government for redress if grievances.
GR Nos. 158786 & 158789 October 19, However, union circulated a Manifesto
2007 urging its members to participate in a strike
and abandon posts. It filed another notice of
strike. Toyota required the employees to
attend an investigative review, but they
Toyota Motor Philippines Corporation v. refused to do so. Toyota terminated 227
Toyota Motor Philippines Workers employees for participation in concerted
Association actions in violation of its Code of Conduct
and misconduct under Article 282 of Labor
GR Nos. 158789 & 158799 October 19,
Code. In view of the dismissal, union went
2007
on strike and from March 28, 2001 to April
12, 2001, it barricaded the gates of Toyota’s
Bicutan and Sta. Rosa plants. This
Facts: Union filed a petition for certification prevented others from reporting to work and
election among Toyota rank and file delayed the operations. Toyota filed petition
employees with NCMB and was certified as for injunction with a prayer for issuance of
sole and exclusive bargaining agent of all TRO which was granted. A petition to
Toyota rank and file employees. It submitted declare the strike illegal was also filed.
CBA proposals to Toyota, but the latter
refused to negotiate because of its pending
appeal. Union filed notice of strike in view of
DOLE Secretary assumed
Toyota’s refusal to bargain. NCMB
jurisdiction over the labor dispute and
converted the notice of strike into a
issued an order certifying the labor dispute
preventive mediation case on the ground
to NLRC. The striking workers were directed
that the issue of whether or not union is
to return to work, Toyota to accept the
exclusive bargaining agent was still
workers or put them under payroll
unresolved. On February 21, 2001, 135
reinstatement, and both of them to enjoin
union officers and members failed to render
from committing acts that may worsen the
the required overtime work and instead
situation. Despite issuance of Certification
marched to and staged a picket in front of
Order, several payroll reinstated members
BLR office. The union members requested
of Union staged a protest in front of Bicutan
to be absent the following day to attend the
plant on May 23, 2001. Five days after, 44
hearing but the request was denied by
union members staged another protest in
Toyota. Despite denial, more than 200
the same plant and 29 reinstated
employees staged mass actions in front of
employees in Sta. Rosa plant.
BLR and DOLE Offices which led to acute
Notwithstanding certification order, another
lack of manpower and failure to meet
notice of strike was filed. NLRC declared
production goals of Toyota. The company
the strikes illegal and ordered severance

100
| Labor Relations-2A

compensation to employees despite legal dispute and certified the case for
dismissal. compulsory arbitration. Although there was
no stoppage of work, the fact still remains
that union members picketed and performed
Issues: (1) Whether or not the mass actions concerted actions which clearly violated the
committed by the union on different assumption of jurisdiction and certification
occasions are order of DOLE Secretary.

illegal strikes

(2) Whether or not separation pay (2) No, it should not be awarded. The
should be awarded to union members general rule is that when just causes for
terminating the services of an employee
Held: (1) Yes, the strikes were illegal. There exist, employee is not entitled to separation
were 6 categories of illegal strike: a) when pay. The rationale behind this is that the
contrary to specific prohibition of law; b) lawbreaker should not benefit from their
when it violates specific requirement of law; illegal acts. Exception to this is when the
c) when it is for unlawful purpose; d) when it court finds justification in applying principles
employs unlawful means to achieve of social justice. However, the principle will
objective; e) when it is declared in violation only be considered when cause of dismissal
of existing document; and f) when contrary is other than serious misconduct or that
to existing agreement. which reflects adversely employee’s moral
character. The Court ruled that, considering
that the dismissal of employees was due to
participation in illegal strikes as well as
The Court ruled that the protest
violation of Code of Conduct, the same
actions on February 2001 were not exercise
constitute serious misconduct. It is a
of constitutional right to assemble and ask
transgression of established and definite
Government for redress but in reality
rule of action and implies wrongful intent.
temporary work stoppages that crippled
The claim of union that Toyota breach its
Toyota. The protests were done without
duty to bargain has not been substantiated
satisfying prerequisites of valid strike under
by evidence as to justify union’s strategy to
Article 263 of Labor Code. Also, such
cripple operations of Toyota inflicting almost
violated the Toyota’s Code of Conduct to
50 Million financial damage. The employees
which dismissal is the penalty. The March
were also fully aware of the company’s
2001 strikes were also illegal because
policy on prohibition against concerted
unlawful means were employed. Union
action inimical to company’s interest yet
blocked the the free ingress and egress of 2
they still resorted to mass actions on
plants. The Court disagreed with the
several occasions. Above all, they blatantly
contention of union that there can be no
violated the assumption and certification
strike to speak in May 2001 because
order of DOLE Secretary which only proved
participants were the dismissed employees
their lack of obeisance to the law.
who were on payroll. The Court emphasized
that by that time the DOLE Secretary
already assumed jurisdiction over the labor

101
| Labor Relations-2A

77_Questin submitted to the NCMB the minutes of the


alleged strike vote purportedly held on
CAPITOL MEDICAL CENTER, INC. v. November 10, 1997 at the parking lot in
NATIONAL LABOR RELATIONS front of CMC’s premises, at the corner of
COMMISSION, ET AL. Scout Magbanua Street and Panay Avenue,
Quezon City. It appears that 178 out of the
G.R. No. 147080, April 26, 2005, SECOND 300 union members participated therein,
DIVISION, (Callejo, Sr., J.) and the results were as follows: 156
DOCTRINE OF THE CASE members voted to strike; 14 members cast
negative votes; and eight votes were
The failure of a union to comply with spoiled. On November 28, 1997, the officers
the requirement of the giving of notice to the and members of the Union staged a strike.
NCMB at least 24 hours prior to the holding
of a strike vote meeting will render the The Union filed an ex parte motion
subsequent strike staged by the union with the DOLE, praying for its assumption of
illegal. jurisdiction over the dispute. The SOLE
assumed jurisdiction over the dispute. Thus,
FACTS: the officers and the members of the Union
stopped their strike and returned to work.
Capitol Medical Center Employees
Association-Alliance of Filipino Workers (the For its part, CMC filed a petition
Union) was declared by the Supreme Court, before the National Labor Relations
in a separate petition, as the exclusive Commission (NLRC) to declare the strike
bargaining agent of the rank-and-file illegal. The Labor Arbiter (LA) ruled in favor
employees of Capitol Medical Center, Inc. of CMC, and declared that the strike
(CMC). conducted by the Union as illegal. The LA
reasoned that there was no strike vote
Due to CMC’s failure to accede to conducted, and the Union failed to comply
the Union’s repeated requests for a meeting with the requirements set by law before a
to discuss matters pertaining to a strike could be staged. Aggrieved, the Union
negotiation for a CBA, the Union filed a appealed to the NLRC. The NLRC granted
Notice of Strike on October 29, 1997 with the appeal and reversed the decision of the
the National Conciliation and Mediation LA. CMC then elevated the case to the
Board (NCMB), serving a copy thereof to Court of Appeals, but to no avail. Hence,
CMC. The Union alleged as grounds for the this petition.
projected strike the following acts of CMC:
(a) refusal to bargain; (b) coercion on ISSUE: Did the Union comply with all the
employees; and (c) interference/ restraint to requirements for a lawful strike?
self-organization.
RULING:

NO. The Union failed to comply with


A series of conferences was the second paragraph of Section 10, Rule
conducted before the NCMB (National XXII of the Omnibus Rules of the NLRC.
Capital Region), but no agreement was
reached. On November 20, 1997, the Union

102
| Labor Relations-2A

In National Federation of Labor v. strike vote, to give it ample time to prepare


NLRC, the Court enumerated the notices for the deployment of the requisite
required by Article 263 of the Labor Code personnel, including peace officers if need
and the Implementing Rules, which include be. Unless and until the NCMB is notified at
the 24-hour prior notice to the NCMB: least 24 hours of the unions decision to
conduct a strike vote, and the date, place,
and time thereof, the NCMB cannot
1) A notice of strike, with the required contents,
determine for itself whether to supervise a
should be filed with the DOLE, specifically the
strike vote meeting or not and insure its
Regional Branch of the NCMB, copy furnished
peaceful and regular conduct.
the employer of the union;
The failure of a union to comply with
2) A cooling-off period must be observed
the requirement of the giving of notice to the
between the filing of notice and the actual
NCMB at least 24 hours prior to the holding
execution of the strike thirty (30) days in case of
bargaining deadlock and fifteen (15) days in of a strike vote meeting will render the
case of unfair labor practice. However, in the subsequent strike staged by the union
case of union busting where the unions illegal.
existence is threatened, the cooling-off period
need not be observed. Conformably to Article 264 of the
Labor Code of the Philippines and Section 7,
4) Before a strike is actually commenced, a Rule XXII of the Omnibus Rules
strike vote should be taken by secret balloting, Implementing the Labor Code, no labor
with a 24-hour prior notice to NCMB. The organization shall declare a strike unless
decision to declare a strike requires the secret- supported by a majority vote of the members
ballot approval of majority of the total union of the union obtained by secret ballot in a
membership in the bargaining unit concerned. meeting called for that purpose. The
requirement is mandatory and the failure of
5) The result of the strike vote should be a union to comply therewith renders the
reported to the NCMB at least seven (7) days
strike illegal. The union is thus mandated to
before the intended strike or lockout, subject to
allege and prove compliance with the
the cooling-off period.
requirements of the law.

The requirement of giving notice of In this case, the Union failed to


the conduct of a strike vote to the NCMB at comply with the 24-hour prior notice
least 24 hours before the meeting for the requirement to the NCMB before it
said purpose is designed to (a) inform the conducted the alleged strike vote meeting
NCMB of the intent of the union to conduct a on November 10, 1997. Furthermore, the
strike vote; (b) give the NCMB ample time to Court agrees with the finding of the LA that
decide on whether or not there is a need to no secret balloting to strike was conducted
supervise the conduct of the strike vote to by the Union on November 10, 1997 at the
prevent any acts of violence and/or parking lot in front of the hospital, at the
irregularities attendant thereto; and (c) corner of Scout Magbanua Street and Panay
should the NCMB decide on its own initiative Avenue, Quezon City.
or upon the request of an interested party
including the employer, to supervise the

103
| Labor Relations-2A

78_Reonico

ISSUE

Sukhothai vs. CA Whether or not the strike staged by


the Union is illegal.
G.R. No. 150437, July 17, 2006

RULING
FACTS
YES. The Court finds the strike
Majority of the employees of illegal and the union officers who
Sukhothai Cuisine and Restaurant participated in the illegal strike and in the
(Sukhothai) organized into a union affiliated commission of illegal acts are declared to
with the Philippine Labor Alliance Council have lost their employment status. It is a
(PLAC-Sukhothai). Later on, the Union filed fact that at the time the strike was staged in
a Notice of Strike on the ground of unfair June 1999, voluntary arbitration between
labor practice such as harassment, fault- the parties was ongoing by virtue of the
finding and union-busting. Sukhothai and January 1999 Submission Agreement. The
the Union entered into an agreement that issue to be resolved under those
there will be no termination of employment proceedings pertained to the very same
during the pendency of the case but that issues stated in the Notice of Strike that is
management may issue memos for violation the commission of unfair labor practices.
of company policies. Until a strike Article 264 of the Labor Code provides: No
transpired where the parties submitted the strike or lockout shall be declared after
issue for voluntary arbitration. However, assumption of jurisdiction by the President
during the proceeding, Sukhothai dismissed or the Secretary or after certification or
a union members due to a quarrel with a co- submission of the dispute to compulsory or
employee, and for undisclosed reasons. voluntary arbitration or during the pendency
Hence, the Union’s Vice President and of cases involving the same grounds for the
other members staged a “wildcat strike” strike or lockout.
which is a strike that is unauthorized by the
Union. When a notice of strike was re-filed,
it turned into a “sit-down strike” and
The Court has held that strikes
thereafter, an “actual strike”. Sukhothai filed
staged in violation of agreements providing
a complaint for illegal strike. The LA ruled
for arbitration are illegal, since these
that the strike was illegal for failure to
agreements must be strictly adhered to and
comply with the mandatory requirements of
respected if their ends are to be achieved.
a lawful strike. The NLRC reversed the
The rationale of the prohibition under Article
decision due to Sukhothai’s violation of their
264 is that once jurisdiction over the labor
agreement that no union member will be
dispute has been properly acquired by
dismissed during the pendency of the case.
competent authority, that jurisdiction should
The CA, however, affirmed the decision of
not be interfered with by the application of
the NLRC. Hence, this present case.
the coercive processes of a strike. Indeed, it

104
| Labor Relations-2A

is among the chief policies of the State to the Union was fully aware that the
promote and emphasize the primacy of free arbitration proceedings were pending, good
collective bargaining and negotiations, faith cannot be invoked as a defense.
including voluntary arbitration, mediation, Hence, the Court declared that their strike
and conciliation, as modes of settling labor, was illegal.
or industrial disputes.

Furthermore, the dismissals of union


members, which allegedly triggered the
wildcat strike, are not sufficient grounds to
justify the radical recourse of the Union. The
questions that surround their dismissal are
connected to the alleged breach of the
"guarantee" by Sukhothai not to dismiss its
employees during the pendency of the
arbitration case, the very questions which
they also link to the other incidents of unfair
labor practices allegedly committed by
Sukhothai—these matters should have
been raised and resolved in the voluntary
arbitration proceedings that were
commenced precisely to address them.

On the other hand, if the Union


believed that the disciplinary measures had
nothing to do with the issues under
arbitration, then they should have availed of
the appropriate remedies under the Labor
Code, such as the institution of cases of
illegal dismissal or, by agreement of the
parties, the submission of the cases to the
grievance machinery of the CBA, if one is
available, so that they may be subjected to
separate voluntary arbitration proceedings,
or simply seek to terminate the pending
voluntary arbitration case and complete the
mandatory procedure for a lawful strike. The
Union should have availed themselves of
any of these alternative remedies instead of
resorting to a drastic and unlawful measure,
specifically, the holding a wildcat strike. And

105
| Labor Relations-2A

Reonico_Strkes and Lockouts YES. A strike, considered as the


most effective weapon of labor, is defined
as any temporary stoppage of work by the
Gold City Inc. vs. NLRC concerted action of employees as a result of
an industrial or labor dispute. A labor
G.R. No. 103560, July 6, 1995 dispute includes any controversy or matter
concerning terms or conditions of
employment or the association or
representation of persons in negotiating,
FACTS
fixing, maintaining, changing or arranging
Petitioner was engaged in the terms and conditions of employment,
stevedoring and arrastre services. Members regardless of whether or not the disputants
of Macajalar Labor Union who are stand in the proximate relation of employers
employees of the petitioner, gathered in and employees. Private respondents and
mass action to express their grievances their co-workers stopped working and held
regarding wages among others. Such strike the mass action on April 30, 1985 to press
paralyzed the operations at the port in for their wages and other benefits. What
Cagayan de Oro. On the same day, the transpired then was clearly a strike, for the
strikers filed individual notices of strike. cessation of work by concerted action
After failed conciliation conferences, resulted from a labor dispute. The Arbiter
petitioner filed a case for illegal strike. The correctly ruled that the strike was illegal for
NLRC issued a temporary restraining order failure to comply with the requirements of
wherein majority of the strikers returned to Article 264 (now Article 263) paragraphs (c)
work, while others continued their protest. and (f) of the Labor Code.
The notice of strike was withdrawn after
verifying that the strike was staged by the
minority of the labor union without the Furthermore, the individual notices
approval of the majority. The LA found the of strike filed by the workers did not conform
strike illegal contrary to Article 264 of the to the notice required by the law to be filed
Labor Code. The NLRC affirmed the since they were represented by a union
decision but noted that the action was more (MLU-FFW) which even had an existing
of a “protest action” than a strike. Hence, collective bargaining agreement with
the petition for certiorari. petitioner. Neither did the striking workers
observe the strike vote by secret ballot,
cooling-off period and reporting
ISSUE requirements.

Whether or not participants were


engaged in an illegal strike.
On the other hand, the effects of
such illegal strikes, outlined in Article 265
(now Article 264) of the Labor Code, make a
RULING distinction between workers and union
officers who participate therein.

106
| Labor Relations-2A

losses, closing or cessation of operation of


the establishment, or in case the employee
A union officer who knowingly was found to have been suffering from a
participates in an illegal strike and any disease such that his continued
worker or union officer who knowingly employment is prohibited by law.
participates in the commission of illegal acts
during a strike may be declared to have lost
their employment status. An ordinary
striking worker cannot be terminated for Backwages, on the other hand, is a
mere participation in an illegal strike. There form of relief that restores the income that
must be proof that he committed illegal acts was lost by reason of unlawful dismissal. It
during a strike. A union officer, on the other is clear that there must generally be unjust
hand, may be terminated from work when or illegal dismissal from work, before
he knowingly participates in an illegal strike, reinstatement and backwages may be
and like other workers, when he commits an granted. And in cases where reinstatement
illegal act during a strike. In this case, is not possible or when dismissal is due to
petitioner accepted the majority of the valid causes, separation pay may be
striking workers, including union officers, granted.
back to work. Private respondents were left
to continue with the strike after they refused
to submit to the "screening" required by the Private respondents contend that
company. they were terminated for failure to submit to
the controversial "screening" requirement.
The Court finds that private respondents
The question to be resolved now is were indeed dismissed when petitioner
what these remaining strikers are entitled to refused to accept them back to work after
receive under the law, if any. An employee the former refused to submit to the
is entitled to reinstatement and to his full "screening" process. However, considering
backwages when he is unjustly dismissed. that a decade has already lapsed from the
time the disputed strike occurred, the award
separation pay in lieu of reinstatement
would be more practical and appropriate.
Reinstatement means restoration to No backwages will be awarded to private
a state or condition from which one had respondent-union members as a penalty for
been removed or separated. Reinstatement their participation in the illegal strike. Their
and backwages are separate and distinct continued participation in said strike, even
reliefs given to an illegally dismissed after most of their co-workers had returned
employee. Separation pay is awarded when to work, can hardly be rewarded by such an
reinstatement is not possible, due, for award.
instance, to strained relations between
employer and employee. It is also given as
a form of financial assistance when a
worker is dismissed in cases such as the
installation of labor saving devices,
redundancy, retrenchment to prevent

107
| Labor Relations-2A

79_Terre The LA dismissed the complaint, which the


NLRC affirmed.
UFE v. Nestle

ISSUE: W/N the strikes were illegal.


FACTS: UFE filed a notice of strike before
the Bureau of Labor Relations against
Nestle for unfair labor practice for violation
HELD: YES. UFE seeks to absolve itself by
of Labor Code provisions on holiday pay
pointing out qualifying factors such as
and non-implementation of CBA provisions.
motives, good faith, absence of findings on
Nestle filed a petition seeking assumption of
specific participation and/or liability, and
jurisdiction or to undergo compulsory
limiting the no-strike provision to economic
arbitration, thus Labor Minister Blas Ople
strikes. UFE completely misses the
assumed jurisdiction and issued an order
underlying principle embodied in Art. 264(g)
enjoining any strike, lockout, or any
on the settlement of labor disputes and this
concerted activity. Despite this, the union
is, that assumption and certification orders
members staged a strike at all factories and
are executory in character and are to be
warehouses of Nestle in December 1985.
strictly complied with by the parties even
Nestle filed a petition to declare the strike
during the pendency of any petition
illegal; Ople issued another order, directing
questioning their validity.
all strikers to return to work within 48 hours,
but still they refused to do so. Nestle then
filed criminal charges against them, while
the new Labor Minister Sanchez ordered Regardless therefore of their motives, or the
the workers to return to their work, which validity of their claims, the striking workers
they did in March 1986. Sanchez also must cease and/or desist from any and all
dismissed UFE’s complaint for illegal acts that tend to, or undermine this authority
dismissal for lack of merit. Meanwhile, the of the Secretary of Labor, once an
Labor Arbiter after trial on the merits assumption and/or certification order is
declared the strike illegal and found the issued. They cannot, for instance, ignore
Union guilty of unfair labor practice. return-to-work orders, citing unfair labor
practices on the part of the company, to
justify their actions.

Throughout August 1986 to September


1987, several strikes have been organized
by the UFE in the Cabuyao and Alabang An assumption and/or certification order of
factories without notices of strike and in the Secretary of Labor automatically results
defiance of the orders released by the in a return-to-work of all striking workers,
Minister of Labor. Nestle sent letters of whether or not a corresponding order has
termination to the union members for illegal been issued by the Secretary of Labor.
strike and other illegal acts while UFE filed a Thus, the striking workers erred when they
complaint for illegal dismissal and ULP. continued with their strike alleging absence
of a return-to-work order. Article 264(g) is
clear. Once an assumption/certification

108
| Labor Relations-2A

order is issued, strikes are enjoined, or if 80_Santos


one has already taken place, all strikers
shall immediately return to work. INTERPHIL EMPLOYEES V. INTERPHIL
G. R. NO. 142824, DEC. 19, 2001

A strike that is undertaken despite the FACTS:


issuance by the Secretary of Labor of an
assumption or certification order becomes a
Interphil Laboratories Employees Union-
prohibited activity and thus illegal, pursuant
FFW is the sole and exclusive bargaining
to the second paragraph of Art. 264 of the agent of the rank- and- file employees of
Labor Code as amended. The Union Interphil Laboratories, Inc- a company
officers and members, as a result, are engaged in the business of manufacturing
deemed to have lost their employment and packaging pharmaceutical products.
status for having knowingly participated in They had a Collective Bargaining
an illegal act. Agreement (CBA) effective from August 01,
1990 to July 31, 1993.

Prior to the expiration of the CBA,


Allesandro G. Salazar, the Vice President of
the Human Resources Department of the
respondent company was approached by
Nestor Ocampo, the union president and
Hernando Clemente. Salazar told the union
officers that the matter could be discussed
during formal negotiations. On March 1993,
the union officers again approached
Salazar. They required once more about the
CBA status and received the same reply
from Salazar. Ocampo requested for a
meeting to discuss the duration and
effectivity of CBA. Salazar however,
declared that it would still be premature to
discuss the matter and that the company
could not make a decision at a moment.
The following day all the rank- and file
employees refused to follow their regular
shift work schedule: From 6:00 am to 6:00
pm /6:00 pm to 6:00 am; 2:00 pm to 2:00
am

The employees stopped working and left


their workplace without sealing the
containers and securing the raw materials
they were working on. To minimize the
damage the overtime boycott was causing
the company, Salazar immediately asked
for the meeting with the union officers.
Gonzales told Salazar that the employees
will return to their normal work schedule if

109
| Labor Relations-2A

the company would agree to their demands Whether or not the overtime boycott
as to the effectivity and duration of the new or work slowdown” by the employees
CBA. The agreement must be effective for 2 constitutes a violation of the CBA which
years. Again, Salazar told the union officers
prohibits the union to stage a strike or
that the matter could be discussed during
formal negotiations unsatisfied with the engage in slowdown or interruption of work.
answer the employees started to engage in
a work slowdown campaign to delay the
production of the company.
HELD:
On September 1993, respondent YES. It’s clear that the actions taken
company filed with NLRC a petition to
declare illegal petitioner union’s “overtime by the union amounted to illegal strike. The
boycott” and work slowdown” . It amounted SC classified the acts by the union as a
to illegal strike. October 1993, respondent “strike on installment basis.” “Slowdown” is
company filed with National Conciliation and an inherently illegal activity wherein the
Mediation Board (NCMB) an urgent request workers purposefully remain at their
for preventive mediation aimed to help the positions and accept wages, but at the
parties in their CBA negotiations. On
same time, select what parts of their allotted
January 1994, petitioner union filed with the
NCMB a “Notice of Strike” citing unfair labor task they want to perform, and refuse to do
allegedly committed by the respondent other work
company. Thereafter a month, Secretary of
Labor Nieves Confessor issued an order
directing respondent company to a).
Immediately accept all striking workers, In the instant case, since there is a
including the 53 terminated union officers; contractual commitment “that there shall be
b). Shop stewards and union members back no strikes, walkouts, stoppage or slowdown
to work under the same terms and of work, boycotts or secondary boycotts x x
conditions; and c). To pay all the unpaid x or any interference with any of the
accrued year end benefits of its employees. operations of the company during CBA. The
workers’ refusal to adhere to the work
On the other hand, petitioner union was
directed to “strictly and immediately comply schedule in force is a slowdown and it is
with the return-to work order”. By inherently illegal activity essentially illegal
September 1995, Secretary Quisumbing even in the absence of a no-strike clause in
approved and adopted the decision of the a CBA. The court also agrees that such
Labor Arbiter Caday declaring the overtime slowdown is generally condemned as
boycott and work slowdown as illegal strike
inherently illicit and unjustifiable because
and found out that the respondent company
is guilty of unfair labor practice. while the employees “continue to work and
remain at their positions and accept the
Petitioner union’s reconsideration and wages paid to them. They select what part
petition for certiorari were denied. Hence, of their allotted tasks they care to perform.
this petition. In other words, they work on their own
terms.

ISSUE:

110
| Labor Relations-2A

81 THE NLRC GRAVELY ABUSED ITS


DISCRETION IN ALLOWING THE TRO TO
SAN MIGUEL CORPORATION, petitioner, vs. LAPSE WITHOUT RESOLVING THE PRAYER
NATIONAL LABOR RELATIONS FOR INJUNCTION, DENYING INJUNCTION
COMMISSION, Second Division, ILAW AT WITHOUT EXPRESSING THE FACTS AND
BUKLOD NG MANGGAGAWA THE LAW ON WHICH IT IS BASED AND
(IBM), respondents. ISSUING ITS DENIAL FIVE MONTHS AFTER
[25]
THE LAPSE OF THE TRO.
DECISION
(b) We find for the petitioner.
(a) Petitioner San Miguel Corporation (SMC)
and respondent Ilaw at Buklod ng (c) Article 254 of the Labor Code provides that
Manggagawa (IBM), exclusive bargaining no temporary or permanent injunction or
agent of petitioners daily-paid rank and file restraining order in any case involving or
employees, executed a Collective growing out of labor disputes shall be issued
Bargaining Agreement (CBA) under which by any court or other entity except as
they agreed to submit all disputes to otherwise provided in Articles 218 and 264
grievance and arbitration proceedings. The of the Labor Code. Under the first exception,
CBA also included a mutually enforceable Article 218 (e) of the Labor Code expressly
no-strike no-lockout agreement. The confers upon the NLRC the power to enjoin
pertinent provisions of the said CBA are or restrain actual and threatened
quoted hereunder: commission of any or all prohibited or
unlawful acts, or to require the performance
A perusal of the records of the case clearly of a particular act in any labor dispute which,
shows that the basic point to be resolved entails if not restrained or performed forthwith, may
the question of as to who between the two (2) cause grave or irreparable damage to any
groups shall represent the workers for collective party or render ineffectual any decision in
bargaining purposes, which has been the favor of such party x x x. The second
subject of a Petition for Interpleader case exception, on the other hand, is when the
pending resolution before the Office of the labor organization or the employer engages
Secretary of Labor and Employment. Similarly, in any of the prohibited activities
the other issues raised which have been enumerated in Article 264.
discussed by the parties at the plant level, are
ancillary issues to the main question, that is, the (d) Pursuant to Article 218 (e), the coercive
[12]
union leadership... (Emphasis supplied) measure of injunction may also be used to
restrain an actual or threatened unlawful
Aggrieved by public respondents denial of a strike. In the case of San Miguel Corporation
permanent injunction, petitioner contends that: [26]
v. NLRC, where the same issue of NLRCs
duty to enjoin an unlawful strike was raised,
A. we ruled that the NLRC committed grave
abuse of discretion when it denied the
THE NLRC GRAVELY ABUSED ITS
petition for injunction to restrain the union
DISCRETION WHEN IT FAILED TO ENFORCE,
from declaring a strike based on non-
BY INJUNCTION, THE PARTIES RECIPROCAL
strikeable grounds. Further, in IBM v.
OBLIGATIONS TO SUBMIT TO ARBITRATION [27]
NLRC, we held that it is the legal duty and
AND NOT TO STRIKE.
obligation of the NLRC to enjoin a partial
B. strike staged in violation of the law. Failure
promptly to issue an injunction by the public
THE NLRC GRAVELY ABUSED ITS respondent was likewise held therein to be
DISCRETION IN WITHHOLDING INJUNCTION an abuse of discretion.
WHICH IS THE ONLY IMMEDIATE AND
EFFECTIVE SUBSTITUTE FOR THE (e) In the case at bar, petitioner sought a
DISASTROUS ECONOMIC WARFARE THAT permanent injunction to enjoin the
ARBITRATION IS DESIGNED TO AVOID. respondents strike. A strike is considered as
the most effective weapon in protecting the
C. rights of the employees to improve the terms

111
| Labor Relations-2A

and conditions of their employment. similarly defied a prohibition by the NCMB,


However, to be valid, a strike must be we said:
[28]
pursued within legal bounds. One of the
procedural requisites that Article 263 of the Petitioners should have complied with the
Labor Code and its Implementing Rules prohibition to strike ordered by the NCMB when
prescribe is the filing of a valid notice of the latter dismissed the notices of strike after
strike with the NCMB. Imposed for the finding that the alleged acts of discrimination of
purpose of encouraging the voluntary the hotel were not ULP, hence not strikeable.
[29]
settlement of disputes, this requirement The refusal of the petitioners to heed said
has been held to be mandatory, the lack of proscription of the NCMB is reflective of bad
[30]
which shall render a strike illegal. faith.

(f) In the present case, NCMB converted IBMs (h) Such disregard of the mediation
notices into preventive mediation as it found proceedings was a blatant violation of the
that the real issues raised are non- Implementing Rules, which explicitly oblige
strikeable. Such order is in pursuance of the the parties to bargain collectively in good
NCMBs duty to exert all efforts at mediation faith and prohibit them from impeding or
[36]
and conciliation to enable the parties to disrupting the proceedings.
[31]
settle the dispute amicably, and in line
with the state policy of favoring voluntary (i) The NCMB having no coercive powers of
modes of settling labor disputes. In
[32] injunction, petitioner sought recourse from
accordance with the Implementing Rules of the public respondent. The NLRC issued a
the Labor Code, the said conversion has the TRO only for free ingress to and egress from
effect of dismissing the notices of strike filed petitioners plants, but did not enjoin the
[33]
by respondent. A case in point is PAL v. unlawful strike itself. It ignored the fatal lack
[34]
Drilon, where we declared a strike illegal of notice of strike, and five months after
for lack of a valid notice of strike, in view of came out with a decision summarily
the NCMBs conversion of the notice therein rejecting petitioners cited jurisprudence in
into a preventive mediation case. We ruled, this wise:
thus:
Complainants scholarly and impressive
The NCMB had declared the notice of strike as arguments, formidably supported by a long line
appropriate for preventive mediation. The effect of jurisprudence cannot however be
of that declaration (which PALEA did not ask to appropriately considered in the favorable
be reconsidered or set aside) was to drop the resolution of the instant case for the
case from the docket of notice of strikes, as complainant. The cited jurisprudence do not
provided in Rule 41 of the NCMB Rules, as if squarely cover and apply in this case, as they
there was no notice of strike. During the are not similarly situated and the remedy sought
[37]
pendency of preventive mediation proceedings for were different.
no strike could be legally declared... The strike
which the union mounted, while preventive (j) Unfortunately, the NLRC decision stated no
mediation proceedings were ongoing, was aptly reason to substantiate the above conclusion.
described by the petitioner as an ambush.
(k) Public respondent, in its decision, moreover
(Emphasis supplied)
ruled that there was a lack of factual basis in
(g) Clearly, therefore, applying the aforecited issuing the injunction. Contrary to the
ruling to the case at bar, when the NCMB NLRCs finding, we find that at the time the
ordered the preventive mediation on May 2, injunction was being sought, there existed a
1994, respondent had thereupon lost the threat to revive the unlawful strike as
notices of strike it had filed. Subsequently, evidenced by the flyers then being circulated
however, it still defiantly proceeded with the by the IBM-NCR Council which led the
strike while mediation was ongoing, and union. These flyers categorically
notwithstanding the letter-advisories of declared: Ipaalala nyo sa management na
NCMB warning it of its lack of notice of hindi iniaatras ang ating Notice of Strike
strike. In the case of NUWHRAIN v. (NOS) at anumang oras ay pwede nating
[38]
[35]
NLRC, where the petitioner-union therein muling itirik ang picket line. These flyers
were not denied by respondent, and were

112
| Labor Relations-2A

dated June 19, 1994, just a day after the to and respected if their ends have to be
[41]
unions manifestation with the NLRC that achieved.
there existed no threat of commission of
prohibited activities. (o) As to petitioners allegation of violation of the
no-strike provision in the CBA, jurisprudence
(l) Moreover, it bears stressing that Article has enunciated that such clauses only bar
[39]
264(a) of the Labor Code explicitly states strikes which are economic in nature, but not
that a declaration of strike without first strikes grounded on unfair labor
[42]
having filed the required notice is a practices. The notices filed in the case at
prohibited activity, which may be prevented bar alleged unfair labor practices, the initial
through an injunction in accordance with determination of which would entail fact-
Article 254. Clearly, public respondent finding that is best left for the labor arbiters.
should have granted the injunctive relief to Nevertheless, our finding herein of the
prevent the grave damage brought about by invalidity of the notices of strike dispenses
the unlawful strike. with the need to discuss this issue.

(m) Also noteworthy is public respondents (p) We cannot sanction the respondent-unions
disregard of petitioners argument pointing brazen disregard of legal requirements
out the unions failure to observe the CBA imposed purposely to carry out the state
provisions on grievance and arbitration. In policy of promoting voluntary modes of
the case of San Miguel Corp. v. settling disputes. The states commitment
[40]
NLRC, we ruled that the union therein to enforce mutual compliance therewith to
violated the mandatory provisions of the foster industrial peace is affirmed by no less
[43]
CBA when it filed a notice of strike without than our Constitution. Trade unionism and
availing of the remedies prescribed therein. strikes are legitimate weapons of labor
Thus we held: granted by our statutes. But misuse of these
instruments can be the subject of judicial
x x x For failing to exhaust all steps in the intervention to forestall grave injury to a
[44]
grievance machinery and arbitration business enterprise.
proceedings provided in the Collective
Bargaining Agreement, the notice of strike (q) WHEREFORE, the instant petition is hereby
should have been dismissed by the NLRC and GRANTED. The decision and resolution of
private respondent union ordered to proceed the NLRC in Injunction Case No. 00468-94
with the grievance and arbitration proceedings. are REVERSED and SET ASIDE. Petitioner
In the case of Liberal Labor Union vs. Phil. and private respondent are hereby directed
Can Co., the court declared as illegal the strike to submit the issues raised in the dismissed
staged by the union for not complying with the notices of strike to grievance procedure and
grievance procedure provided in the collective proceed with arbitration proceedings as
bargaining agreement. . . (Citations omitted) prescribed in their CBA, if necessary. No
pronouncement as to costs.
(n) As in the abovecited case, petitioner herein
evinced its willingness to negotiate with the
union by seeking for an order from the
NLRC to compel observance of the
grievance and arbitration proceedings.
Respondent however resorted to force
without exhausting all available means
within its reach. Such infringement of the
aforecited CBA provisions constitutes further
justification for the issuance of an injunction
against the strike. As we said long ago:
Strikes held in violation of the terms
contained in a collective bargaining
agreement are illegal especially when they
provide for conclusive arbitration clauses.
These agreements must be strictly adhered

113
| Labor Relations-2A

82_PALACIOS granted, the Union later refused to sign the


agreement.

The Management circulated a


PERFECTO FERRER v. COURT OF Memorandum and distributed it to the
INDUSTRIAL RELATIONS members of the union, alleging that the
FACTS: Union refused to sign the CBA despite of it
yielded to the demand for an increase in the
After an election held, which high cost of living allowance and the
Inhelder Laboratories Inc. and Sister creation of a grievance committee. Soon
Companies Employees Union obtained the thereafter, several members of the Union
requisites majority, the Union submitted to resigned in view of the apparent failure of
the Management of said corporations a their Union officer to enter into a working
set of demands for a collective agreement with the Management. With that,
bargaining agreement (first draft). This the petitioners believed that said
led to negotiations, held, sometimes, with resignations had been inspired by the
the intervention of the Conciliation Division Management, and that
of the Bureau of Labor, and, sometimes, the latter had resorted thereto in order to
directly, between the representatives of the exert pressure upon the Union and compel
parties, and lasting for several weeks. the same to sign the draft of agreement
without the said union shop or union
In a meeting held before said security clause. Hence, the petitioners filed
Conciliation Division a draft was drawn, to a 30-day notice of strike upon the ground
which the Management refers as "final that respondents had been "bargaining in
draft"(second draft). However, petitioners' bad faith". However, the Management, in
representatives pressed for the inclusion, in turn, filed unfair labor practice charges
the agreement, of a union clause, an against the Union, for alleged refusal to
accumulated sick leave clause, and an bargain and alleged that the strike is illegal
accumulated vacation leave clause, apart on the ground of bad faith of the Union.
from the increase of the high cost of living
monthly allowance from P20.00 to P30.00, ISSUE: Is the strike illegal?
the creation of a grievance committee and a
general salary increase. HELD: NO

The negotiations continued, in the The contention of the Management


course of which the Management agreed to that there was already a perfected contract
increase the high cost of living allowance to and that the Union’s refusal to sign the
P25.00, provided that the other demands contract constitute unfair labor practice and
were withdrawn, to which petitioners bad faith on the part of the Union is without
allegedly gave their conformity. Another merit. The draft of agreement made and
draft of agreement, which the amended had been initialed by
representatives of Management, again, representatives of both parties and that the
characterizes as "final" (third draft) was Management refers thereto as a "final draft"
prepared, and the representatives of both (1). However, subsequent draft is likewise,
parties initialed it, with the understanding called by the Management as a "final" draft
that the agreement would be signed on (2). Yet, negotiations between the parties
June 1, 1963. However, petitioners' continued after the preparation of said
representative asked for the inclusion in the "final" draft, thus indicating it as not an
agreement a union shop or union security expressive of a complete, definite and
clause. This request not having been perfected agreement with the petitioners, for
otherwise, the Management would not have

114
| Labor Relations-2A

participated in the negotiations that took 83


place or continued thereafter. Hence, the so
called “final draft” was no more than a draft Malayang Samahan ng mga
of contract not a contract itself. Manggagawa sa M. Greenfield v. Ramos

The surrounding circumstances GR NO. 113907


were such that petitioners were reasonably February 28, 2000
justified in believing that the respondents'
acts constituted unfair labor practices and Facts:
that petitioners had to strike forthwith in
order to arrest the evil effects of said The petitioner, Malayang Samahan
practices upon the Union and its members. ng mga Manggagawa sa M. Greenfield,
The Court held that the strike in question Inc., (B) (MSMG), hereinafter referred to as
had been called to offset what petitioners the "local union", is an affiliate of the private
were warranted in believing in good faith to respondent, United Lumber and General
be unfair labor practices on the part of Workers of the Philippines (ULGWP),
Management, that petitioners were not referred to as the "federation". A local union
bound, therefore, to wait for the expiration of election was held under the auspices of the
thirty (30) days from notice of strike before ULGWP wherein the herein petitioner, Beda
staging the same, that said strike was not, Magdalena Villanueva, and the other union
accordingly, illegal and that the strikers had officers were proclaimed as winners.
not thereby lost their status as employees of Petition for Impeachment was filed with the
respondents herein. national federation ULGWP by the defeated
candidates in the aforementioned election.
The federation conducted an audit of the
local union funds. The investigation did not
yield any unfavorable result and the local
union officers were cleared of the charges
of anomaly in the custody, handling and
disposition of the union funds. The defeated
candidates filed a Petition for
Impeachment/Expulsion of the local union
officers with the DOLE NCR but were
dismissed for failure to substantiate the
charges and to present evidence. The local
union held a general membership meeting
at the Caruncho Complex in Pasig. Several
union members failed to attend the meeting,
prompting the Executive Board to create a
committee tasked to investigate the non-
attendance of several union members in the
said assembly, pursuant to Sections 4 and
5, Article V of the Constitution and By-Laws
of the union. The local union wrote
respondent company a letter requesting it to
deduct the union fines from the
wages/salaries of those union members
who failed to attend the general
membership meeting. The Secretary
General of the national federation,
Godofredo Paceño, Jr. disapproved the

115
| Labor Relations-2A

resolution of the local union imposing the company informing the latter of its
P50.00 fine. The union officers protested designation of a certain Alfredo Kalingking
such action by the Federation. The as local union president and "disauthorizing"
Federation wrote respondent company a the incumbent union officers from
letter advising the latter not to deduct the representing the employees. Petitioner
fifty-peso fine from the salaries of the union union officers received identical letters from
members requesting that:. . . any and all the administrator requiring them to explain
future representations by MSMG affecting a within 72 hours why they should not be
number of members be first cleared from removed from their office and expelled from
the federation before corresponding action union membership. The officers were
by the Company thereafter expelled from ULGWP. The
federation advised respondent company of
Respondent company sent a reply to the expulsion of the 30 union officers and
petitioner union's request in a letter, stating demanded their separation from
that it cannot deduct fines from the employment pursuant to the Union Security
employees' salary without going against Clause in their collective bargaining
certain laws. The company suggested that agreement.
the union refer the matter to the proper
government office for resolution in order to Thereafter, the Federation filed a Notice of
avoid placing the company in the middle of Strike with the National Conciliation and
the issue. The imposition of P50.00 fine Mediation Board to compel the company to
became the subject of bitter disagreement effect the immediate termination of the
between the Federation and the local union expelled union officers. Under the pressure
culminating in the latter's declaration of of a threatened strike, respondent company
general autonomy from the former through terminated the 30 union officers from
Resolution No. 10 passed by the local employment, serving them identical copies
executive board and ratified by the general of the termination letter. The expelled union
membership officers assigned in the first shift were
physically or bodily brought out of the
In retaliation, the national federation asked company premises by the company's
respondent company to stop the remittance security guards. Likewise, those assigned to
of the local union's share in the education the second shift were not allowed to report
funds effective August 1988. This was for work. This provoked some of the
objected to by the local union which members of the local union to demonstrate
demanded that the education fund be their protest for the dismissal of the said
remitted to it in full. union officers. Some union members left
their work posts and walked out of the
The company was thus constrained to file a company premises. The Federation, having
Complaint for Interpleader with a Petition for achieved its objective, withdrew the Notice
Declaratory Relief with the Med-Arbitration of Strike filed with the NCMB. Petitioners
Branch of the Department of Labor and filed a Notice of Strike with the NCMB,
Employment where it was resolved in favor DOLE, Manila, docketed as Case No.
of the company. The resolution was later NCMB-NCR-NS-03-216-89, alleging the
modified. following grounds for the strike:
Officials of ULGWP called a Special (a) Discrimination
National Executive Board Meeting at
Nasipit, Agusan del Norte where a (b) Interference in union activities
Resolution was passed placing the MSMG
under trusteeship and appointing (c) Mass dismissal of union officers and
respondent Cesar Clarete as administrator. shop stewards
The administrator wrote the respondent

116
| Labor Relations-2A

(d) Threats, coercion and intimidation Parañaque expired and were not renewed.
Upon demand of the owners of the
(e) Union busting premises, the company was compelled to
vacate its office and factory.
A strike vote referendum was conducted
and out of 2, 103 union members who cast Thereafter, the company transferred its
their votes, 2,086 members voted to declare administration and account/client servicing
a strike. The thirty (30) dismissed union department at AFP-RSBS Industrial Park in
officers filed an urgent petition with the Taguig, Metro Manila. For failure to find a
Office of the Secretary of the Department of suitable place in Metro Manila for relocation
Labor and Employment praying for the of its factory and manufacturing operations,
suspension of the effects of their termination the company was constrained to move the
from employment. However, the petition said departments to Tacloban, Leyte. The
was dismissed by then Secretary Franklin complaint for unfair labor practice was
Drilon. assigned to Labor Arbiter Manuel Asuncion
but was thereafter reassigned to Labor
A total of 78 union shop stewards were Arbiter Cresencio Ramos when respondents
placed under preventive suspension by moved to inhibit him from acting on the
respondent company. This prompted the case. Finding the termination to be valid in
union members to again stage a walk-out compliance with the union security clause of
and resulted in the official declaration of the collective bargaining agreement, Labor
strike. The strike was attended with Arbiter Cresencio Ramos dismissed the
violence, force and intimidation on both complaint. The First Division affirmed the
sides resulting to physical injuries to several Labor Arbiter’s disposition.
employees, both striking and non-striking,
and damage to company properties. Isssue:
The employees who participated in the WON the strike was illegal.
strike and allegedly figured in the violent
incident were placed under preventive Held:
suspension by respondent company. The
company also sent return-to-work notices to This ruling of the NLRC is erroneous
the home addresses of the striking
employees thrice successively, on March With regard to the issue of the legality or
27, April 8 and April 31, 1989, respectively. illegality of the strike, the Labor Arbiter held
However, respondent company admitted that the strike was illegal for the following
that only 261 employees were eventually reasons: (1) it was based on an intra-union
accepted back to work. Those who did not dispute which cannot properly be the
respond to the return-to-work notice were subject of a strike, the right to strike being
sent termination letters . limited to cases of bargaining deadlocks
and unfair labor practice (2) it was made in
Petitioners filed a complaint charging private violation of the "no strike, no lock-out"
respondents of unfair labor practice which clause in the CBA, and (3) it was attended
consists of union busting, illegal dismissal, with violence, force and intimidation upon
illegal suspension, interference in union the persons of the company officials, other
activities, discrimination, threats, employees reporting for work and third
intimidation, coercion, violence, and persons having legitimate business with the
oppression. company, resulting to serious physical
injuries to several employees and damage
After the filing of the complaint, the lease to company property.
contracts on the respondent company's
office and factory at Merville Subdivision,

117
| Labor Relations-2A

On the submission that the strike was illegal 84_ACOSTA


for being grounded on a non-strikeable
issue, that is, the intra-union conflict GRAND BOULEVARD HOTEL
between the federation and the local union, (FORMERLY KNOWN AS SILAHIS
it bears reiterating that when respondent INTERNATIONAL HOTEL, INC.) v
company dismissed the union officers, the GENUINE LABOR ORGANIZATION OF
issue was transformed into a termination WORKERS IN HOTEL, RESTAURANT
dispute and brought respondent company AND ALLIED INDUSTRIES (GLOWHRAIN)
into the picture. Petitioners believed in good G.R. NO. 153664 JULY 18, 2003
faith that in dismissing them upon request
by the federation, respondent company was GRAND BOULEVARD HOTEL v EDNA B.
guilty of unfair labor practice in that it DACANAY
violated the petitioner's right to self- G.R. NO. 153665 JULY 18, 2003
organization. The strike was staged to
protest respondent company's act of
dismissing the union officers. Even if the FACTS: GLOWHRAIN and petitioner GBH
allegations of unfair labor practice are executed a CBA. Later, petitioner dismissed
subsequently found out to be untrue, the some of its employees and suspended
presumption of legality of the strike prevails. others who were members of the
respondent union. The union filed a notice
A no strike, no lock out provision can only of strike with the NLRC alleging illegal
be invoked when the strike is economic in dismissal, illegal suspension, CBA
nature, i.e. to force wage or other violations, and harassment.
concessions from the employer which he is
not required by law to grant. Such a The acting SOLE issued as status quo ante
provision cannot be used to assail the bellum order certifying the labor dispute to
legality of a strike which is grounded on the NLRC0 for compulsory arbitration
unfair labor practice, as was the honest pursuant to Article 263 (g) of the Labor
belief of herein petitioners. Again, whether Code. Subsequently, the union filed another
or not there was indeed unfair labor practice notice of strike due to its belief that GBH
does not affect the strike. was committing ULP. The SOLE responded
by issuing another status quo ante bellum
On the allegation of violence committed in order. Further, it directed the employees to
the course of the strike, it must be return to work, enjoined the parties from
remembered that the Labor Arbiter and the engaging in strike or lockouts, and ordered
Commission found that "the parties are the parties to sign a new CBA.
agreed that there were violent incident
resulting to injuries on both the union and GBH, through its general manager, then
management." The evidence on record wrote to SOLE to inform him of the
show that the violence cannot be attributed company’s decision to retrench around 200
to the striking employees alone for the employees due to financial difficulties.
company itself employed hired men to Meanwhile, the union wrote to the SOLE
pacify the strikers. With violence committed that the union will conduct a strike vote
on both sides, the management and the referendum. This led to the issuance by the
employees, such violence cannot be a SOLE of another status quo ante bellum
ground for declaring the strike as illegal. order certifying the case to the NLRC for
compulsory arbitration and enjoining the
parties from engaging in any strike or
lockout. But another notice of strike was
filed by the union on September 27, 1990.

118
| Labor Relations-2A

The SOLE issued another certification of strike of September 27 had already been
order. taken cognizance of by the SOLE when he
issued a status quo ante bellum order
When GBH implemented its retrenchment enjoining the union from staging a strike.
program, the union filed a notice of strike on Thus, when the union went on strike despite
November 16, 1900 and staged a picket on the SOLE order, it violated of Article 264 of
the same day. The SOLE thus issued a the LC.
Cease and Desist order and ordered the
reinstatement of some of the employees. The union further claims that it went on
GBH subsequently filed a complaint for strike because of its belief in good faith that
illegal strike against the union with the the company was committing ULP. Still, the
NLRC. The LA found in favor of GBH, Court held that a claim of good faith is not a
finding that the union was guilty of illegal valid excuse to dispense with the procedural
strike and declaring the union officers to steps for lawful strike.
have forfeited their employment. The union
appealed to the NLRC, which later affirmed
the Labor Arbiter’s decision. However, on
certiorari, the CA reversed. Hence, this
petition.

ISSUE: Whether or not the strike staged by


the union is legal.

HELD: NO. Under Article 263, the requisites


for a valid strike are as follows: (a) a notice
of strike fled with the DOLE thirty days
before the intended date thereof or fifteen
days in case of ULP; (b) strike vote
approved by a majority of the total union
membership in the bargaining unit
concerned obtained by secret ballot in a
meeting called for that purpose; (c) notice
given to the DOLE of the results of the
voting at least seven days before the
intended strike.

In the case, the union went on strike


simultaneously with the filing of the notice of
strike. This rendered the strike illegal for
failure to comply with the requisite periods
as stated above. Moreover, a strike
undertaken despite the issuance by the
SOLE of certification order becomes a
prohibited activity and thus, illegal pursuant
to Article 264 (a) of the LC.

The union claimed that its November 16


notice merely reiterated its September 27
notice of strike, thus it claims that it
complied with the statutory procedural
requirements for lawful strike. The court
held that the matters contained in the notice

119
| Labor Relations-2A

85_BARRIETA
The University filed a Petition to Declare
UNIVERSITY OF SAN AGUSTIN v. CA Illegal Strike and Loss of Employment
G.R. NO. 169632, March 28, 2006 Status at NLRC. SOLE DISMISSED the
Garcia, J.: petition to declare the strike illegal for want
of legal and factual basis. The University
FACTS thereafter appeal the decision with the CA.
On July 27, 2000, the University and its
Union entered into a 5-year CBA. The CBA CA DECLARED the strike held on
contained a "no strike, no lockout" clause September 19, 2003 by the Union as
and a grievance machinery procedure to ILLEGAL and ordered that the union
resolve management-labor disputes, officers are therefore deemed to have lost
including a voluntary arbitration mechanism their employment status.
should the grievance committee fail to
satisfactorily settle such disputes. ISSUE
Whether or not the strike was illegal.
During the negotiations, the parties could
not agree on the manner of computing the HELD
Tuition Incremental Proceeds (TIP), thus the YES. To be sure, the Union was not able to
need to undergo preventive mediation sufficiently dispute the truth of the narration
proceedings before the National Conciliation of facts contained in the sheriff's report.
and Mediation Board (NCMB), Iloilo City. Hence, it was not unreasonable for the CA
to conclude that there was a deliberate
The impasse respecting the computation of intent by the Union and its officers to
TIP was not resolved. This development disregard the AJO and proceed with their
prompted the Union to declare a bargaining strike, which, by their act of disregarding
deadlock grounded on the parties' failure to said AJO made said strike illegal. The AJO
arrive at a mutually acceptable position on was issued by the SOLE pursuant to Article
the manner of computing the seventy 263(g) of the Labor Code, which reads:
percent (70%) of the net TIP to be allotted
for salary and other benefits for SY2003- Art. 263. Strikes, picketing, and
2004 and SY2004-2005. lockouts. - ... (g) When, in his opinion,
there exists a labor dispute causing or likely
Thereafter, the Union filed a Notice of Strike to cause a strike or lockout in an industry
before the NCMB. The parties then made a indispensable to the national interest, the
joint request for the SOLE to assume Secretary of Labor and Employment may
jurisdiction over the dispute. On September assume jurisdiction over the dispute and
19, 2003, the Union staged a strike. At decide it or certify the same to the
approximately 8:45 a.m., the sheriffs posted Commission for compulsory arbitration.
copies of the AJO at the main gate of San Such assumption or certification shall have
Agustin University, at the main entrance of the effect of automatically enjoining the
its buildings and at the Union's office inside intended or impending strike or lockout as
the campus. At 9:20 a.m., the sheriffs specified in the assumption or certification
served the AJO on the University. order. If one has already taken place at the
Notwithstanding the sheriffs' advice as to time of assumption or certification, all
the legal implication of the Union's refusal to striking or locked out employees shall
be served with the AJO, the Union went immediately return to work and the
ahead with the strike. At around 5:25 p.m., employer shall immediately resume
the Union president arrived at the operations and readmit all workers under
respondent University's premises and the same terms and conditions prevailing
received the AJO from the sheriffs. before the strike or lockout. The Secretary

120
| Labor Relations-2A

of Labor and Employment or the 263(g) of the Labor Code, supra, is explicit
Commission may seek the assistance of law that if a strike has already taken place at
enforcement agencies to ensure compliance the time of assumption of jurisdiction or
with this provision as well as with such certification, all striking or locked out
orders as he may issue to enforce the employees shall immediately return to work
same. (Emphasis supplied). and the employer shall immediately resume
operations and readmit all workers under
Conclusively, when the SOLE assumes the same terms and conditions prevailing
jurisdiction over a labor dispute in an before the strike or lock-out. This is
industry indispensable to national interest or compounded further by this Court's rulings
certifies the same to the NLRC for which have never interpreted the phrase
compulsory arbitration, such assumption or "immediately return to work" found in Article
certification shall have the effect of 263(g) to mean "within twenty four (24)
automatically enjoining the intended or hours." On the other hand, the tenor of
impending strike or lockout. Moreover, if these ponencias indicates an almost
one had already taken place, all striking instantaneous or automatic compliance for a
workers shall immediately return to work striker to return to work once an AJO has
and the employer shall immediately resume been duly served.
operations and readmit all workers under
the same terms and conditions prevailing Thus, we see no reversible error in the CA's
before the strike or lockout. In Trans-Asia finding that the strike of September 19,
Shipping Lines, Inc., et al. vs. CA, et al., the 2003 was illegal. Consequently, the Union
Court declared that when the Secretary officers were deemed to have lost their
exercises these powers, he is granted great employment status for having knowingly
breadth of discretion in order to find a participated in said illegal act.
solution to a labor dispute. The most
obvious of these powers is the automatic
enjoining of an impending strike or lockout
or the lifting thereof if one has already taken
place. Assumption of jurisdiction over a
labor dispute, or the certification of the
same to the NLRC for compulsory
arbitration, always co-exists with an order
for workers to return to work immediately
and for employers to readmit all workers
under the same terms and conditions
prevailing before the strike or lockout.

In this case, the AJO was served at 8:45


a.m. of September 19, 2003. The strikers
then should have returned to work
immediately. However, they persisted with
their refusal to receive the AJO and waited
for their union president to receive the same
at 5:25 p.m. The Union's assertion of a well
settled practice that the SOLE always gives
twenty-four hours (24) to the striking
workers within which to return to work,
offers no refuge. Aside from the fact that
this alleged well settled practice has no
basis in law and jurisprudence, Article

121
| Labor Relations-2A

86_BATALLER YES.
It is clear from Art. 263(g) that the moment
TELEFUNKEN SEMICONDUCTORS EMP the Secretary of Labor assumes jurisdiction
LOYEES UNION-FFW vs. CA, DOLE- over a labor dispute in an industry
SEC, and TEMIC TELEFUNKEN MICROE indispensable to national interest, such
LECTRONICS, (PHILS.), INC. assumption shall have the effect of
automatically enjoining the intended or
G.R. Nos. 143013-14, 18 December 2000 impending strike. It was not even necessary
for the Secretary of Labor to issue another
FACTS: order directing them to return to work. The
mere issuance of an assumption order by
The labor dispute started when the the Secretary of Labor automatically carries
Company and the Union reached a with it a return-to-work order, even if the
deadlock in their negotiations for a new directive to return to work is not expressly
collective bargaining agreement. The Union stated in the assumption order. However,
filed a Notice of Strike with the NCMB. The petitioners refused to acknowledge this
Acting DOLE Secretary intervened and directive of the Secretary of Labor thereby
assumed jurisdiction over the dispute and necessitating the issuance of another order
thus enjoined any strike or lockout, whether expressly directing the striking workers to
actual or intended, between the parties. cease and desist from their actual strike,
and to immediately return to work but which
Despite the assumption Order, the Union directive the herein petitioners opted to
struck. And also on despite the Notice of the ignore.
Return-to-Work Order, still some of the
striking workers refused to heed the order In this connection, Article 264 provides
and continued with their picket. The prohibited activities. One of which is that
Company then issued letters of termination “No strike or lock out shall be declared after
for cause to the workers who did not report the assumption of jurisdiction by the
back to work despite the Notice of President or the Secretary or after
Assumption and Return-to-Work Orders certification or submission of the dispute to
issued by the Acting DOLE-Sec. However, compulsory or voluntary arbitration”. The
the Acting DOLE Secretary issued another rationale of this prohibition is that once
Order directing the Company to reinstate all jurisdiction over the labor dispute has been
striking workers “except the Union Officers, properly acquired by the competent
shop stewards, and those with pending authority, that jurisdiction should not be
criminal charges, x x x” while the resolution interfered with by the application of the
of the legality of the strike was pending. The coercive processes of a strike. We have
strike was later declared illegal by the Sec. held in a number of cases that defiance to
the assumption and return-to-work orders of
the Secretary of Labor after he has
ISSUE: assumed jurisdiction is a valid ground for
loss of the employment status of any
Whether or not defiance to the assumption striking union officer or member.
and return-to-work orders of the Secretary
of Labor after assumption of jurisdiction is a
valid ground to terminate the employment of
striking members

HELD:

122
| Labor Relations-2A

87_CANAPE position that any return to work


order should be unconditional.
ST. SCHOLASTICA’S COLLEGE VS.
TORRES  The College manifested to
respondent Sec. that the union
210 SCRA 565 continued to defy his return to
work order.
GR. NO 100158 JUNE 29, 1992
 The College sent termination
letters to individual strikers and
FACTS: filed a complaint for illegal strike
against the union.

 The union moved for the


 The Union and College initiated enforcement of the return to work
negotiations for a first ever CBA order before the Sec.
which resulted in a deadlock and
prompted the union to file a  The Sec. issued an order directing
notice of strike with the DOLE. reinstatement of striking union
members and holding union
 Union declared a strike which officers responsible for the
paralyzed the operations of the violation of the return to work
College and public respondent order and were correspondingly
Sec. of Labor immediately terminated.
assumed jurisdiction over the
labor dispute.  Both parties moved for the partial
consideration of the return to
 Instead of returning to work, the work order.
union filed a motion for
reconsideration of the return to  Hence tis petition.
work order.

 The college sent individual letters


ISSUE:
to the striking employees
requiring them to return to work. WON striking union members,
In response union presented terminated for abandonment of work
demands, the most important of after failing to comply with the return to
which is the unconditional work order of Sec. of labor, reinstated.
acceptance back to work of the
striking employees. But these
were rejected.
HELD:
 Sec. of Labor denied the motion
for reconsideration for his return The Labor Code provides that if a
to work order and sternly warned strike has already taken place at the time
striking employees to comply with of assumption, all striking employees
its terms. should immediately return to work. This
means that a return to work order is
 Conciliation meetings were held immediately effective and executory,
but this proved futile as the notwithstanding the filing of a motion for
college remained steadfast in its reconsideration. It must be strictly
complied with even during the pendency

123
| Labor Relations-2A

of any petition questioning its validity. 88_CARILLO


After all, the assumption and/or
certification order issued in the exercise MANILA DIAMOND UNION vs CA
of the Sec.’s compulsive power of
arbitration and until set aside, must FACTS:
therefore be complied with immediately. The Union filed a PCE which was later on
The college correspondingly had dismissed. The Union then sent a letter to
every right to terminate the services of the Hotel informing it of its desire to
those who chose to disregard the return negotiate for a collective bargaining
to work order issued by the Sec. of agreement but the Hotel refused in view of
Labor in order to protect the interest of the earlier dismissed PCE. A Notice of
the students who form part of the youth Strike was thereafter filed with NCMB for
of the land. the Hotels alleged refusal to bargain and for
alleged acts of unfair labor practice. The
Union staged a strike against the Hotel. The
Hotel claims that the strike was illegal and it
had to dismiss some employees for their
participation. The Union, on the other hand,
accused the Hotel of illegally dismissing the
workers.

What is pertinent to this case, however, is


the Order issued by the then Secretary of
Labor and Employment Cresenciano B.
Trajano assuming jurisdiction over the labor
dispute. A Petition for Assumption of
Jurisdiction was filed by the Union.
Accordingly, the striking officers and
members of the Manila Diamond Hotel
Employees Union --- were directed to return
to work within 24 hours upon receipt of this
Order and the Hotel to accept them back
under the same terms and conditions
prevailing prior to the strike. The Union
received the aforesaid Order and its
members reported for work the next day.
The Hotel, however, refused to accept the
returning workers and instead filed a Motion
for Reconsideration of the Secretary’s
Order.

The then Acting Secretary of Labor Jose M.


Espaol, issued the disputed Order, which
modified the earlier one issued by Secretary
Trajano. Instead of an actual return to work,
Acting Secretary Espaol directed that the
strikers be reinstated only in the payroll. The
Union moved for the reconsideration of this
Order, but its motion was denied. Hence it
filed this petition.

124
| Labor Relations-2A

ISSUE: relationship between the striking employees


and management is no reason for payroll
W/N CA GRIEVIOUSLY ERRED IN reinstatement in lieu of actual reinstatement.
RULING THAT THE SECRETARY OF
LABOR’S UNAUTHORIZED ORDER OF This question is answered by the nature of
MERE PAYROLL REINSTATEMENT IS Article 263(g). As a general rule, the State
NOT GRAVE ABUSE OF DISCRETION encourages an environment wherein
employers and employees themselves must
HELD: deal with their problems in a manner that
mutually suits them best. This is the basic
Petition has merit. policy embodied in Article XIII, Section 3 of
CA based its decision on this Courts ruling the Constitution, which was further echoed
in University of Santo Tomas (UST) v. in Article 211 of the Labor Code. Hence, a
NLRC. There, the Secretary assumed voluntary, instead of compulsory, mode of
jurisdiction over the labor dispute between dispute settlement is the general rule.
striking teachers and the university. He However, Article 263, paragraph (g) of the
ordered the striking teachers to return to Labor Code, which allows the Secretary of
work and the university to accept them Labor to assume jurisdiction over a labor
under the same terms and conditions. dispute involving an industry indispensable
However, in a subsequent order, the NLRC to the national interest, provides an
provided payroll reinstatement for the exception:
striking teachers as an alternative remedy to
actual reinstatement. True, this Court held (g) When, in his opinion, there exists a labor
therein that the NLRC did not commit grave dispute causing or likely to cause a strike or
abuse of discretion in providing for the lockout in an industry indispensable to the
alternative remedy of payroll reinstatement. national interest, the Secretary of Labor and
This Court found that it was merely an error Employment may assume jurisdiction over
of judgment, which is not correctible by a the dispute and decide it or certify the same
special civil action for certiorari. The NLRC to the Commission for compulsory
was only trying its best to work out a arbitration. Such assumption or certification
satisfactory ad hoc solution to a festering shall have the effect of automatically
and serious problem. enjoining the intended or impending strike
or lockout as specified in the assumption or
However, this Court notes that certification order. If one has already taken
the UST ruling was made in the light of one place at the time of assumption or
very important fact: the teachers could not certification, all striking or locked out
be given back their academic assignments employees shall immediately return to work
since the order of the Secretary for them to and the employer shall immediately resume
return to work was given in the middle of the operations and readmit all workers under
first semester of the academic year. The the same terms and conditions prevailing
NLRC was, therefore, faced with a situation before the strike or lockout. x x x
where the striking teachers were entitled to
a return to work order, but the university This provision is viewed as an exercise of
could not immediately reinstate them since the police power of the State. A prolonged
it would be impracticable and detrimental to strike or lockout can be inimical to the
the students to change teachers at that national economy and, therefore, the
point in time. situation is imbued with public necessity and
involves the right of the State and the public
In the present case, there is no showing that to self-protection.
the facts called for payroll reinstatement as
an alternative remedy. A strained

125
| Labor Relations-2A

Under Article 263(g), all workers must 89_CASTRO


immediately return to work and all
employers must readmit all of them under
the same terms and conditions prevailing
before the strike or lockout. It is, therefore, UNIVERSITY OF IMMACULATE,
evident from the foregoing that the CONCEPCION, INC., v. SECRETARY OF
Secretary’s subsequent order for mere LABOR G.R. No. 151379. January 14,
payroll reinstatement constitutes grave 2005
abuse of discretion amounting to lack or FACTS:
excess of jurisdiction.
The case stemmed from the collective
bargaining negotiations between petitioner
University of Immaculate Concepcion, Inc.
(UNIVERSITY) and respondent The UIC
Teaching and Non-Teaching Personnel and
Employees Union (UNION). The UNION, as
the certified bargaining agent of all rank and
file employees of the UNIVERSITY,
submitted its collective bargaining
proposals. However, the inclusion or
exclusion of the Secretaries, Registrars,
Accounting Personnel, and Guidance
Counselors in the scope of the bargaining
unit remained unresolved. When the case
was submitted for voluntary arbitration, the
panel of voluntary arbitrators rendered a
decision excluding the above-mentioned
secretaries, registrars, chief of the
accounting department, cashiers and
guidance counselors from the coverage of
the bargaining unit.
Pending the UNION’s Motion for
Reconsideration, it filed a notice of strike
with the NCMB on the grounds of
bargaining deadlock and unfair labor
practice. During the thirty (30) day cooling-
off period, two union members were
dismissed by petitioner. Consequently, the
UNION went on strike. The Secretary of
Labor then issued an Order assuming
jurisdiction over the labor dispute and
ordered all workers to return to work within
24 hours and for Management to accept
them back under the same terms and
conditions prevailing prior to the strike. The
panel of voluntary arbitrators denied the
motion for reconsideration filed by the
UNION. Thereafter, the UNIVERSITY gave
the individual respondents two choices: to
resign from the UNION and remain

126
| Labor Relations-2A

employed as confidential employees or However, this privilege is not absolute, but


resign from their confidential positions and subject to exceptions. One of these is when
remain members of the UNION. The the Secretary of Labor assumes jurisdiction
UNIVERSITY said they could not remain as over labor disputes involving industries
confidential employees and at the same indispensable to the national interest under
time as members or officers of the Union. Article 263(g) of the Labor Code.
However, the respondents remained in their
claim that they could still retain their When the Secretary of Labor ordered
confidential positions while being members the UNIVERSITY to suspend the effect of
or officers of the Union. Hence, the the termination of the individual
UNIVERSITY sent notices of termination. respondents, the Secretary did not exceed
her jurisdiction, nor did the Secretary
The UNION filed another notice of gravely abuse the same. The rationale for
strike due to the termination. The UNION the exercise of the Secretary of Labor and
alleged that the UNIVERSITYs act of Employments power under Article 263(g) of
terminating the individual respondents is in the Labor Code is the maintenance and
violation of the Order of the Secretary of upholding of the status quo while the
Labor. The Secretary of Labor issued dispute is being adjudicated. Hence, the
another Order stating that the effects of the directive to the parties to refrain from
termination from employment of these performing acts that will exacerbate the
individual respondents be suspended situation is intended to ensure that the
pending the determination of the legality dispute does not get out of hand, thereby
thereof. Hence, the UNIVERSITY was negating the direct intervention of this office.
directed to reinstate the individual
respondents under the same terms and The University’s act of suspending and
conditions prevailing prior to the labor terminating union members and the Unions
dispute. The UNIVERSITY argued that the act of filing another Notice of Strike after this
Secretary’s Order directing the Office has assumed jurisdiction are certainly
reinstatement of the individual respondents in conflict with the status quo ante. The act
would render nugatory the decision of the of the UNIVERSITY of dismissing the
panel of voluntary arbitrators to exclude individual respondents from their
them from the collective bargaining unit. employment became the impetus for the
However, the Acting Secretary find that UNION to declare a second notice of strike.
superseding circumstances would not Any act committed during the pendency of
warrant the physical reinstatement of the the dispute that tends to give rise to further
twelve (12) terminated employees. Hence, contentious issues or increase the tensions
they are hereby ordered placed under between the parties should be considered
payroll reinstatement until the validity of an act of exacerbation and should not be
their termination is finally resolved. allowed. However, the final decision of the
panel of arbitrators as to the confidential
ISSUE:
nature of the positions of the twelve private
Whether or not the Secretary of Labor may respondents rendered actual and physical
legally order the employer to reinstate the reinstatement impracticable. The payroll
terminated employees even if they are not reinstatement in lieu of actual reinstatement
part of the bargaining unit. is proper.
RULING:
Yes. The Court recognizes the exercise
of management prerogatives and it often
declines to interfere with the legitimate
business decisions of the employer.

127
| Labor Relations-2A

90_DE LA ROSA-REID illegal strike. The Union filed with the NCMB
PASINTABI: Mahaba po ang susunod na a second Notice of Strike on the ground of
kaso. Mahabang pasensya at patnubay ng unfair labor practice and violation of Article
Maykapal ay kailangan. 248(a) of the Labor Code on illegal lockout.

NATIONAL UNION OF WORKERS VS On January 26, 2002, the Hotel


COURT OF APPEALS terminated the services of twenty-nine (29)
Union officers and sixty-one (61) members;
FACTS: The Union is the certified and suspended eighty-one (81) employees
bargaining agent of the regular rank-and-file for 30 days, forty-eight (48) employees for
employees of Dusit Hotel Nikko. On October 15 days, four (4) employees for 10 days,
24, 2000, the Union submitted its Collective and three (3) employees for five days. On
Bargaining Agreement (CBA) negotiation the same day, the Union declared a strike.
proposals to the Hotel. The parties failed to Starting that day, the Union engaged in
arrive at mutually acceptable terms and picketing the premises of the Hotel. During
conditions. Due to the bargaining deadlock, the picket, the Union officials and members
the Union, on December 20, 2001, filed a unlawfully blocked the ingress and egress of
Notice of Strike on the ground of the the Hotel premises.
bargaining deadlock. Conciliation was
proven to be unsuccessful. Consequently, a The Union filed its third Notice of
Strike Vote was conducted by the Union on Strike with the NCMB on the ground of
January 14, 2002 on which it was decided unfair labor practice and union-busting. The
that the Union would wage a strike. Secretary assumed jurisdiction over the
labor dispute and certified the case to the
The Union held a general assembly at its NLRC for compulsory arbitration.
office located in the Hotels basement,
where some members sported closely Pursuant to the Secretary’s Order, the
cropped hair or cleanly shaven heads. The Hotel, issued an Inter-Office Memorandum,
next day, more male Union members came directing some of the employees to return to
to work sporting the same hair style. The work, while advising others not to do so, as
Hotel prevented these workers from they were placed under payroll
entering the premises claiming that they reinstatement. Meanwhile, after due
violated the Hotels Grooming Standards. proceedings, the NLRC issued its Decision
ordering the Hotel and the Union to execute
In view of the Hotels action, the Union a CBA within 30 days from the receipt of the
staged a picket outside the Hotel premises. decision. The NLRC also held that the
Later, other workers were also prevented January 18, 2002 concerted action was an
from entering the Hotel causing them to join illegal strike in which illegal acts were
the picket. For this reason the Hotel committed by the Union; and that the strike
experienced a severe lack of manpower violated the No Strike, No Lockout provision
which forced them to temporarily cease of the CBA, which thereby caused the
operations in three restaurants. dismissal of 29 Union officers and 61 Union
members. The NLRC ordered the Hotel to
The Hotel issued notices to Union grant the 61 dismissed Union members
members, preventively suspending them financial assistance in the amount of
and charging them with the following month’s pay for every year of service or
offenses: (1) violation of the duty to bargain their retirement benefits under their
in good faith; (2) illegal picket; (3) unfair retirement plan whichever was higher. The
labor practice; (4) violation of the Hotels NLRC explained that the strike which
Grooming Standards; (5) illegal strike; and occurred on January 18, 2002 was illegal
(6) commission of illegal acts during the because it failed to comply with the

128
| Labor Relations-2A

mandatory 30-day cooling-off period and the crop their hair not only violated the Hotels
seven-day strike ban, as the strike occurred Grooming Standards but also violated the
only 29 days after the submission of the Unions duty and responsibility to bargain in
notice of strike on December 20, 2001 and good faith. By shaving their heads and
only four days after the submission of the cropping their hair, the Union officers and
strike vote on January 14, 2002. The NLRC members violated then Section 6, Rule XIII
also ruled that even if the Union had of the Implementing Rules of Book V of the
complied with the temporal requirements Labor Code.
mandated by law, the strike would
nonetheless be declared illegal because it Fourth, the Union failed to observe the
was attended by illegal acts committed by mandatory 30-day cooling-off period and the
the Union officers and members. seven-day strike ban before it conducted
the strike on January 18, 2002. The NLRC
ISSUE: Whether or not the union is liable correctly held that the Union failed to
for illegal strike observe the mandatory periods before
conducting or holding a strike. Records
HELD: Yes. First, the Unions violation of the reveal that the Union filed its Notice of
Hotels Grooming Standards was clearly a Strike on the ground of bargaining deadlock
deliberate and concerted action to on December 20, 2001. The 30-day cooling-
undermine the authority of and to off period should have been until January
embarrass the Hotel and was, therefore, not 19, 2002. On top of that, the strike vote was
a protected action. In view of the Unions held on January 14, 2002 and was
collaborative effort to violate the Hotels submitted to the NCMB only on January 18,
Grooming Standards, it succeeded in 2002; therefore, the 7-day strike ban should
forcing the Hotel to choose between have prevented them from holding a strike
allowing its inappropriately hair styled until January 25, 2002. The concerted
employees to continue working, to the action committed by the Union on January
detriment of its reputation, or to refuse them 18, 2002 which resulted in the disruption of
work, even if it had to cease operations in the Hotels operations clearly violated the
affected departments or service units, which above-stated mandatory periods.
in either way would disrupt the operations of
the Hotel. This Court is of the opinion, Last, the Union committed illegal acts in the
therefore, that the act of the Union was not conduct of its strike. The NLRC ruled that
merely an expression of their grievance or the strike was illegal since, as shown by the
displeasure but, indeed, a calibrated and pictures1[21] presented by the Hotel, the
calculated act designed to inflict serious Union officers and members formed human
damage to the Hotels finances or its barricades and obstructed the driveway of
reputation. Thus, we hold that the Unions the Hotel. There is no merit in the Unions
concerted violation of the Hotels Grooming argument that it was not its members but
Standards which resulted in the temporary the Hotels security guards and the police
cessation and disruption of the Hotels officers who blocked the driveway, as it can
operations is an unprotected act and should be seen that the guards and/or police
be considered as an illegal strike. officers were just trying to secure the
entrance to the Hotel. The pictures clearly
Second, the Unions concerted action which demonstrate the tense and highly explosive
disrupted the Hotels operations clearly situation brought about by the strikers
violated the CBAs No Strike, No Lockout presence in the Hotels driveway.
provision.

Third, the Union officers and members


concerted action to shave their heads and

129
| Labor Relations-2A

ISSUE: What then are the consequent 91_QUESTIN


liabilities of the Union officers and members
BASCON, et al. v. COURT OF
for their participation in the illegal strike?
APPEALS, et al.
HELD: Regarding the Union officers and G.R. No. 144899 February 5, 2004
members liabilities for their participation in
the illegal picket and strike, Art. 264(a),
paragraph 3 of the Labor Code provides
that [a]ny union officer who knowingly FACTS:
participates in an illegal strike and any
worker or union officer who knowingly Bascon and Cole were employees of
participates in the commission of illegal acts Metro Cebu Community Hospital (MCCH),
during a strike may be declared to have lost and members of labor union NAMA-MCCH.
his employment status x x x. The law makes They held the positions of head nurse and
a distinction between union officers and nursing aide, respectively.
mere union members. Union officers may
NAMA-MCCH asked MCCH to
be validly terminated from employment for
renew their CBA, but NFL (NAMA-MCCH’s
their participation in an illegal strike, while
mother federation) opposed such move.
union members have to participate in and
Believing that their union was the certified
commit illegal acts for them to lose their
collective bargaining agent, the members
employment status. Thus, it is necessary for
and officers of NAMA-MCCH staged a
the company to adduce proof of the
series of mass actions inside MCCH’s
participation of the striking employees in the
premises. Meanwhile, DOLE issued two
commission of illegal acts during the strikes
certifications stating that NAMA-MCCH
was not a registered labor organization.
Clearly, the 29 Union officers may
NAMA-MCCH still filed a notice of strike; but
be dismissed pursuant to Art. 264(a), par. 3
the NCMB disregarded it.
of the Labor Code which imposes the
penalty of dismissal on any union officer Consequently, notices were
who knowingly participates in an illegal served on all union members, Bascon
strike. However, there is room for leniency and Cole included, asking them to explain
with respect to the Union members. It is in writing why they were wearing red and
pertinent to note that the Hotel was able to black ribbons and roaming around the
prove before the NLRC that the strikers hospital with placards. In their collective
blocked the ingress to and egress from the response, the employees, explained that
Hotel. But it is quite apparent that the Hotel wearing armbands and putting up
failed to specifically point out the placards was their answer to MCCH’s
participation of each of the Union members illegal refusal to negotiate with NAMA-
in the commission of illegal acts during the MCCH.
picket and the strike. For this lapse in
judgment or diligence, we are constrained to Subsequently, MCCH sent two
reinstate the 61 Union members. notices to Bascon and Cove: first, that they
were to be investigated for their activities
in the mass actions; and second, an order
that they should desist from participating in
the mass actions with a warning. Bascon
and Cove, however, denied receiving said
notices. Bascon and Cole were then
served notices terminating their
employment.

130
| Labor Relations-2A

Bascon and Cove, thus, filed a mantle of constitutional protection under


complaint for illegal dismissal against freedom of speech.
MCCH. The LA found that they were justly
dismissed pursuant to their participation in
the mass actions. On appeal, the NLRC
reversed the LA, and found that Bascon and As regards the appellate court’s
Cove were illegally dismissed. On appeal to finding that petitioners were justly
the CA, the appellate court held that terminated for gross insubordination or
Bascon and Cole were validly terminated willful disobedience, we find lacking the
for their gross insubordination or willful element of willfulness characterized by a
disobedience. Hence, this petition. perverse mental attitude on the part of
petitioners in disobeying their employer’s
order as to warrant the ultimate penalty
of dismissal. Wearing armbands and
ISSUE: Were Bascon and Cove validly putting up placards to express one’s
dismissed for (1) allegedly participating in views without violating the rights of third
an illegal strike and/or (2) gross parties, are legal per se and even
insubordination to the order to stop constitutionally protected.
wearing armbands and putting up
placards. Finally, even if willful
disobedience may be properly
appreciated, still, the penalty of dismissal
is too harsh. Not every case of willful
RULING: disobedience by an employee of a
lawful work connected order of the
NO. employer may be penalized with
As to the first ground, while a union dismissal. Wearing armbands to signify
officer can be terminated for mere union membership and putting up
participation in an illegal strike, an placards to express their views cannot
ordinary striking employee, like be of such great dimension as to
petitioners herein, must have participated warrant the extreme penalty of dismissal.
in the commission of illegal acts during
the strike (italics supplied). There must
be proof that they committed illegal acts
during the strike. But proof beyond
reasonable doubt is not required.
Substantial evidence, which may justify
the imposition of the penalty of
dismissal, may suffice.

In this case, the CA found that


petitioners’ actual participation in the
illegal strike was limited to wearing
armbands and putting up placards. There
was no finding that the armbands or the
placards contained offensive words or
symbols. Thus, neither such wearing of
armbands nor said putting up of
placards can be construed as an illegal
act. In fact, per se, they are within the

131
| Labor Relations-2A

92_DEL ROSARIO asserts that the filing of a petition to declare


the strike illegal was unnecessary since the
JACKBILT INDUSTRIES INC. vs NLRC, in its July 17, 1998 decision, had
JACKBILT EMPLOYEES WORKERS already found that respondent committed
UNION-NAFLU-KMU illegal acts in the course of the strike.
GR No. 171618-19 March 20, ISSUE: Whether or not the filing of a
2009 petition with the labor arbiter to declare a
strike illegal is a condition sine qua non for
FACTS: the valid termination of employees who
Petitioner Jackbilt Industries (the commit an illegal act in the course of such
Company), due to the 1997 Asian Financial strike
Crisis, decided to temporarily stop its HELD: No, filing a petition before
business of producing concrete hollow Labor Arbiter to declare a strike illegal is not
blocks, compelling most of its employees to necessary for the valid termination of the
go on leave for six months. The Union employees.
alleged that such act was motivated by anti-
union sentiments since the CBA was The principle of conclusiveness of
expiring during the period of the shutdown; judgment, embodied in Section 47(c), Rule
hence, the Company is merely avoiding its 39 of the Rules of Court, holds that the
duty to bargain collectively. Accordingly, the parties to a case are bound by the findings
Union went on strike. Its officers and in a previous judgment with respect to
members picketed petitioner’s main gates matters actually raised and adjudged
and deliberately prevented persons and therein.
vehicles from going into and out of the
compound. Article 264(e) of the Labor Code
prohibits any person engaged in picketing
As a response to the Company’s from obstructing the free ingress to and
petition for injunction NLRC issued a TRO egress from the employer’s premises. Since
directing the respondents to refrain from respondent was found in the July 17, 1998
preventing access to petitioner’s property. decision of the NLRC to have prevented the
However respondent union violated the free entry into and exit of vehicles from
order. Meanwhile, the Company dismissed petitioner’s compound, respondent’s officers
the concerned officers and members of the and employees clearly committed illegal
Union for committing illegal acts in the acts in the course of the March 9, 1998
course of the strike. Hence, the Union filed strike. Therefore, pursuant to the principle of
complaints for illegal lockout, runaway shop conclusiveness of judgment, the March 9,
and damages, unfair labor practice, illegal 1998 strike was ipso facto illegal. The filing
dismissal and attorney’s fees and refusal to of a petition to declare the strike illegal was
bargain. thus unnecessary.
In a decision dated October 15, Consequently, we uphold the
1999, the labor arbiter dismissed the legality of the dismissal of respondent’s
complaints for illegal lockout and unfair officers and employees. Article 264 of the
labor practice for lack of merit. However, Labor Code further provides that an
because petitioner did not file a petition to employer may terminate employees found
declare the strike illegal before terminating to have committed illegal acts in the course
respondent’s officers and employees, it was of a strike. Petitioner clearly had the legal
found guilty of illegal dismissal. NLRC right to terminate respondents’ officers and
affirmed the decision of labor arbiter with employees.
modification. In its defense, the Company

132
| Labor Relations-2A

(a) PABX transfer and contractualization


of PABX service and position;
93_DIOLA
(b) Massive contractualization;

PHILCOM EMPLOYEES UNION VS (c) Flexible labor and additional


PHILIPPINE GLOBAL work/function;
COMMUNICATIONS and PHILCOM
CORPORATION (d) Disallowance of union leave
intended for union seminar;
G.R. No. 144315 July 17, 2006
(e) Misimplementation and/or non-
Carpio, J. implementation of employees' benefits
like shoe allowance, rainboots,
Doctrine: Unfair labor practice refers to raincoats, OIC shift allowance, P450.00
acts that violate the workers' right to monthly allowance, driving allowance,
organize. The prohibited acts are related motorcycle award and full-time
to the workers' right to self-organization physician;
and to the observance of a CBA.
Without that element, the acts, no (f) Non-payment, discrimination and/or
matter how unfair, are not unfair labor deprivation of overtime, restday work,
practices. waiting/stand by time and staff
meetings;

(g) Economic inducement by promotion


Facts: during CBA negotiation;

 The Collective Bargaining (h) Disinformation scheme, surveillance


Agreement (CBA) between and interference with union affairs;
petitioner Philcom Employees
Union and the respondent (i) Issuance of memorandum/notice to
Philcom Corporation expired. employees without giving copy to union,
o the parties started change in work schedule at Traffic
negotiations for the Records Section and ITTO policies; and
renewal of their CBA in
July 1997 (j) Inadequate transportation allowance,
o While negotiations were water and facilities."
ongoing, PEU filed a
Notice of Strike with the o The company, then,
National Conciliation and suspended the ongoing
Mediation Board (NCMB) CBA negotiation
– National Capital Region  Ground: bargaining
 Ground: perceived deadlock
unfair labor practice o The union went for another
committed by the strike
company

133
| Labor Relations-2A

 At a conciliation conference held Held: 1.) The Strike was illegal having
at the NCMB-NCR office, the found that PEU's officers and members
parties agreed to consolidate the have committed illegal acts during the
two (2) Notices of Strike filed by strike.
the union and to maintain the 2.) Those acts [enumerated] do
status quo during the pendency not constitute unfair labor practices
of the proceedings under Article 248 of the Labor Code
o While the union and the Ratio:
company officers and 1.) On illegal strike:
representatives were  Philcom is engaged in a vital
meeting, the remaining industry (in this case, Philcom is
union officers and engaged with the communication
members staged a strike industry) protected by
at the company premises. Presidential Decree No. 823 (PD
 The company immediately filed a 823), as amended by Presidential
petition for the Secretary of Labor Decree No. 849, from strikes and
and Employment to assume lockouts.
jurisdiction over the labor dispute. o the striking employees
o Acting Labor Secretary violated the no-strike
Trajano: enjoining any policy of the State in
strike or lockout, whether regard to vital industries
threatened or actual,  The Secretary had already
directing the parties to assumed jurisdiction over the
cease and desist from dispute. Despite the issuance of
committing any act that the return-to-work orders dated
may exacerbate the 19 November and 28 November
situation/ 1997, the striking employees
 Secretary of Labor: The Union's failed to return to work and
Manifestation/Motion to Strike continued with their strike.
Out Portions of and Attachments o A return-to-work order is
in Philcom's Position Paper is immediately effective and
hereby denied for lack of merit. executory despite the filing
The Union's charges of unfair labor of a motion for
practice against the Company are reconsideration. It must be
hereby dismissed. strictly complied with even
 CA: Violations of CBAs, except during the pendency of
those gross in character, are any petition questioning its
mere grievances resolvable validity.
through the appropriate  PEU staged the strike in utter
grievance machinery or voluntary disregard of the grievance
arbitration as provided in the procedure established in the
CBAs CBA.
Issue/s: 1.) Was there an illegal strike? o A strike declared on the
2.) Was there unfair labor basis of grievances which
practice? have not been submitted
to the grievance

134
| Labor Relations-2A

committee as stipulated in labor practice to contract


the CBA of the parties is out work for reason of
premature and illegal. reduction of labor cost
2.) On ULP: through the acquisition of
 Unfair labor practices of the automatic machines.
employer are enumerated in  On the union's charge that
Article 248 of the Labor Code management disallowed
(check your codal, please) leave of union officers and
 In this case, do not fall members to attend union
under any of the prohibited seminar: this is belied by
acts defined and the evidence submitted by
enumerated in Article 248 the union itself. In a letter
of the Labor Code. to PEU's President, the
 The issues of company granted the
misimplementation or non- leave of several union
implementation of officers and members to
employee benefits, non- attend a seminar
payment of overtime and notwithstanding that its
other monetary claims, request to be given more
inadequate transportation details about the affair was
allowance, water, and left unheeded by the
other facilities, are all a union. Those who were
matter of implementation denied leave were urgently
or interpretation of the needed for the operation
economic provisions of the of the company.
CBA between Philcom and
PEU subject to the
grievance procedure.
o The Union failed to convincingly
show that there is flagrant and/or
malicious refusal by the
Company to comply with the
economic provisions stipulated in
the CBA.
 Guys, here inexplain kung
bakit hindi ULP yung acts
nun employer. Medyo
madami sya kung i-
enumerate. Examples na
lang, hehe:
 On contractualization and
economic inducement: the
acts of said company
qualify as a valid exercise
of management
prerogative. It is not unfair

135
| Labor Relations-2A

The grant of the employer of profit-


94_DIZON sharing benefits to the employees outside
the “bargaining unit” falls under the ambit of
WISE AND CO. INC vs. WISE & CO. INC its managerial prerogative. It appears to
EMPLOYEES UNIION-NATU ETC. have been done in good faith and without
G.R. No. L-87672 October 13, 1989 ulterior motive. More so when as in this
case there is a clause in the CBA where the
employees are classified into those who are
FACTS: members of the union and those who are
When the management introduced a not. In the case of the union members, they
profit-sharing scheme for its managers and derive their benefits from the terms and
supervisors, the union wrote the conditions of the CBA which constitutes the
management to ask that the union members law between the contracting parties. Both
be allowed to participate in the profit- the employer and the union members are
sharing program. The management denied bound by such agreement.
the request on the ground that such However, the Court serves notice
participation was not provided in the CBA. that it will not hesitate to strike down any act
Later, when the renegotiation of the of the employer that tends to be
CBA was approaching, the management discriminatory against union members. It is
wrote the union that it was willing to only because of the peculiar circumstances
consider including the union members in the of this case showing there is no such
profit- sharing scheme if the negotiations intention that this court has ruled otherwise.
would be concluded before December 1987.
On March 30, 1988, the company
distributed the profit- sharing benefit not
only to managers and supervisors but also
to all rank-and-file employees not covered
by the CBA because they were excluded
from their agreed definition of bargaining
unit, such as the regular rand-and-file
employees in the office of the president,
vice president, security office, corporate
affairs office, accounting and treasury
department.

ISSUE:
W/N the grant by the management of profit-
sharing benefits to its employees who are
non-union members is discriminatory
against the union members which amounts
to ULP.

RULING:
No. There can be no discrimination
committed by the employer as the situation
of the union employees is different from the
non union employees. Discrimination per se
is not unlawful. There can be no
discrimination where employees concerned
are not similarly situated.

136
| Labor Relations-2A

95_DOÑA In Republic Savings Bank vs. CIR,


where the dismissed employees had written
DABUET, ET AL. v. ROCHE a letter decried by the bank as patently
PHARMACEUTICALS, INC. libellous for alleging immorality, nepotism
and favouritism on the part of the bank
G.R. No. L-45402 April 30, 1987 president, thus amounting to behaviour
Subject: ULP: Interference (Article 258[a]) necessitating their dismissal, the Court
declared the dismissal illegal as the letter
FACTS: was a concerted activity protected by R.A.
875.
The individual petitioners, who were
all officers of the Roche Products Labor In the present case, the letter written
Union, wrote respondent company by and for the union addressed to the
expressing the grievances of the union and management referred to employee
seeking a formal conference with the grievances and/ or labor management
management regarding the previous issues and the employees concerned were
dismissal of the union’s president and vice- all officers of the union, then seeking a
president. At the meeting, the company’s renegotiation of the CBA, a fact which
general manager, instead of discussing the respondent company does not deny, there
problems affecting the labor union and should, all the more, be a recognition of
management, allegedly berated the such letter as an act for the mutual aid,
petitioners for writing that letter and called protection and benefit of the employees
the letter and the person who prepared it concerned. This recognition in turn should
“stupid”. extend to petitioners’ execution of an
affidavit in support of the charge of slander
Feeling that he was the one alluded against private respondent, for calling the
to, since he had prepared the letter, the union’s lawyer, who prepared the letter, and
counsel for labor union filed a case for the contents thereof as “stupid”.
grave slander against the general manager.
The charge was based on the affidavit Breach of trust and confidence, the
executed by the petitioners. The company grounds alleged for petitioners’ dismissal
and the manager, in turn, filed a complaint must not be indiscriminately used as a
for perjury against petitioners alleging that shield to dismiss an employee arbitrarily.
their affidavit contained false statements. Respondent company’s act in dismissing
the petitioners, who then constituted the
The respondent company, remaining and entire officialdom of the
furthermore, construed the execution by Roche Products Labor Union, after the
petitioners of the affidavit as an act of union’s president and vice-president had
breach of trust and confidence and inimical been earlier dismissed and when the CBA
to the interest of the company, for which the in the company was about to be
latter were suspended and later on renegotiated, was an ULP under Sec. 4(a)
dismissed. (1) of the Industrial Peace Act. Their
dismissal, under the circumstances,
ISSUE: Whether respondent company is amounted to interference with, and
guilty of unfair labor practice. restraint or coercion of, the petitioners in
the exercise of their right to engage in
HELD:
concerted activities for their mutual aid
Yes. The respondent company had and protection.
committed ULP practice in dismissing the
petitioners from employment without just
and valid cause.

137
| Labor Relations-2A

96_DUQUILLA secure clearances from the fiscal’s office


and to be screened by a management
INSULAR LIFE ASSURANCE CO. committee, of which Enaje and Garcia were
EMPLOYEES ASSOCIATION-NATU, et al. members. When all the strikers had secured
vs. INSULAR LIFE ASSURANCE CO., et clearance, the companies readmitted only
some and refused readmission to some
G.R. No. L-25291 January 30, 1971 officials and members of the unions who
FACTS: Petitioner unions, represented by were most active in the strike, on the ground
lawyers Enaje and Garcia, entered into that they committed acts inimical to the
separate CBAs with respondent companies. interest of the companies. A complaint for
The unions later on joined the National unfair labor practice was filed against the
Association of Trade Unions (NATU) and companies.
the two lawyers were hired by the ISSUE: Whether or not the companies are
companies. Thereafter, negotiations were guilty of unfair labor practice for (1)
conducted on the unions’ proposals for a interfering with the members of the Unions
modified renewal of their respective CBAs in the exercise of their right to concerted
but these were snagged by a deadlock on action, by sending out individual letters to
the issue of union shop. Instead of making them urging them to abandon their strike
any counter-proposal, the companies and return to work, with a promise of
insisted that the unions first drop their comfortable cots, free coffee and movies,
demand for union security, promising and paid overtime, and, subsequently, by
money benefits if this was done. Despite the warning them that if they did not return to
unions having conceded to the demand, the work, they might be replaced; and (2)
companies still failed to make any counter- discriminating against the members of the
proposal on the economic demands. Thus, Unions as regards readmission to work after
the unions voted to declare a strike in the strike on the basis of their union
protest. Meanwhile, 87 unionists were membership and degree of participation in
reclassified as supervisors without increase the strike.
in salary nor in responsibility; thus, these
employees resigned from the unions. The HELD: YES. The Unions went on strike
unions then went on strike and picketed the because of the unfair labor practices
offices. The companies sent a letter to each committed by the respondents, and that
of the strikers, promising certain benefits if when the strikers reported back for work —
they came back to work voluntarily. The upon the invitation of the respondents —
unions, however, continued on strike, they were discriminatorily dismissed. The
prompting some management men to break members and officials of the Unions
through the unions’ picket lines. Alleging therefore are entitled to reinstatement with
that some non-strikers were injured, the back pay. Discriminatorily dismissed
companies filed criminal charges against employees must receive back pay from the
the strikers, with all but three dismissed by date of the act of discrimination, that is, from
the fiscal’s office, and a petition for the date of their discharge.
injunction with damages. Another letter was
sent individually to the strikers giving them (1) It is an unfair labor practice for an
until a certain date to return to their jobs or employer operating under a collective
else be replaced. With such ultimatum and bargaining agreement to negotiate or to
the issuance of the writ of preliminary attempt to negotiate with his employees
injunction, the striking employees decided to individually in connection with changes in
call off their strike and to report back to the agreement. The letters, directed to the
work. However, before readmitting the striking employees individually without being
strikers, the companies required them to coursed to the unions, is an active

138
| Labor Relations-2A

interference with the right of collective 97_EISMA


bargaining. Such practice tends to
undermine the concerted activity of the SHELL OIL WORKERS’ UNION v. SHELL
employees. Indeed, when the respondents CO. and COURT OF INDUSTRIAL
offered reinstatement and attempted to RELATIONS
"bribe" the strikers with certain benefits, so G.R. No. L-28607. May 31, 1971
they would abandon the strike and return to
work, they were guilty of strikebreaking Non-compliance with collective bargaining
and/or union-busting and, consequently, of agreement constitutes unfair labor
unfair labor practice. Moreover, applying the practice. Belief in good faith that employer
“totality of conduct doctrine”2, the actuations committed unfair labor practice renders
of the companies before and after the strike legal.
issuance of the letters yield the clear
inference that the said letters formed of the FACTS:
companies’ scheme to preclude if not Dispute arose when Shell transferred 18
destroy unionism within them. security guards to its other department and
the consequent hiring of a private security
(2) The companies committed a clear act of agency to undertake the work of said
discrimination in the process of rehiring. The security guards. Shell’s Union went on a
respondents did not merely discriminate strike due to the insistence of Shell to
against all the strikers in general. They dissolve its security guard section in
separated the active from the less active Pandacan notwithstanding its continuance
unionists on the basis of their militancy, or assured by the existing collective bargaining
lack of it, on the picket lines. Unionists contract. While Shell insists that it a valid
belonging to the first category were refused exercise of its management prerogative and
readmission even after they were able to such strike be declared illegal, Union claims
secure clearances from the competent that the transfers and eventual dismissals
authorities with respect to the criminal were illegal being in violation of the existing
charges filed against them. There is an contract. Union now claims to be reinstated
unfair labor practice where the employer, with full back wages.
although authorized to dismiss the
employees who participated in an illegal Prior to the strike, it appears that a joint
strike, dismissed only the leaders of the consultation by the Union and management
strikers, such dismissal being evidence of was undertaken and at that time, no serious
discrimination. Moreover, the mere act of opposition was noted with regard to the
placing in the hands of the employees move for dissolution. Complication started
hostile to the strikers the power of when a CBA was executed containing the
reinstatement is a form of discrimination in usual grievance procedure and no strike
hiring. Equally significant is the fact that clauses and the inclusion of security guards
while the management and the members of in the contract. Needless to say, a strike
the screening committee admitted the was declared by the Union. Conciliation
discrimination committed against the efforts were unsuccessful. It was not until
strikers, they tossed back and around to the Presidential certification came and a
each other the responsibility for the return to work order was issued by CIR, the
discrimination. respondent Court. By virtue of which
pending its resolution, Shell was not to
lockout the employees involved and the
2 The culpability of an employer's remarks were employees in turn were not to strike.
to be evaluated not only on the basis of their
implicit implications, but were to be appraised
against the background of and in conjunction ISSUE: Whether the then existing collective
with collateral circumstances. bargaining contract running for three years

139
| Labor Relations-2A

constituted a bar to such a decision reached dissolution without thereby incurring a


by management. violation of the collective bargaining
agreement.
HELD: Yes.
Shell, in failing to manifest fealty to what
There was specific coverage concerning the was stipulated in an existing collective
security guard section in the collective bargaining contract, was thus guilty of an
bargaining contract. It is found not only in unfair labor practice.
the body but in the two appendices
concerning the wage schedules as well as
the premium pay and the night
compensation to which the personnel in
such section were entitled. It was thus an
assurance of security of tenure, at least,
during the lifetime of the agreement. It was
not right defense to say that the guards
would not be unemployed as they would be
transferred to another position with an
increase in pay and with a transfer bonus.
For what is involved is the integrity of the
agreement reached, the terms of which
should be binding of both parties.

What renders the stand of Shell even more


vulnerable is the fact that as set forth in its
brief and as found by respondent Court as
far back as 1964, it had already been
studying the matter of dissolving the
security guard section and contracting out
such service to an outside agency.
Apparently, it had reached a decision to that
effect for implementation the next year. In
July 1966, there was a joint consultation
between it and the Union on the matter.
Nonetheless on August 26, 1966, a
collective bargaining contract was entered
into which, as indicated above, did assure
the continued existence of the security
guard section. The Shell did not have to
agree to such a stipulation. Or it could have
reserved the right to effect a dissolution and
reassign the guards. It did not do so.
Instead, when it decided to take such a step
resulting in the strike, it would rely primarily
on provisions in the collective bargaining
contract couched in general terms, merely
declaratory of certain management
prerogatives. Considering the
circumstances of record, there can be no
justification then for Shell's insistence on
pushing through its project of such

140
| Labor Relations-2A

98_FABON Ionics contended that it was an entity


separate and distinct from Complex and had
COMPLEX ELECTRIC v. NLRC been in existence for 8 years before the
labor dispute arose at Complex. Like
G.R. No. 121315 July 19, 1999 Complex, it was also engaged in the semi-
FACTS: conductor business where the machinery,
equipment and materials were consigned to
Complex Electronics Corporation was them by their customers.
engaged in the manufacture of electronic
products. It was actually a subcontractor of The President of Complex was also the
electronic products where its customers President of Ionics. Ionics denied having
gave their job orders, sent their own Qua as their owner since he had no
materials and consigned their equipment to recorded subscription of P1,200,00.00 in
it. Ionics as claimed by the Union. Ionics
further argued that the hiring of some
The rank and file workers of Complex were displaced workers of Complex was an
organized into a union known as the exercise of management prerogatives.
Complex Electronics Employees
Association (the Union). Complaint was filed for unfair labor practice
and illegal closure/illegal lockout, among
Complex received a fax message from Lite- others. The Union alleged that the pull-out
On Philippines Electronics Co., requiring it of the machinery, equipment and materials
to lower its price by 10%. Complex informed from the company premises, which resulted
its Lite-On personnel that such request of to the sudden closure of the company was
lowering their selling price by 10% was not in violation of the Labor Code, and the
feasible as they were already incurring existing CBA.
losses at the present prices of their
products. Complex regretfully informed the ISSUE:
employees that it was left with no alternative W/N there was unfair labor practice.
but to close down the operations of the Lite-
On Line. HELD:
NO.
Complex filed a notice of closure of the Lite-
On Line with the DOLE and the
A "runaway shop" is defined as an industrial
retrenchment of the 97 affected employees.
plant moved by its owners from one location
The Union filed a notice of strike with the to another to escape union labor regulations
NCMB. or state laws, but the term is also used to
describe a plant removed to a new location
Thereafter, the machinery, equipment and in order to discriminate against employees
materials being used for production at at the old plant because of their union
Complex were pulled-out from the company activities. It is one wherein the employer
premises and transferred to the premises of moves its business to another location or it
Ionics Circuit, Inc. (Ionics) at Laguna. temporarily closes its business for anti-
Fearful that the machinery, equipment and union purposes. It is motivated by anti-
materials would be rendered inoperative union animus rather than for business
and unproductive due to the impending reasons.
strike of the workers, the customers ordered
their pull-out and their transfer to Ionics. Ionics was not set up merely for the purpose
Complex was compelled to cease of transferring the business of Complex. At
operations. the time the labor dispute arose at Complex,

141
| Labor Relations-2A

Ionics was already existing as an 99_FAJARDO


independent company.
MALAYANG SAMAHAN NG MGA
The Union failed to show that the primary MANGGAGAWA SA M. GREENFIELD
reason for the closure of the establishment ET.AL
was due to the union activities of the vs. HON. CRESENCIO RAMOS ET.AL
employees. G.R. No. 113907 February 28, 2000

The mere fact that one or more corporations FACTS:


are owned or controlled by the same or Malayang Samahan ng mga
single stockholder is not a sufficient ground Manggagawa sa M. Greenfield (MSMG), the
for disregarding separate corporate local union, is an affiliate of United Lumber
personalities. and General Workers of the Philippines
(ULGWP), the federation. In the CBA
Closure, therefore, was not motivated by the between the company and the union, there
union activities of the employees, but rather is a Union Security clause which requires
by necessity since it can no longer engage being a member of the union in good
in production without the much needed standing (payment of union dues and non-
materials, equipment and machinery. violation of union constitution and by-laws),
failure of which would result to termination
The determination to cease operation is a of employment of the erring employees or
prerogative of management that is usually member.
not interfered with by the State as no Subsequent issues lead to the
employer can be required to continue relationship between the local union and its
operating at a loss simply to maintain the affiliate federation to turn bitter. This leads
workers in employment. to the expulsion of the union officers for
alleged acts of disloyalty and violation of
Personal liability of Lawrence Qua- absence constitution and by-laws. The federation
of malice or bad faith, a stockholder or an then recommended to the employer the
officer of a corporation cannot be made termination of employment of these union
personally liable for corporate liabilities. officers pursuant to the Union Security
Clause. The employer heed to the request
of the federation because of the Notice of
Strike filed by latter. The company
immediately send notice of dismissal to the
employees concerned and on the same day
physically and bodily brought them out of
the company premises, while some were
refused entrance thereto.
The local union then filed a case for
unfair labor practice against the employer
consisting of union busting, illegal dismissal,
illegal suspension, interference in union
activities, discrimination, threats,
intimidation, coercion, violence and
oppression.

ISSUE:
WON the company is guilty of unfair
labor practice in dismissing petitioner
employees merely upon the labor

142
| Labor Relations-2A

federation’s demand for the enforcement of issue is an intra-union dispute is


the union security clause embodied in their erroneous. It is true that the issue of
CBA. expulsion of local union officers is
originally between the local union
HELD: and the federation, hence, intra-
The company is NOT guilty of unfair union in character, the issue was
labor practice however, the dismissal, later on converted into a termination
relying solely on the federation’s dispute when the company
recommendation, is rendered illegal. dismissed the petitioners from work
Union security clauses in collective without the benefit of a separate
bargaining agreements, if freely and notice and hearing.
voluntarily entered into, are valid and  The union officers were dismissed
binding. Dismissal of an employee by the because its acts intending to
company pursuant to a labor union’s disaffiliate from the federation. The
demand in accordance with a union security federation deemed it a violation of
agreement does not constitute unfair labor the constitution and by-laws. There
practice. But this dismissal is subject to the is no such provision prohibiting
requirement of due process, that is, notice disaffiliation, hence, the right to
and hearing prior to dismissal. This the exercise it must be upheld and must
company failed to comply. The dismissal is not be a ground for expulsion the in
done hastily and summarily eroding the the union.
employees’ right to due process, self-
organization and security of tenure.
The company terminated the
employees without conducting a separate
and independent investigation. It did not
inquire into the cause of the expulsion and
whether or not the federation had sufficient
grounds to effect the same. And even on
the assumption that the federation had valid
grounds to expel the union officers, due
process requires that these union officers
be accorded a separate hearing.
The right of an employee to be
informed of the charges against him and to
reasonable opportunity to present his side in
a controversy with either the company or his
own union is not wiped away by a union
security clause or a union shop clause in a
CBA. An employee is entitled to be
protected not only from a company which
disregards his rights but also from his own
union the leadership of which could yield to
the temptation of swift and arbitrary
expulsion from membership and mere
dismissal from his job.
Ancillary Issue:
 The Labor Arbiter’s declaration that
the recourse of the complainants
should be against the federation and
not against the employer since the

143
| Labor Relations-2A

100_ indication that she has no intention to


abandon her work with the hotel. Even the
NORMA MABEZA vs. NATIONAL LABOR employer knows that his purported reason
RELATIONS COMMISSION, PETER of dismissing her due to abandonment will
NG/HOTEL SUPREME not fly so he amended his reply to indicate
[G.R. No. 118506. April 18, 1997] that it is actually “loss of confidence” that led
to Mabeza’s dismissal.
Norma Mabeza was an employee hired by Loss of Confidence
Hotel Supreme in Baguio City. In 1991, an It is true that loss of confidence is a valid
inspection was made by the Department of ground to dismiss an employee. But this is
Labor and Employment (DOLE) at Hotel ideally only applied to workers whose
Supreme and the DOLE inspectors positions require a certain level or degree of
discovered several violations by the hotel trust particularly those who are members of
management. Immediately, the owner of the the managerial staff. Evidently, an ordinary
hotel, Peter Ng, directed his employees to chambermaid who has to sign out for linen
execute an affidavit which would purport and other hotel property from the property
that they have no complaints whatsoever custodian each day and who has to account
against Hotel Supreme. Mabeza signed the for each and every towel or bedsheet
affidavit but she refused to certify it with the utilized by the hotel’s guests at the end of
prosecutor’s office. Later, when she her shift would not fall under any of these
reported to work, she was not allowed to two classes of employees for which loss of
take her shift. She then asked for a leave confidence, if ably supported by evidence,
but was not granted yet she’s not being would normally apply. Further, the
allowed to work. In May 1991, she then suspicious filing by Peter Ng of a criminal
sued Peter Ng for illegal dismissal. Peter case against Mabeza long after she initiated
Ng, in his defense, said that Mabeza her labor complaint against him hardly
abandoned her work. In July 1991, Peter Ng warrants serious consideration of loss of
also filed a criminal complaint against confidence as a ground of Mabeza’s
Mabeza as he alleged that she had stolen a dismissal.
blanket and some other stuff from the hotel.
Peter Ng went on to amend his reply in the
labor case to make it appear that the reason
why he dismissed Mabeza was because of
his loss of confidence by reason of the theft
allegedly committed by Mabeza. The labor
arbiter who handled the case, a certain
Felipe Pati, ruled in favor of Peter Ng.
ISSUE: Whether or not there is
abandonment in the case at bar. Whether or
not loss of confidence as ground for
dismissal applies in the case at bar.
HELD: No. The side of Peter Ng is bereft of
merit so is the decision of the Labor Arbiter
which was unfortunately affirmed by the
NLRC.
Abandonment
Abandonment is not present. Mabeza
returned several times to inquire about the
status of her work or her employment
status. She even asked for a leave but was
not granted. Her asking for leave is a clear

144

También podría gustarte